SBI P.O. Solved Paper - goldencareer.co.in

200
The information provided here is only for refrence. It may vary from the original. www.recruitment.guru SBI P.O. Solved Paper REASONING ABILITY 1. In a certain code language ‘how many goals scored’ is written as ‘5 3 9 7’; ‘many more matches’ is written as ‘9 8 2’ and ‘he scored five’ is written as ‘1 6 3’. How is ‘goals’ written in that code language ? (1) 5 (2) 7 (3) 5 or 7 (4) Data inadequate (5) None of these 2. In a certain code TEMPORAL is written as OLDSMBSP. How is CONSIDER written in that code? (1) RMNBSFEJ (2) BNMRSFEJ (3) RMNBJEFS (4) TOPDQDCH (5) None of these 3. How many meaningful English words can be made with the letters DLEI using each letter only once in each word ? (1) None (2) One (3) Two (4) Three (5) More than three 4. Among A, B, C, D and E each having different weight, D is heavier than only A and C is lighter than B and E. Who among them is the heaviest ? (1) B (2) E (3) C (4) Data inadequate (5) None of these 5. Each odd digit in the number 5263187 is substituted by the next higher digit and each even digit is substituted by the previous lower digit and the digits so obtained are rearranged in ascending order, which of the following will be the , third digit from the left end after the rearrangement ? (1) 2 (2) 4 (3) 5 (4) 6 (5) None of these 6. Pratap corrrectly remembers that his mother’s birthday is before twenty third April but after Nineteenth April, whert as his sister correctly remembers that their mother’s birthday is not on or after twenty second April. On which day in April is definitely their mother’s birthday ? (1) Twentieth (2) Twenty-first (3) Twentieth or twenty-first (4) Cannot be determined (5) None of these 7. Ashok started walking towards South. After walking 50 meters he took a right turn and walked 30 meters. He then took a right turn and walked 100 meters. He again took a right turn and walked 30 meters and stopped. How far and in which direction was he from the starting point? (1) 50 meters South (2) 150 meters North (3) 180 meters East (4) 50 meters North (5) None of these 8. If’–‘ means ‘+’; ‘–’means × ; × means ‘÷’and +’means ‘–’; then 15 – 8 × 6 + 12 + 4 = ? (1) 20 (2) 28 (3) 4 8 7 (4) 2 3 (5) None of these 9. Town D is towards East of town F. Town B is towards North of town D. Town H is towards South of town B. Towards which direction is town H from town F ? (1) East (2) South-East (3) North-East (4) Data inadequate (5) None of these Held on: 18-04-10

Transcript of SBI P.O. Solved Paper - goldencareer.co.in

The information provided here is only for refrence. It may vary from the original.

www.recruitment.guru

SBI P.O.Solved Paper

REASONING ABILITY

1. In a certain code language ‘how many goalsscored’ is written as ‘5 3 9 7’; ‘many morematches’ is written as ‘9 8 2’ and ‘he scored five’is written as ‘1 6 3’. How is ‘goals’ written inthat code language ?(1) 5 (2) 7 (3) 5 or 7(4) Data inadequate(5) None of these

2. In a certain code TEMPORAL is written asOLDSMBSP. How is CONSIDER written in thatcode?(1) RMNBSFEJ (2) BNMRSFEJ(3) RMNBJEFS (4) TOPDQDCH(5) None of these

3. How many meaningful English words can bemade with the letters DLEI using each letter onlyonce in each word ?(1) None (2) One (3) Two(4) Three (5) More than three

4. Among A, B, C, D and E each having differentweight, D is heavier than only A and C is lighterthan B and E. Who among them is the heaviest ?(1) B (2) E (3) C(4) Data inadequate(5) None of these

5. Each odd digit in the number 5263187 issubstituted by the next higher digit and each evendigit is substituted by the previous lower digitand the digits so obtained are rearranged inascending order, which of the following will bethe, third digit from the left end after therearrangement ?

(1) 2 (2) 4 (3) 5

(4) 6 (5) None of these

6. Pratap corrrectly remembers that his mother’s

birthday is before twenty third April but after

Nineteenth April, whert as his sister correctly

remembers that their mother’s birthday is not on

or after twenty second April. On which day in

April is definitely their mother’s birthday ?

(1) Twentieth (2) Twenty-first

(3) Twentieth or twenty-first

(4) Cannot be determined

(5) None of these

7. Ashok started walking towards South. After

walking 50 meters he took a right turn and

walked 30 meters. He then took a right turn and

walked 100 meters. He again took a right turn

and walked 30 meters and stopped. How far and

in which direction was he from the starting point?

(1) 50 meters South (2) 150 meters North

(3) 180 meters East (4) 50 meters North

(5) None of these

8. If’–‘ means ‘+’; ‘–’means ‘×’; ‘×’ means ‘÷’and‘+’means ‘–’; then 15 – 8 × 6 + 12 + 4 = ?

(1) 20 (2) 28 (3)4

87

(4) 2 3 (5) None of these

9. Town D is towards East of town F. Town B is

towards North of town D. Town H is towards

South of town B. Towards which direction is

town H from town F ?

(1) East (2) South-East

(3) North-East (4) Data inadequate

(5) None of these

Held on:18-04-10

The information provided here is only for refrence. It may vary from the original.

www.recruitment.guru

10. How many such pairs of letters are there in the

word SEARCHES each of which has as many

letters between them in the word as in the English

alphabet ?

(1) None (2) One (3) Two

(4) Three (5) More than three

Direction: In each of the questions below are given

four statements followed by four conclusions

numbered I, II, III 8s IV. You have to take the given

statements to be true even if they seem to be at given

conclusions logically follows from the given

statements disregarding commonly known facts.

11. Statements: All cups are bottles. Some bottles

are jugs. No jug is plate. Some plates are tables.

Conclusions:

I. Some tables are bottles.

II. Some plates are cups.

III. No table is bottle.

IV. Some jugs are cups.

(1) Only I follows (2) Only II follows

(3) Only III follows (4) Only IV follows

(5) Only either I or III follows

12. Statements: Some chairs are handles. All

handles are pots. All pots are mats. Some mats

are buses.

Conclusions:

I. Some buses are handles.

II. Some mats are chairs.

III. No bus is handle.

IV. Some mats are handles.

(1) Only I, II and IV follow

(2) Only II, III and IV follow

(3) Only either I or III and II follow

(4) Only either I or III and IV follow

(5) Only either I or III and II and IV follow

13. Statements: All birds are horses. All horses are

tigers. Some tigers are lions. Some lions are

monkeys.

Conclusions:

I. Some tigers are horses.

II. Some monkeys are birds.

III. Some tigers are birds.

IV. Some monkeys are horses.

(1) Only I and III follow

(2) Only I, II and III follow

(3) Only II, III and IV follow

(4) All I, II, III and IV follow

(5) None of these

14. Statements: Some benches are walls. All walls

are houses. Some houses are jungles. All jungles

are roads.

Conclusions:

I. Some roads are benches.

II. Some jungles are walls.

III. Some houses are benches.

IV. Some roads are houses.

(1) Only land II follow

(2) Only I and III follow

(3) Only III and IV follow

(4) Only II, III and IV follow

(5) None of these

15. Statements: Some sticks are lamps. Some

flowers are lamps. Some lamps are dresses. All

dresses are shirts.

Conclusions:

I. Some shirts are sticks.

II. Some shirts are flowers.

III. Some flowers are sticks.

IV. Some dresses are sticks.

(1) None follows (2) Only I follows

(3) Only II follows (4) Only III follows

(5) Only IV follows

Direction: Study the following information

carefully and answer the questions given below:

A, B, C, D, E, F, G and H are eight employees

of an organization working in three departments viz.

Personnel, Administration and Marketing with not

more than three of them in any department. Each of

them has a different choice of sports from football,

cricket, volleyball, badminton, lawn tennis, basketball,

hockey and table tennis not necessarily in the same

order.

D works in Administration and does not like

either football or cricket. F works in Personnel with

only A who likes table tennis. Eand H do not work in

the same department as D. C likes hockey and does

The information provided here is only for refrence. It may vary from the original.

www.recruitment.guru

not work in marketing. G does not work in

administration and does not like either cricket or

badminton. One of those who work in administration

likes football. The one who likes volleyball works in

personnel. None of those who work in administration

likes either badminton or lawn tennis. H does not like

cricket.

16. Which of the following groups of employees

work in Administration department ?

(1) EGH (2) AF (3) BCD

(4) BGD (5) Data inadequate

17. In which department does E work ?

(1) Personnel (2) Marketing

(3) Administration (4) Data inadequate

(5) None of these

18. Which of the following combinations of

employee-department-favourite sport is correct?

(1) E – Administration – Cricket

(2) F – Personnel – Lawn Tennis

(3) H – Marketing – Lawn Tennis

(4) B – Administration – Table Tennis

(5) None of these

19. What is E’s favourite sport ?

(1) Cricket (2) Badminton

(3) Basketball (4) Lawn Tennis

(5) None of these

20. What is G’s favourite sport ?

(1) Cricket (2) Badminton

(3) Basketball (4) Lawn Tennis

(5) None of these

Direction: In the following questions, the symbols

# and S are used with the following meaning as

illustrated below.

P $ Q’ means ‘P is not smaller than Q’.

P @ Q’ means ‘P is neither smaller than nor equal to ‘Q

‘P # Q’ means ‘P is neither greater than nor equal to Q’.

P δ Q’ means ‘P is neither greater than nor smaller than

Q’.

‘P Q’ means ‘P is not greater than Q’.

Now in each of the following questions

assuming the given statements to be true, find which

of the four conclusions I, II, III and IV given below

them is/are definitely true and give your answer

accordingly.

21. Statements: H @ T, T # F, F δ E, E VV

Conclusions: I. V $ F II. E @ T

III. H @ V IV. T # V

(1) Only I, II and III are true

(2) Only I, II and IV are true

(3) Only II, III and IV are true

(4) Only I, III and IV are true

(5) All I, II, III and IV are true

22. Statements: D#R, R K, K@F, F$J

Conclusions: I. J # R II. J # K

III. R # F IV. K @ D

(1) Only I, II and III are true

(2) Only II, III and IV are true

(3) Only I, III and IV are true

(4) All I, II, III and IV are true

(5) None of these

23. Statements: N δ B, B $ W, W # H, H M

Conclusions : I. M @ W II. H @ N

III. W S N IV. W# N

(1) Only I is true (2) Only III is true

(3) Only IV is true

(4) Only either III or IV is true

(5) Only either III or IV and I are true

24. Statements: R D, D $ J, J # M, M @ K

Conclusions: I. K # J II. D @ M

III. R # M

IV. D @ K

(1) None is true (2) Only I is true

(3) Only II is true (4) Only III is true

(5) Only IV is true

25. Statements: M $ K, K @ N, N R, R # WW

Conclusions: I. W @ K II. M $ R

III. K @ W

IV. M @ N

(1) Only I and II are true

(2) Only I, II and III are true

(3) Only III and IV are true

(4) Only II, III and IV are true

(5) None of these

The information provided here is only for refrence. It may vary from the original.

www.recruitment.guru

Direction: Study the following information

carefully and answer the questions given below

Following are the conditions for selecting Senior

Manager-Credit in a bank. The candidate must—

(i) be a graduate in any discipline with atleast 60

percent marks.

(ii) have post qualification work experience of at

least ten years in the Advances Section of a bank.

(iii) be at least 30 years and not more than 40 years

as on 1.4.2010. (iv) have secured at least 40

percent marks in the group discussion. (v) have

secured at least 50 percent marks in interview.

In the case of a candidate who satisfies all the

conditions EXCEPT—

(A) at (i) above but has secured at least 50 percent

marks in graduation and at least 60 percent marks

in post graduation in any discipline the case is

to be referred to the General Manager Advances.

(B) at (ii) above but has total post qualification work

experience of at least seven years out of which

at least three years as Manager-Credit in a bank,

the case is to be referred to Executive

Director.

In each question below details of one candidate

is given. You have to take one of the following

courses of action based on the information

provided and the conditions and sub-conditions

given above and mark the number of that course

of action as your answer. You are not to assume

anything other than the information provided in

each question. All these cases are given to you

as on 01.04.2010.

Give answer

(1) if the case is to be referred to Executive

Director.

(2) if the case is to be referred to General

Manager-Advances.

(3) if the data are inadequate to take a decision.

(4) if the candidate is not to be selected.

(5) if the candidate is to be selected.

26. Shobha Gupta has secured 50 percent marks in

the Interview and 40 percent marks in the Group

Discussion. She has been working for the past

eight years out of which four years as Manager-

Credit in a bank after completing her B. A. degree

with 60 percent marks. She was born on 12th

September 1978.

27. Rohan Maskare was born on 8th March 1974. He

has been working in a bank for the past twelve years

after completing his B.Com. degree with 70 percent

marks. He has secured 50 percent marks in both

the Group Discussion and the Interview.

28. Prakash Gokhale was born on 4th August 1977.

He has secured 65 percent marks in. post

graduation and 58 percent marks in graduation.

He has been working for the past ten years in

the Advances Department of a bank after

completing his post graduation. He has secured

45 percent marks in the Group Discussion and

50 percent marks in the Interview.

29. Sudha Mehrotra has been working in the

Advances department of a bank for the past

twelve years after completing her B.Com. degree

with 60 percent marks. She has secured 50

percent marks in the Group Discussion and 40

percent marks in the Interview. She was born on

15th February 1972.

30. Amit Narayan was born on 28th May 1974. He

has been working in the Advances department of a

bank for the past eleven years after completing his

B.Sc. degree with 65 percent marks. He has secured

55 percent marks in the Group discussion and 50

percent marks in the interview.

Direcion: In each question below is given a

statement followed by three courses of action

numbered (A), (B) and (C). A course of action is a

step or administrative decision to be taken for.

improvement, follow-up or further action in regard

to the problem, policy, etc. On the basis of the

information given in the statement, you have to

assume everything in the statement to be true, then

decide which of the suggested courses of action

logically follow(s) for pursuing.

31. Statement : A heavy unseasonal downpour

during tho last two days has paralysed the,

normal life in the state in which five persons were

The information provided here is only for refrence. It may vary from the original.

www.recruitment.guru

killed but this has provided a huge relief to the

problem of acute water crisis in the state.

Courses of action:

(A) The state government should set up a

committee to review the alarming situation.

(B) The state government should immediately

remove all the restrictions, on use of potable

water in all the major cities in the state.

(C) The state government should send relief

supplies to all the affected areas in the state.

(1) None (2) Only (A)

(3) Only (B) and (C) (4) Only (C)

(5) All (A), (B), (C)

32. Statement: A large private bank has decided to

retrench one-third of its employees in view of

the huge losses incurred by it during the past

three quarters.

Courses of action:

(A) The Govt. should issue a notification to

general public to immediately: stop all

transactions with the bank.

(B) The Govt. should direct the bank to refrain

from retrenching its employees.

(C) The Govt. should ask the central bank of the

country to initiate an enquiry into the bank’s

activities and submit its report.

(1) None (2) Only (A)

(3) Only (B) (4) Only (C)

(5) Only (A) and (C)

33. Statement: Many political activists have

decided to stage demonstrations and block traffic

movement in the city during peak hours to protest

against the steep rise in prices of essential

commodities.

Courses of action:

(A) The Govt. should immediately ban all forms

of agitations in the country.

(B) The police authority of the city should deploy

additional forces all over the city to help

traffic movement in the city.

(C) The state administration should carry out

preventive arrests of the known criminals

staying in the city.

(1) Only (A) (2) Only (B)

(3) Only (C) (4) Only (A) and (B)

(5) None of these

34. Statement: The school dropout rate in many

districts in the state has increased sharply during

the last few years as the parents of these children

make them work in the fields owned by others

to earn enough for them to get at least one meal

a day.

Courses of action:

(A) The Govt. should put up a mechanism to

provide foodgrains to the poor people in these

districts through public distribution system

to encourage the parents to send their wards

to school.

(B) The Govt. should close down some of these

schools in the district and deploy the teachers

of these schools to nearby schools and also

ask remaining students to join these schools.

(C) Govt. should issue arrest warrants for all the

parents who force their children to work in

fields instead of attending classes.

(1) Only (A) (2) Only (B)

(3) Only (C) (4) Only (A) and (B)

(5) None of these

35. Statement: One aspirant was killed due to

stampede while participating in a recruitment

drive of police constables.

Courses of action:

(A) The officials incharge of the recruitment

process should immediately be suspended.

(B) A team of officials should be asked to find

out the circumstances which led to the death

of the aspirant and submit its report within a

week.

(C) The Govt. should ask the home department

to stagger the number of aspirants over more

number of days to avoid such incidents in

future.

(1) Only (A) (2) Only (B)

(3) Only (C) (4) Only (B) and (C)

(5) None of these

36. Effect: Majority of the employees of the ailing

organization opted for voluntary retirement

scheme and left the organization with all their

The information provided here is only for refrence. It may vary from the original.

www.recruitment.guru

retirement benefits within a fortnight of

launching the scheme.

Which of the following can be a probable cause

of the above effect ?

(1) The company has been making huge losses

for the past five years and is unable to pay

salaryto its employees in time.

(2) The management of the company made huge

personal gains through unlawful activities.

(3) One of the competitors of the company went

bankrupt last year.

(4) The company owns large tracts of land in the

state which will fetch huge sum to its owners.

(5) None of these

37. Statement: Most of the companies in IT and

ITES sectors in India have started hiring from

engineering college campuses this year and are

likely to recruit much more than yearly

recruitment of the earlier years.

Which of the following substantiates the facts

stated in the above statement ?

(1) IT and ITES are the only sectors in India

which are hiring from engineering college

campuses.

(2) Govt. has stepped up recruitment activities

after a gap of five years.

(3) The IT and ITES companies have now

decided to vi sit the engineering college

campuses for tier II cities in India as well.

(4) Availability of qualified engineers will

substantially in crease in the near future.

(5) None of these

38. Cause: The Govt. has recently increased its taxes

or, petrol and diesel by about 10 percent. Which

of the following can be a possible effect of the

above cause ?

(1) The petroleum companies will reduce the

prices of petrol and diesel by about 10

percent.

(3) The petroleum companies will increase the

prices of petrol and diesel by about 5 percent.

(4) The petrol pumps will stop selling petrol and

diesel till the taxes are rolled back by the govt.

(5) None of these

39. Statement : The Govt. has decided to instruct

the banks to open new branches in such a way

that there is one branch of any of the banks in

every village of population 1000 and above or a

cluster of villages with population less than 1000

to provide banking services to all the citizens.

Which of the following will weaken the step

taken by the Govt. ?

(1) The private sector banks in India have

stepped up their branch expansion activities

in rural India.

(2) Many Govt. owned banks have surplus

manpower in its urban branches.

(3) All the banks including those in private sector

will follow the govt. directive.

(4) Large number of branches of many Govt.

owned banks in the rural areas are making

huge losses every year due to lack of adequate

business activities.

(5) None of these

Direction: Study the following information

carefully and answer the questions given below.

The centre reportedly wants to continue

providing subsidy to consumers for cooking gas and

kerosene for five more years. This is not good news

from the point of view of reining in the fiscal deficit.

Mounting subventions for subsidies means diversion

of savings by the government from investment to

consumption, raising the cost of capital in the process.

The government must cut expenditure on subsidies

to create more fiscal space for investments in both

physical and social infrastructure. It should outline a

plan for comprehensive reform in major subsidies

including petroleum, food and fertilizers and set goal

posts.

40. Which of the following is a conclusion which

can be drawn from the facts stated in the above

paragraph ?

(1) Subsidy provided by the government under

various heads to the citizen increases the cost

of capital.

(2) Govt. is unable to withdraw subsidies

provided to various items.

The information provided here is only for refrence. It may vary from the original.

www.recruitment.guru

(3) Govt. subsidy on kerosene is purely apolitical decision.

(4) Govt. does not have enough resources tocontinue providing subsidy on petroleumproducts.

(5) None of these

41. Which of the following is an inference whichcan be made from the facts stated in the aboveparagraph ?(1) India’s fiscal deficit is negligible in

(omparison to other emerging economies inthe world.

(2) Subsidy on food and fertilizers are essentialfor growth of Indian economy.

(3) Reform in financial sector will weakenIndia’s position in the international arena.

(4) Gradual withdrawal of subsidy is essentialfor effectively managing fiscal deficit inIndia.

(5) None of these

42. Which of the following is an assumption whichis implicit in the facts stated in the aboveparagraph ?(1) People in India may not be able to pay more

for petroleum products.(2) Many people in India are rich enough to buy

petroleum products at market cost.(3) Govt. may not be able to create more

infrastructural facilities if the present levelof subsidy continues for a longer time.

(4) Govt. of India has sought assistance frominternational financial organizations for itsinfrastructural projects

(5) None of these

Direcion: Study the following information carefully

and answer the questions given below.

Poverty measurement is an unsettled issue, bothconceptually and methodologically. Since poverty isa process as well as an outcome; many come out of itwhile others may be falling into it. The net effect ofthese two parallel processes is a proportion commonlyidentified as the ‘head count ratio’, but these ratioshide the fundamental dynamism that characterizespoverty in practice. The most recent povertyreestimates by an expert group has also missed the

crucial dynamism. In a study conducted on 13,000households which represented the crucial dynamism.In a study conducted on 13,000 households whichrepresented the entire country in 1993-94 and againon 2004-05, it was found that in the ten-year period18.2% rural population moved out of poverty whereasanother 22.1 % fell into it over this period. This netincrease of about four percentage points was seen tohave a considerable variation across states andregions.

43. Which of the following is a conclusion whichcan be drawn from the facts stated in the aboveparagraph ?(1) Accurate estimates of number of people

living below poverty line in India is possibleto be made.

(2) Many expert groups in India are notinterested to measure poverty objectively.

(3) Process of poverty measurement needs totake into account various factors to tackle itsdynamic nature.

(4) People living below poverty line remain inthat position for a very long time.

(5) None of these

44. Which of the following is an assumption whichis implicit in the facts stated in the aboveparagraph ?(1) It may not be possible to have an accurate

poverty measurement in India.(2) Level of poverty in India is static over the

years.(3) Researchers avoid making conclusions on

poverty measurement data in India.(4) Govt. of India has a mechanism to measure

level of poverty effectively and accurately.(5) None of these

45. Which of the following is an inference whichcan be made from the facts stated in the aboveparagraph ?(1) Poverty measurement tools in India are

outdated.(2) Increase in number of persons falling into

poverty varies considerably across thecountry over a period of time.

(3) Govt. of India has stopped measuring poverty

related studies.

The information provided here is only for refrence. It may vary from the original.

www.recruitment.guru

(4) People living in rural areas are more

susceptible to fall into poverty over the time.

(5) None of these

Direction: In each of the questions given below

which one of the five answer figures on the right

should come after the problem figures on the left,

if the sequence were continued ?

46. Problem Figure

Answer Figure

1 2 3 4 5

47. Problem Figure

Answer Figure

1 2 3 4 5

48. Problem Figure

A O T

A

T T

A

T AZ

TA

Answer Figure

1 2 3 4 5

49. Problem Figure

A

A

A

A

A K K

K

K K

O OO O OO O

Answer Figure

A A

A A

A K

KKKK O O OOOOOO

1 2 3 4 550. Problem Figure

D D

D

D DZ

Z

Z Z Z

O O

O

O O

Answer Figure

D

D

D D D

Z Z Z

Z

ZO O O

O

O

1 2 3 4 5

DATA ANALYSIS AND INTERPRETATION

Direction: Study the following table carefully to answer the questions that follow

Number (n) of candidates (in lakhs) appearing for an entrance examination From six different states

and the percentage (p) of candidates clearing the same over the years

STATE A B C D E F

YEAR N P N P N P N P N P N P

2004 1.23 42 1.04 51 1.11 32 1.32 24 1.23 36 1.33 31

2005 1.05 43 1.12 62 1.07 47 1.15 49 1.18 55 1.24 24

2006 2.04 38 1.48 32 1.08 28 1.96 35 1.42 49 1.58 26

2007 1.98 41 2.07 43 1.19 30 1.88 46 1.36 47 1.79 29

2008 1.66 53 1.81 50 1.56 42 1.83 60 1.73 57 1.86 34

2009 1.57 39 1.73 36 1.64 52 2.01 56 1.69 55 1.95 37

The information provided here is only for refrence. It may vary from the original.

www.recruitment.guru

51. What is the respective ratio of total number of

candidates clearing the entrance exam from State

B in the year 2004 to those clearing the entrance

exam from State C in the same year

(1) 221: 148 (2) 218: 143

(3) 148: 221 (4) 143: 218

(5) None of these

52. In which year did the highest number of candidates

clear the entrance exam from State D ?

(1) 2008 (2) 2006

(3) 2009 (4) 2007

(5) None of these

53. What is the number of candidates not clearing

the entrance exam from State A in the year 2007?

(1) 186820 (2).11682

(3) 1868200 (4) 1.16820

(5) None of these

54. What is the total numberof candidates clearing

the entrance exam from States E and F together

in the year 2006 ?

(1) 16160 (2) 110660

(3) 1.1066 (4) 1106600

(5) None of these

55. What is the average number of candidates

appearing for the entrance exam from State D in

the years 2007, 2008 and 2009 together ?

(1)2

1.9073

(2)1

186663

(3)1

1.8663

(4)2

1906663

(5) None of these

Direction: Study the given information carefully

and answer the questions that follow

An urn contains 6 red, 4 blue, 2 green and 3

yellow marbles.

56. If four marbles are picked at random, what is

the probability that at least one is blue ?

(1)4

15(2)

69

91(3)

11

15

(4)22

91(5) None of these

57. If two marbles are picked at random, what is the

probability that both are red ?

(1)1

6(2)

1

3(3)

2

15

(4)2

5(5) None of these

58. If three marbles are picked at random, what is

the probability that two are blue and one is

yellow?

(1)3

91(2)

1

5(3)

18

455

(4)7

15(5) None of these

59. If four marbles are picked at random, what is

the probability that one is green, two are blue

and one is red ?

(1)24

455(2)

13

35(3)

11

15

(4)7

91(5) None of these

60. If two marbles are picked at random, what is the

probability that either both are green or both are

yellow ?

(1)5

91(2)

1

35(3)

1

3

(4)4

105(5) None of these

Direction: Study the given pie-charts carefully to

answer the questions that follow

Breakup of number of employees working

in different departments of an organisation,

the, number of males and the number of

employees who recently got promoted in

each department break-up of employees

working in different departments total

number of employees = 3,600 Employees

working in different departments:

The information provided here is only for refrence. It may vary from the original.

www.recruitment.guru

Accounts20%

Production35%

Marketing18%

HR12%

IT15%

Break-up of number of males in each

department

Total number of males in the organisation

= 2,040

Break-up of number of males working in

each department

Accts

. 5%

Marketing

15%

HR 10%

IT 20%

Production50%

Break-up of number of employees who

recently got promoted in each department

Total number of employees who got

promoted = 1,200

Number of employees who recently got

promoted from each department

Acc

ou

nt

8%

Marketing22%

HR 1

1%

IT 26%

Production33%

61. If half of the number of employees who got

promoted from the IT department were males,

what was the approximate percentage of males

who got promoted from the IT department ?

(1) 61 (2) 29 (3) 54

(4) 42 (5) 38

62. What is the total number of females working in

the Production and Marketing departments

together ?(1) 468 (2) 812 (3) 582(4) 972 (5) None of these

63. How many females work in the Accountsdepartment ?(1) 618 (2) 592 (3) 566(4) 624 (5) None of these

64. The total number of employees who gotpromoted from all the departments together waswhat percent of the total number of employeesworking in all the departments together ?(Rounded off to the nearest integer)(1) 56 (2) 21 (3) 45(4) 33 (5) 51

65. The number of employees who got promotedfrom the HR department was what percent ofthe total number of employees working in thatdepartment ? (rounded off to two digits afterdecimal)(1) 36.18 (2) 30.56 (3) 47.22(4) 28.16 (5) None of these

Direction: Study the graph carefully to answer thequestions that follow

Percent rise in profit of two companies over the years40

35

30

25

20

15

10

5

0

Pro

fit P

erc

en

t

2004 2005 2006 2007 2008 2009

Company L Company M

66. If the profit earned by Company L in the year

2005 was Rs. 1.84 lakhs, what was the profit

earned by the company in the year 2006 ?

(1) Rs. 2.12 lakhs (2) Rs. 2.3 lakhs

(3) Rs. 2.04 lakhs

(4) Cannot be determined

(5) None of these

The information provided here is only for refrence. It may vary from the original.

www.recruitment.guru

67. If the profit earned by Company M in the year

2008 was Rs. 3.63 lakhs, what was he amount

of profit earned by it in the year 2006 ?

(1) Rs. 2.16 lakhs (2) Rs.1.98 lakhs

(3) Rs. 2.42 lakhs

(4) Cannot be determined

(5) None of these

68. What is the average percent rise in profit of

Company L over all the years together ?

(1)1

153

(2)1

253

(3)5

186

(4)5

216

(5) None of these

69. Which of the following statements is TRUE with

respect to the above graph ?

(1) Company M made the highest profit in the

year 2009

(2) Company L made least profit in the year 2005

(3) The respective ratio between the profits

earned by Company L and M In the year 2006

was 6:5

(4) Company L made the highest profit in the

year 2008

(5) All are true

70. What is the percentage increase in percent rise

in profit of Company M in the year 2009 from

the previour year ?

(1) 25 (2) 15 (3) 50

(4) 75 (5) None of these

Direction: Study th a information carefully to

answer the questions that follow.

A school consisting of a total of 1560 students

has boys and girls in the ratio of 7:5 respectively. All

the students are enrolled in different types of hobby

classes, viz: Singing, Dancing and Painting. One-fifth

of the boys are enrolled in only Dancing classes.

Twenty percent of the girls are enrolled in only

Painting classes. Ten percent of the boys are enrolled

in only Singing classes. Twenty four percent of the

girls are enrolled in both Singing and Dancing classes

together. The number of girls enrolled in only Singing

classes is two hundred percent of the boys enrolled

in the same. One-thirteenth of the boys are enrolled

in all the three classes together. The respective ratio

of boys enrolled in Dancing and Painting classes

together to the girls enrolled in the same is 2 :1

respectively. Ten percent of the girls are enrolled in

only Dancing classes whereas eight percent of the girls

are enrolled in both Dancing and Painting classes

together. The remaining girls are enrolled in all the

three classes together. The number of boys enrolled

in Singing and Dancing classes together is fifty

percent of the number of girls enrolled in the same.

The remaining boys are enrolled in only Painting

classes.

71. What is the total number of boys who are

enrolled in Dancing ?

(1) 318 (2) 364

(3) 292 (4) 434

(5) None of these

72. Total number of girls enrolled in Singing is

approximately what percent of the total number

of students in the school ?

(1) 37 (2) 19

(3) 32 (4) 14

(5) 26

73. What is the total number of students enrolled in

all the three classes together ?

(1) 135 (2) 164

(3) 187 (4) 142

(5) None of these

74. Number of girls enrolled in only Dancing classes

is what percent of the boys enrolled in the same?

(rounded off to two digits after decimal)

(1) 38.67 (2) 35.71

(3) 41.83 (4) 28.62

(5) None of these

75. What is the respective ratio of the number of

girls enrolled in only Painting classes to the

number of boys enrolled in the same ?

(1) 77 : 26 (2) 21 : 73

(3) 26 : 77 (4) 73 : 21

(5) None of these

The information provided here is only for refrence. It may vary from the original.

www.recruitment.guru

76. What is the respective ratio between the profit

earned by shopkeeper U in the months

February-2010 and March - 2010 together to that

earned by shopkeeper Q in the same months ?

(1) 637: 512 (2) 621 : 508

(3) 512: 637 (4) 508: 621

(5) None of these

77. What is the percent increase in profit of

shopkeeper S in the month of December - 2009

over the previous month ? (rounded off to two

digits after decimal)

(1) 3.15 (2) 2.67

(3) 2.18 (4) 3.33

(5) None of these

78. Which shopkeeper’s profit kept increasing

continuously over the given months ?

(1) R (2) Q (3) T

(4) U (5) None of these

79. What is the difference in profit earned by

shopkeeper T in January - 2010 from the

previous month ?

(1) Rs. 640/- (2) Rs. 420/-

(3) Rs. 380/- (4) Rs. 760/-

(5) None of these

80. What was the average profit earned by

shopkeeper R in the months of October – 2009

and November – 2009 together ?

(1) 5405 (2) 5040 (3) 4825

(4) 4950 (5) None of these

Direction: Study the given graph carefully to answer

the questions that follow

Number of days taken by three carpenters to

finish making one piece each of four different

items of furniture

16

14

12

10

8

6

4

2

0

Chair Table Bed Cupboard

Company X Company Y Company Z

81. If Carpenter X and Carpenter Y were to make a

chair together how many days would they take?

(1) 1 day (2) 4 days (3) 3 days

(4) 2 days (5) None of these

82. If Carpenters X, Y and Z were to make a table

together how many days would they take ?

(1) 4 days (2) 3 days (3) 1 day

(4) 2 days (5) None of these

83. What is the total number of days that Carpenter

Z will take to make one piece each of all the

four items together ?

(1) 32 days (2) 24 days

(3)1

159

days (4)1

132

days

(5) None of these

Direction: Study the table carefully to answer the questions that follow.

Profit (in rs. ‘000) made by six different shopkeepers over the months

Month October November December January February March

Shopkeeper 2009 2009 2009 2010 2010 2010

P 5.25 6.04 5.84 6.10 5.95 6.02

Q 4.84 4.28 4.97 4.88 5.04 5.12

R 4.99 5.82 5.48 5.45 5.68 5.36

S 5.06 5.11 5.28 5.38 5.44 5.59

T 5.28 4.96 5.31 5.69 4.93 5.72

U 5.94 6.23 5.87 6.07 6.19 6.23

The information provided here is only for refrence. It may vary from the original.

www.recruitment.guru

84. The radius of a circular field is equal to the side

of a square field whose perimeter is 784 feet.

What is the area of the circular field ?

(1) 107914 Sq.ft (2) 120736 Sq.ft.

(3) 107362 Sq.ft. (4) 127306 Sq.ft.

(5) None of these

85. In how many different ways can the letters of

the word ‘STRESS’ be arranged

(1) 360 (2) 240

(3) 720

(4) 120

(5) None of these

86. Total number of people staying in locality J forms

approximately what percent of the total number

of people staying in locality F ?

(1) 81 (2) 72 (3) 78

(4) 93 (5) 87

87. What is the total number of children staying in

localities H and I together ?

(1) 1287 (2) 1278 (3) 1827

(4) 1728 (5) None of these

88. The number of women staying in which locality

is the highest ?

(1) H (2) J (3) F

(4) G (5) None of these

89. What is the total number of men and children

staying in locality I together

(1) 4115 (2) 4551 (3) 4515

(4) 4155 (5) None of these

90. What is the respective ratio of number of men

staying in locality F to the number of men staying

in locality H ?

(1) 517: 416 (2) 403: 522

(3) 416: 517 (4) 522: 403

(5) None of these

91. The compound interest earned by Suresh on acertain amount at the end of two years at the rateof 8 p.c.p.a was Rs. 1,414.4. What was the total

Direction: Study the given table carefully to answer the questions that follow

Number of people staying in five different localities and the percentage breakup of men, women and

children in them

LOCALITY TOTAL NO. PERCENTAGE

OF PEOPLE MEN WOMEN CHILDREN

F 5640 55 35 10

G 4850 34 44 22

H 5200 48 39 13

I 6020 65 25 10

J 4900 42 41 17

amount that Suresh got back at the end of twoyears in the form of principal plus interestearned?(1) Rs. 9,414.4 (2) Rs. 9,914.4(3) Rs. 9,014.4 (4) Rs. 8,914.4(5) None of these

92. The respective ratio of the present ages of amother and daughter is 7 : 1. Four years ago therespective ratio of their ages was 19:1. What willbe the mother’s age four years from now ?(1) 42 years (2) 38 years(3) 46 years (4) 36 years(5) None of these

93. Three friends J, K and Ljog around a circularstadium and complete one round in 12, 18 and20 seconds respectively. In how many minuteswill all the three meet again at the starting point(1) 5 (2) 8 (3) 12(4) 3 (5) None of these

94. 4 men can complete a piece of work in 2 days. 4women can complete the same piece of work in4 days whereas 5 children can complete the samepiece of work in 4 days. If, 2 men, 4 womenanW 10 children work together, in how manydays can the work be completed ?(1) 1 day (2) 3 days (3) 2 days(4) 4 days (5) None of these

The information provided here is only for refrence. It may vary from the original.

www.recruitment.guru

95. The speed of a boat when travelling downstream

is 32 Kms. / Hr. , whereas when travelling

upstream it is 28 kms/hr. What is the speed of

the boat in still water ?

(1) 27 Kms./Hr. (2) 29 Kms./ Hr.

(3) 31 Kms./ Hr.

(4) Cannot be determined

(5) None of these

Direction: Study the following tables carefully and

answer the questions given below

Number of Candidates appeared in a

CompetitiveExamination from five centres over

the years

NUMBER

Center Mumbai Delhi Kolkata Hydera ChennaiYear -bad

2001 35145 65139 45192 51124 37346

2002 17264 58248 52314 50248 48932

2003 24800 63309 56469 52368 51406

2004 28316 70316 71253 54196 52315

2005 36503 69294 69632 58350 55492

2006 29129 59216 64178 48230 57365

2007 32438 61345 563041 49178 58492

Approximate-Percentages of candidatesqualified

to appeared in the-Competitiveexamination from

five centres over the years

PERCENTAGE

Mumbai Delhi Kolkata Hyderabad Chennai

12 24 18 17 9

10 28 12 21 12

15 21 23 25 10

11 27 19 24 8

13 23 16 23 13

14 20 21 19 11

16 19 24 20 14

96. In which of the following years was the

difference in number of candidates appeared

from Mumbai over the previous year the

minimum ?

(1) 2004 (2) 2006 (3) 2007

(4) 2002 (5) None of these

97. In which of the following years was the number

of candidates qualified from Chennai, the

maximum among the given years ?

(1) 2007 (2) 2006 (3) 2005

(4) 2003 (5) None of these

98. Approximately what was the total number of

candidates qualified from Delhi in 2002 and

2006 together ?

(1) 27250 (2) 25230 (3) 30150

(4) 28150 (5) 26250

99. Approximately how many candidates appearing

from Kolkata in 2004 qualified in the

competitive examination ?

(1) 13230 (2) 13540 (3) 15130

(4) 15400 (5) 19240

100.Approximately what was the difference between

the number of candidates qualified from

Hyderabad in 2001 and 2002 ?

(1) 1680 (2) 2440 (3) 1450

(4) 2060 (5) 1860

GENERAL AWARENESS /

MARKETING / COMPUTERS

101. What is the full form of ‘NBFC’ as used in the

Fir ancial Sector ?

(1) New Banking Finance Company

(2) National Banking & Finance Corporation

(3) New Business Finance & Credit

(4) Non Business Fund Company

(5) None of these

102.100% concession has been given for travelling

in the Indian Railways for patients of ...

(1) AIDS (2) Cancer (3) Swine Flu

(4) T. B. (5) None of these

103.Many a times, we read about Special Drawing

Right (SDR) in newspapers. As per its definition,

SDR is a monetary unit of the reserve assets of

which of the following organizations / agencies?

(1) World Bank

(2) International Monetary Fund (IMF)

(3) Asian Development Bank

(4) Reserve Bank of India

(5) None of these

The information provided here is only for refrence. It may vary from the original.

www.recruitment.guru

104.Which of the following is/are the highlights of

the Union Budget 2010-11 ?

(A) Number of new steps taken to simplify the

Foreign Direct Investment (FDI) regime.

(B) Rs. 16,500 crore provided to ensure that the

Public Sector Banks are able to attain a

minimum 8% capital (Tier I) by March 2011.

(C) More than Rs. 1,74,000 crore provided for

the development of the infrastructure in the

country.

(1) Only A (2) Only B

(3) Only C (4) All A, B & C

(5) None of these

105.In how many Routes special tourist trains called

‘Bharat Tirth’ is to start?

(1) 19 (2) 16 (3) 17

(4) 18 (5) None of these

106.As per the newspaper reports, the Govt. of India

made an auction of the Third Generation

Spectrum (3G) recently. Which of the following

ministries was actively involved in the process a

(1) Ministry of Heavy Industries

(2) Ministry of Science & Technology

(3) Ministry of Commerce

(4) Ministry of Foreign Affairs

(5) None of these

107.As per the recent announcement, the Govt. of

India will provide an amount of Rs.48,000 crore

to develop Rural Infrastructure in the country.

This planned development is being undertaken

under which of the following schemes ?

(1) Bharat Nirman

(2) Indira Aawas Yojana

(3) Backward Region Grant Fund

(4) Drought Mitigation Fund

(5) None of these

108.Ladies special trains to be renamed with the

Name of ...

(1) Bharat Bhoomi Specials

(2) Sonia Gandhi Specials

(3) Matri Bhoomi Specials

(4) Rajiv Gandhi Specials

(5) None of these

109.How much funds has been allocated to the

Unique Identification Authority of India?

(1) Rs 1,500 Crore (2) Rs 1,900 Crore

(3) Rs 1,600 Crore (4) Rs 1,800 Crore

(5) None of these

110. What is the rate of Income Tax for incomesz

above Rs 1.6 lakh upto Rs 5 lakh?

(1) 10% (2) 12% (3) 15%

(4) 20% (5) None of these

111. Recently, India took part in “Nuclear New Build

2010 Conference” organized in -

(1) New Delhi (2) London

(3) Paris (4) Hong Kong

(5) None of these

112. What is the reduction in Service Charges on e-

tickets?

(1) Sleeper Class Rs. 20 & AC Class Rs. 10

(2) Sleeper Class Rs. 10 & AC Class Rs. 20

(3) Sleeper Class Rs. 30 & AC Class Rs. 20

(4) Sleeper Class Rs. 20 & AC Class Rs. 30

(5) None of these

113. As we know, with the launch of Nano by Tatas,

India has become favourite Small Car

Destination of the world. Other than India, which

of the following countries is also a popular

destination of small cars ?

(1) Britain (2) France

(3) Germany (4) Thailand

(5) None of these

114. Imports from China in the Year of 2008-09 in

Rs...

(1) 100,000 Crore (2) 140,000 Crore

(3) 147,605 Crore (4) 151,000 Crore

(5) None of these

115. How many new teams have been added in IPL

2010?

(1) 2 (2) 1 (3) 4

(4) 7 (5) None of these

116.Which countries have recently faced Tsunami

Waves?

(1) Japan & Chile (2) Sallie & Korea

(3) China & Thailand (4) Japan & China

(5) None of these

The information provided here is only for refrence. It may vary from the original.

www.recruitment.guru

117. North Korea and USA decided to resume their

peace talks after a gap of several months. Both

the countries have a dispute over which of the

following issues ?

(1) Bailout package offered by USA

(2) Membership of ASEAN to North Korea

(3) Nuclear programme of North Korea

(4) Soaring relations of China with North Korea

(5) None of these

118. Who amongst the following was the Chairperson

of the 13th Finance Commission which

submitted its report to the President of India

recently ?

(1) Mr. M. V. Kamath (2) Dr. C. Rangarajan

(3) Dr. D. Subbarao (4) Dr. Rakesh Mohan

(5) Dr. Vijay Kelkar

119.As per the news published in various

newspapers, the RBI is considering the grant of

licence to some new companies, particularly

NBFCs to act as full-fledged banks. Which of

the following will be considered NBFC ?

(1) NABARD

(2) Life Insurance Corporation of India

(3) Reliance Capital

(4) SEBI

(5) None of these

120.Who has scored the highest individual ODI

Score?

(1) Saeed Anwar (Pak)

(2) Charls Coventry (Zim)

(3) Sachin Tendulkar (IND)

(5) Ricky Pointing (Aus)

(5) None of these

121. What is an ‘intranet’

(1) Internal internet used to transfer information

internally

(2) Internal internet used to transfer information

to the outside company

(3) Internal network designed to serve the

internal informational needs of a single

organization

(4) Internal network designed to transfer the

information between two organizations

(5) None of these

122.Which of the following groups of cricket teams

was declared joint winner of the Col. C K Naidu

Trophy for 2009 ?

(1) Tamil Nadu & Gujarat

(2) Maharashtra & Kerala

(3) Punjab & Delhi

(4) West Bengal & Maharashtra

(5) None of these

123. Kaiane Aldorino who was crowned Miss World

2009 is from which of the following countries?

(1) Germany (2) Russia

(3) Austria (4) Belgium

(5) Gibraltar

124. Which of the following teams won the Davis Cup

Tennis Finals 2009 ?

(1) Germany (2) Spain

(3) Russia (4) France

(5) None of these

125. Who amongst the following is the recipient of

the “CNN-IBN Indian of the Year” Award for

2009?

(1) Ratan Tata (2) A.R.Rahman

(3) Manmohan Singh (4) Sachin Tendulkar

(5) None of these

126. The deficit reduction plan of which of the

following countries was reviewed recently in the

meeting of the Finance Ministers of the European

Union ?

(1) Germany (2) Romania

(3) Brazil (4) Hungary

(5) Greece

127. The database administrator’s function in an

organization is —

(1) To be responsible for the more technical

aspects of managing the information

contained in organizational databases

(2) To be responsible for the executive level

aspects of decisions regarding the

information management

(3) To show the relationship among entity classes

in a data warehouse

(4) To define which data mining tools must be

used to extract data

(5) None of these

The information provided here is only for refrence. It may vary from the original.

www.recruitment.guru

128.Every device on the Internet has a unique______

address (also called an “Internet address”) that

identifies it in the same way that a street address

identifies the location of a house.

(1) DH (2) DA (3) IP

(4) IA (5) None of these

129. To send another station a message, the main thing

a user has to know is

(1) how the network works

(2) the other station’s address

(3) whether the network is packet-switched or

circuit-switched

(4) whether this is a voice or data network

(5) None of these

130. In a client/ server model, a client orogram -

(1) asks for information

(2) provides information and files

(3) serves software files to other computers

(4) distributes data files to other computers

(5) None of these

131. Control in design of an information system is

used to —

(1) inspect the system and check that it is buiit

as per specifications

(2) ensure that the system processes data as it

was designed to and that the results are

reliable

(3) ensure privacy of data processed by it

(4) protect data from accidental or intentional

loss

(5) None of these

132. Each of the following is a true statement

except—

(1) on-line systems continually update the master

file

(2) in on-line processing, the user enters

transactions into a device that is directly

connected to the computer system

(3) batch processing is still used today in older

systems or in some systems with massive

volumes of transactions

(4) information in batch systems will always be

up-to-date”

(5) None of these

133.A set of interrelated components that collect,

process, store, and distribute information to

support decision making and control in an

organization best defines -

(1) communications technology

(2) a network

(3) an information system

(4) hardware

(5) None of these

134.Ais a computer connected to two networks.

(1) link (2) server

(3) gateway (4) bridge way

(5) None of these

135. When you save a presentation,

(1) all slides in a presentation are saved in the

same file

(2) two files are created; one for graphics and

one for content

(3) a file is created for each slide

(4) a file is created for each animation or graphic

(5) None of these

136.In a customer database, a customer’s surname

would be keyed into a —

(1) row (2) text field

(3) record (4) computed field

(5) None of these

137. Who is the new Prime Minister of Hungry ?

(1) Victor Orban (2) Gorden Bajnai

(3) Jeno Fock (4) Ference Gyurcsany

(5) None of these

138. Storing same data in many places is called

(1) iteration (2) concurrency

(3) redundancy (4) enumeration

(5) None of these

139.Which of the following is the first step in the

‘transaction processing cycle’, which captures

business data through various modes such as

optical scanning or at an electronic commerce

website ?

(1) Document and report generation

(2) Database maintenance

(3) Transaction processing

(4) Data Entry

(5) None of these

The information provided here is only for refrence. It may vary from the original.

www.recruitment.guru

140.CRM (Customer Relationship Management)

is—

(1) Apre-sales activity

(2) A tool for lead generation

(3) An ongoing daily activity

(4) The task of a DSA

(5) All of the above

141. Who is the new prime minister of Denmark ?

(1) Anders Fogh Rasmussen

(2) Lars Looke Rasmussen

(3) Poul Nyrup Rasmussen

(4) Poul Hartling

(5) None of these

142. Who is the Author of the book “My China Diary”

(1) Kanwal Sibal (2) Salman Haider

(3) J.N. Dixit (4) Natwar Singh

(5) None of these

143.One of the following is not involved in the

Growth Strategies of a Company -

(1) Horizontal integration

(2) Vertical integration

(3) Diversification

(4) Intensification (5) None of these

144.A successful “Blue Ocean Strategy” requires -

(1) Effective communication

(2) Innovative skills

(3) Motivation

(4) All of the above (5) None of these

145.Programs from the same developer , sold

bundled together , that provide better integration

and share common features , toolbars and menus

are known as ....

(1) software suites

(2) integrated software packages

(3) software processing packages

(4) personal information managers

(5) none of these

146.A data warehouse is which of the following ?

(1) Can be updated by the end users

(2) Contains numerous naming conventions and

formats

(3) Organized around important subject areas

(4) Contains only current data

(5) None of these

147.__________ servers store and manages files for

network users.

(1) Authentication (2) Main

(3) Web (4) File

(5) None of these

148.One of the following is not included in the 7 P’s

of Marketing. Find the same

(1) Product (2) Price

(3) Production (4) Promotion

(5) None of these

149. The target group for SME loans is -

(1) All Businessmen (2) All Professionals

(3) All SSIs (4) All of the above

(5) None of these

150. Home Loans can be best canvassed among -

(1) Builders (2) Flat owners

(3) Land developers (4) Agriculturists

(5) Individuals wanting to buy a flat or house

ENGLISH LANGUAGE

Direction: Read the following passage carefully and

answer the questions given below it. Certain words/

phrases have been printec in bold to help you locate

them while answering some of the questions.

Governments have traditionally equated

economic progress with steel mills and cement

factories. While urban centers thrive and city dwellers

get rich, hundreds of millions of farmers remain mired

in poverty. However, fears of food shortages, a

rethinking of antipoverty priorities and the crushing

recession in 2008 are causing a dramatic shift in world

economic policy in favour of greater support for

agriculture.

The last time when the world’s farmers felt such

love was in the 1970s. At that time, as food prices

spiked, there was real concern that the world was

facing a crisis in which the planet was simply unable

to produce enough grain and meat for an expanding

population. Governments across the developing world

and international aid organisations plowed investment

into agriculture in the early 1970s, while technological

breakthroughs, like high-yield strains of important

The information provided here is only for refrence. It may vary from the original.

www.recruitment.guru

food crops, boosted production. The result was the

Green Revolution and food production exploded.

But the Green Revolution became a victim of

its own success. Food prices plunged by some 60%

by the late 1980s from their peak in the mid- 1970s.

Policymakers and aid workers turned their attention

to the poor’s other pressing needs, such as health care

and education. Farming got starved of resources and

investment. By 2004, aid directed at agriculture sank

to 3.5% and “Agriculture lost its glitter.” Also, as

consumers in high-growth giants such as China and

India became wealthier, they began eating more meat,

so grain once used for human consumption got

diverted to beef up livestock. By early 2008, panicked

buying by importing countries and restrictions slapped

on grain exports by some big producers helped drive

prices upto heights not seen for three decades. Making

matters worse, land and resources got reallocated to

produce cash crops such as biofuels and the result was

that voluminous reserves of grain evaporated. Protests

broke out across the emerging world and fierce food

riots toppled governments.

This spurred global leaders into action. This

made them aware that food security is one of the

fundamental issues in the world that has to be dealt

with in order to maintain administrative and political

stability. This also spurred the U.S. which traditionally

provisioned food aid from American grain surpluses

to help needy nations, to move towards investing in

farm sectors around the globe to boost productivity.

This move helped countries become more productive

for themselves and be in a better position to feed their

own people.

Africa, which missed out on the first Green

Revolution due to poor policy and limited resources,

also witnessed a ‘change’. Swayed by the success of

East Asia, the primary poverty-fighting method

favoured by many policymakers in Africa was to get

farmers off their farms and into modern jobs in

factories and urban centers. But that strategy proved

to be highly insufficient. Income levels in the

countryside badly trailed those in cities while the FAO

estimated that the number of poor going hungry in

2009 reached an all time high at more than one billion.

In India on the other hand, with only 40% of its

farmland irrigated, entire economic boom currently

underway is held hostage by the unpredictable

monsoon. With much of India’s farming areas

suffering from drought this year, the government will

have a tough time meeting its economic growth

targets. In a report, Goldman Sachs predicted that if

this year too receives weak rains, it could cause

agriculture to contract by 2% this fiscal year, making

the government’s 7% GDP-growth target look “a bit

rich”. Another green revolution is the need of the hour

and to make it a reality, the global community still

has much backbreaking farm work to do.

151. What is the author’s main objective in writing

the passage

(1) Criticising developed countries for not

bolstering economic growth in poor nations

(2) Analysing the disadvantages of the Green

Revolution

(3) Persuading experts that a strong economy

depends on industr ialization and not

agriculture

(4) Making a case for the international society

to engineer a second Green Revolution

(5) Rationalising the faulty agriculture policies

of emerging countries

152.Which of the following is an adverse impact of

the Green Revolution ?

(1) Unchecked crop yields resulted in large tracts

of land becoming barren

(2) Withdrawal of fiscal impetus from agriculture

to other sectors

(3) Farmers began soliciting government

subsidies for their produce

(4) Farmers rioted as food prices fell so low that

they could not make ends meet

(5) None of these

153.What is the author trying to convey through the

phrase “making the government’s 7% GDP

growth target look “a bit rich” ?

(1) India is unlikely to achieve the targeted

growth rate

(2) Allocation of funds to agriculture has raised

India’s chances of having a high GDP

The information provided here is only for refrence. It may vary from the original.

www.recruitment.guru

(3) Agricultural growth has artificially inflated

India’s GDP and such growth is not real

(4) India is likely to rave one of the highest GDP

growth rates

(5) A large portion of India’s GDP is contributed

by agriculture

154.Which of the following factors was/were

responsible for the neglect of the farming sector

after the green revolution ?

(A) Steel and cement sectors generated more

revenue for the government as compared to

agriculture.

(B) Large scale protests against favouring

agriculture at the cost of other important

sectors such as education and healthcare.

(C) Attention of policy makers and aid

organizations was diverted from agriculture

to other sectors.

(1) None (2) Only (C)

(3) Only (B) & (C) (4) Only (A) 8s (B)

(5) All (A), (B) & (C)

155.What prompted leaders throughout the world to

take action to boost the agriculture sector in

2008?

(1) Coercive tactics by the U.S. which restricted

food aid to poor nations

(2) The realization of the link between food

security and political stability

(3) Awareness that performance in agriculture is

necessary in order to achieve the targeted

GDP

(4) Reports that high-growth countries like China

and India were boosting their agriculture

sectors to capture the international markets

(5) Their desire to influence developing nations

to slow down their industrial development.

156.What motivated the U.S. to focus on investing

in agriculture across the globe ?

(1) To make developing countries become more

reliant on U.S. aid

(2) To ensure grain surpluses so that the U.S. had

no need to import food

(3) To make those countries more self sufficient

to whom it previously provided food

(4) To establish itself in the market before the

high-growth giants such as India and China

could establish themselves

(5) None of these

157. What impact did the economic recession of 2008

have on agriculture ?

(1) Governments equated economic stability

with industrial development and shifted away

from agriculture

(2) Lack of implementation of several innovative

agriculture programmes owing to shortage of

funds

(3) It prompted increased investment and interest

in agriculture

(4) The GDP as targeted by India was never

achieved because of losses in agriculture

(5) None of these

158. What encouraged African policymakers to focus

on urban jobs ?

(1) Misapprehension that it would alleviate

poverty as it did in other countries

(2) Rural development outstripped urban

development in many parts of Africa

(3) Breaking out of protests in the country and

the fear that the government would topple

(4) Blind imitation of western models of

development

(5) None of these

159.Which of the following had contributed to

exorbitant food prices in 2008 ?

(A) Hoarding of food stocks by local wholesalers

which inadvertently created a food shortage.

(B) Export of foodgrains was reduced by large

producers.

(C) Diverting resources from cultivation of

foodgrains to that of more profitable crops.

(1) None (2) Only (C)

(3) Only (B) (4) All (A), (B) & (C)

(5) Only (B) & (C)

160.Which of the following is true about the state of

agriculture in India at present ?

(A) Of all the sectors, agriculture needs the

highest allocation of funds.

(B) Contribution of agriculture to India’s GDP

The information provided here is only for refrence. It may vary from the original.

www.recruitment.guru

this year would depend greatly upon the

monsoon rains.

(C) As India is one of the high-growth countries,it has s jrplus food reserves to export to othernations.

(1) Only (A) and (C) (2) Only (C)

(3) Only (B) (4) Only (B) and (C)(5) None of these

Direction: Choose the word/group of words which

is most similar it meaning to the word printed in

bold as used in the passage.

161. STARVED(1) Deprived (2) Disadvantaged(3) Hungry (4) Fasting(5) Emaciated

162. SLAPPED(1) Beaten (2) Imposed(3) Withdrawn (4) Avoided(5) Persuaded

163. PLOWED(1) Cultivated (2) Bulldozed

(3) Recovered (4) Instilled(5) Withdrew

Direction: Choose the word/phrase which is most

opposite in meaning to the word printed in bold as

used in the passage.

164. PRESSING(1) Unpopular (2) Undemanding(3) Unobtrusive (4) Unsuitable(5) Unimportant

165. EVAPORATED

(1) Absorbed (2) Accelerated(3) Grew (4) Plunged(5) Mismanaged

Direction: Which of the phrases (1), (2),(3) and (4)

given below each statement should be placed in the

blank space provided so as to make a meaningful

and grammatically correct sentence ? If none of

the sentences is appropriate, mark (5) i.e. ‘None of

these’ as the answer.

166.Refuting the rationale behind frequent agitations

for formation of separate States, a recent report

(1) proved that such agitations result in loss of

governmental property

(2) indicated that the formation of small states

does not necessarily improve the economy

(3) suggested that only large scale agitations havebeen effective in bringing out desired changein the past

(4) recommended dividing large States into

smaller ones to improve governance(5) None of these

167. Overlooking the fact that water scarcityintensifies during summer,(1) the government issued guidelines to all

builders to limit their consumption toacceptable limits

(2) provision for rainwater harvesting has beenmade to aid irrigation in drought prone areas

(3) the water table did not improve even afterreceiving normal monsoon in the current year

(4) many residential areas continue to useswimming pools, wasting large quantities ofwater

(5) None of these

168.He has lost most of his life’s earning in the stockmarket but

(1) He still seems to be leading his lifeluxuriously and extravagantly

(2) he could not save enough to repay hisenormous debts

(3) stock market is not a safe option to investmoney unless done with caution

(4) experts have been suggesting to avoidinvestments in stock market because of itsunpredictable nature

(5) None of these

169.Achieving equality for women is not only alaudable goal,

(1) political reforms are also neglectedpreventing women from entering legislaturesand positions of power

(2) the problem is also deep rooted in the societyand supported by it

(3) their empowerment is purposefully hampered

by people with vested interests in all sectionsof the society

The information provided here is only for refrence. It may vary from the original.

www.recruitment.guru

(4) it is also equally difficult to achieve and

maintain for a long term

(5) None of these

170. _______or else they would not keep electing him

year after year.

(1) The party leader gave a strong message to

the mayor for improving his political style

(2) Owing to numerous scandals against the

mayor, he was told to resign from the post

immed iately

(3) The mayor threatened the residents against

filing a complaint against him

(4) The residents must really be impressed with

the political style of their mayor

(5) None of these

Direction: Each question below has two blanks,

each blank indicating that something has been

omitted. Choose the set of words for each blank that

best fits the meaning of the sentence as a whole.

171. Drawing attention to the pitfalls of______solely

on Uranium as a fuel for nuclear reactors, Indian

scientists warned that Uranium will not last for

long and thus research on Thorium as its____

must be revived.

(1) using, substitute

(2) believing, replacement

(3) depending, reserve

(4) reckoning, option

(5) relying, alternative

172. In an effort to provide ______ for higher

education to all, most of the universities have

been providing education without adequate

infrastructure, thus churning out ______

graduates every year.

(1) chances, fresh

(2) platform, capable

(3) opportunities, unemployable

(4) prospects, eligible

(5) policy, incompetent

173.The move to allow dumping of mercury _____

an outcry from residents of the area who _____

that high levels of mercury will affect their health

and destroy ecologically sensitive forest area.

(1) resulted, insist

(2) provoked, fear

(3) incited, determined

(4) activated, accept

(5) angered, believe

174. _______ has been taken against some wholesale

drug dealers for dealing in surgical items without

a valid license and maintaining a stock of _____

drugs.

(1) Note, overwhelming

(2) Step, impressive

(3) Execution, outdated

(4) Action, expired

(5) Lawsuit, invalid

175. Even as the _____ else where in the world are

struggling to come out of recession, Indian

consumers are splurging on consumer goods and

to _____ this growth, companies are investing

heavily in various sectors.

(1) economies, meet

(2) countries, inhibit

(3) governments, measure

(4) nations, inflict

(5) companies, counter

Direction:Rearrange the following sentences (A),

(B), (C), (D), (E) and (F) to make a meaningful

paragraph and then answer the questions which

follow

(A) While these disadvantages of bio fuels are

serious, they are the only alternate energy source

of the future and the sooner we find solutions to

these problems the faster we will be able to solve

the problems wo are now facing with gasoline.

(B) This fuel can also help to stimulate jobs locally

since they are also much safer to handle thaw”

gasoline and can thus have the potential to

turnaround a global economy.

(C) These include dependence on fossil fuels for the

machinery required to produce biofuel which

ends up polluting as much as the burning of fossil

fuels on roads and exorbitant cost of biofuels

which makes it very difficult for the common

man to switch to this option.

The information provided here is only for refrence. It may vary from the original.

www.recruitment.guru

(D) This turnaround can potentially help to bring

world peace and end the need to depend on

foreign countries for energy requirements.

(E) Biofuels are made from plant sources and since

these sources are available in abundance and can

be reproduced on a massive scale they form an

energy source that is potentially unlimited.

(F) However everything is not as green with the

biofuels as it seems as there are numerous

disadvantages involved which at times

overshadow their positive impact.

176. Which of the following sentence should be the

FIFTH after rearrangement ?

(1) A (2) B (3) C

(4) E (5) F

177. Which of the following sentence should be the

THIRD after rearrangement ?

(1) A (2) B (3) C

(4) D (5) E

178. Which of the following sentence should be the

FIRST after rearrangement ?

(1) A (2) B (3) C

(4) D (5) E

179. Which of the following sentence should be the

SIXTH (LAST) after rearrangement ?

(1) A (2) C (3) D

(4) E (5) F

180. Which of the following sentence should be the

SECOND after rearrangement ?

(1) A (2) B (3) D

(4) E (5) F

Direction: Which of the phrases (1), (2), (3) and

(4) given below each statement should replace the

phrase printed in bold in the sentence to make it

grammatically correct ? If the sentence is correct

as it is given and ‘No correction is required’, mark

(5) as the answer.

181. Soon after the Tsunami had killed thousands of

people along the coasts of southern India,

parliament psssas a bill that proposed to set up

an institutional mechanism to respond promptly

to natural disasters.

(1) passed a bill that proposed

(2) passes a bill with purpose

(3) pass a bill proposing

(4) passed a bill which propose

(5) No correction required

182. Denial of wages forced scientists and teachers

at the agriculture universities throughout the

country to go on strike, crippling crucial

research that could help the state of agriculture

in the country.

(1) from going on strike

(2) which went on strike

(3) on going for a strike

(4) for going to strike

(5) No correction required

183. In an attempt to boost their profits many edible

oil producing companies have been engaging

themselves in propaganda against commonly

used oils and. promoting exotic and expensive

varieties of oils as more healthier options.

(1) as most healthiest options

(2) as less healthy option

(3) as a healthier option

(4) as much healthiest option

(5) No correction required

184. Thanks to numerous government initiatives, rural

masses which was earlier unaware of the

luxuries of urban ways of living are now

connected to the same lifestyle.

(1) who was earlier unaware

(2) which were earlier aware

(3) who were earlier conversant

(4) who were earlier unaware

(5) No correction required

185. Over the last few months, while most industries

are busy in restructuring operations, cutting costs

and firing, the Indian pharmaceutical and

healthcare industry was adding manpower and

giving salary hikes.

(1) as many industries are

(2) while most industries were

(3) while many industries is

(4) where many industries were

(5) No correction required

The information provided here is only for refrence. It may vary from the original.

www.recruitment.guru

Direction: In the following passage there are blanks,

each of which ]’as been numbered. These numbers

are printed below the passage and against each, five

words/phrases are suggested, one of which fits the

blank appropriately. Find out the appropriate word/

phrase in each case.

There is a considerable amount of research about

the factors that make a company innovate. So is it

possible to create an environment (186 ) to

innovation? This is a particularly pertinent (187) for

India today. Massive problems in health, education

etc. (188) be solved using a conventional approach

but (189) creative and innovative solutions that can

ensure radical change and (190). There are several

factors in India’s (191). Few countries have the rich

diversity that India or its large, young population

(192). While these (193) innovation policy

interventions certain additional steps are also required.

These include (194) investment in research and

development by (195) the government and the private

sector, easy transfer of technology from the academic

world etc. To fulfill its promise of beng prosperous

and to be at the forefront, India must be innovative.

186. (1) stimuli (2) conducive

(3) incentive (4) facilitated

(5) impetus

187. (1) objective (2) controversy

(3) doubt (4) question

(5) inference

188. (1) cannot (2) possibly

(3) should (4) never

(5) must

189. (1) necessary (2) apply

(3) need (4) consider

(5) requires

190. (1) quantity (2) advantages

(3) increase (4) chaos

(5) growth

191. (1) challenges (2) praises

(3) favour (4) leverage

(5) esteem

192. (1) blessed (2) enjoys

(3) endows (4) prevails

(5) occurs

193. (1) aid . (2) jeopardise

(3) promotes (4) endure

(5) cater

194. (1) acute (2) utilising

(3) restricting (4) inspiring

(5) increased

195. (1) both (2) besides

(3) combining (4) participating

(5) a ;o

Direction: In each of the following questions four

words are given of which two words are most nearly

the same or opposite in meaning. Find the two words

which are most nearly the same or opposite in

meaning and indicate the number of the correct

letter combination, by darkening the appropriate

oval in your answer sheet.

196.(A) consent (B) nascent

(C) emerging (D) insecure

(1) A–C (2) B–D

(3) B–C (4) A–D

(5) A–B

197.(A) elated (B) eccentric

(C) explicit (D) abnormal

(1) A–B (2) B–D

(3) A–C (4) A–D

(5) D–C

198.(A) abundance (B) incomparable

(C) projection (D) plethora

(1) A–C (2) A–B

(3) C–D (4) B–D

(5) A–D

199.(A) purposefully (B) inaccurately

(C) inadvertently (D) unchangeably

(1) A–C (2) A–B

(3) B–C (4) B–D

(5) A–D

200.(A) germane (B) generate

(C) reliable (D) irrelevant

(1) B–D (2) B–C

(3) A–B (4) C–D

(5) A–D

www.gradeup.co

2

Direction (1-10): Read the passage carefully

and answer the question that follow. Certain

words are printed in bold to help you locate

them while answering some of the question:

Technology can be most broadly defined as

the entities, both material and immaterial,

created by the application of mental and

physical effort in order to achieve some value.

In this usage, technology refers to tools and

machines that may be used to solve real-

world problems. It is a far-reaching term that

may include simple tools, such as a crowbar

or wooden spoon, or more complex machines,

such as a space station or particle accelerator.

Tools and machines need not be material;

virtual technology, such as computer software

and business methods, fall under this

definition of technology.

The word "technology" can also be used to

refer to a collection of techniques. In this

context, it is the current state of humanity's

knowledge of how to combine resources to

produce desired products, to solve problems,

fulfill needs, or satisfies wants; it includes

technical methods, skills, processes,

techniques, tools and raw materials. When

combined with another term, such as "medical

technology" or "space technology," it refers to

the state of the respective field's knowledge

and tools. "State-of-the-art technology" refers

to the high technology available to humanity

in any field.

Technology can be viewed as an activity that

forms or changes culture. Additionally,

technology is the application of math, science,

and the arts for the benefit of life as it is

known. A modern example is the rise of

communication technology, which has

lessened barriers to human interaction and,

as a result, has helped spawn new

subcultures; the rise of cyber culture has, at

its basis, the development of the Internet and

the computer. Not all technology enhances

culture in a creative way; technology can also

help facilitate political oppression and war

via tools such as guns. As a cultural activity,

technology predates both science and

engineering, each of which formalizes some

aspects of technological endeavour.

1. What is virtual technology, according to the

passage?

A. A space station or particle accelerator.

B. Computer software and business methods.

C. Technical methods, skills, processes,

techniques, tools and raw materials.

D. The state of the respective field's

knowledge and tools.

E. None of the above.

2. How do developments of technology bring out

cultural changes?

A. It has lessened barriers to human

interaction and, as a result, has helped spawn

new subcultures; the rise of cyber culture has,

at its basis, the development of the Internet

and the computer.

B. It combines resources to produce desired

products, to solve problems, fulfill needs, or

satisfies wants.

C. Technology refers to tools and machines

that may be used to solve real-world

problems.

D. As a cultural activity, technology predates

both science and engineering.

E. None of the above.

3. Which two fields have been stated as an

example which when combined with

technology state the respective field's

knowledge and tools?

A. Virtual technology, space technology.

B. Science, engineering technology.

C. Medical technology, space technology.

D. All of the above.

E. None of the above.

4. Pick out the word that is most nearly the

same in meaning as the word printed in BOLD

type, as used in the passage.

SPAWN

A. Generate B. Beget

C. Ruin D. Halt

E. None of these

5. Pick out the word that is most nearly the

same in meaning as the word printed in BOLD

type, as used in the passage.

OPPRESSION

A. Persecution B. Democracy

C. Emergency D. Fair

E. B and D.

6. Pick out the word that is most opposite in

meaning as the word printed in BOLD type, as

used in the passage.

www.gradeup.co

3

ENDEAVOR

A. Negligence B. Avocation

C. Contemplate D. Exertion

E. B and D.

7. Pick out the word that is most opposite in

meaning as the word printed in BOLD type, as

used in the passage.

PREDATES

A. Antedate B. Follow

C. Antecede D. Forego

E. None of the above

8. According to the passage, how is technology

beneficial for human being?

A. It helped spawn new subcultures.

B. Technology can be used to solve real-world

problems.

C. It has lessened barriers to human

interaction.

D. Only A and B

E. All the above

9. According to the passage, why is development

of technology referred to as a problem solving

tool?

A. It consists of material and immaterial

entities which help in problem solving.

B. It is a far-reaching term that includes

complex tools which help to solve the

problems.

C. It combines resources to produce desired

products and to solve problems.

D. Only B and D

E. All the above

10. According to the author, the central idea of

the passage is to….

A. Demonstrate award winning achievements

of technology.

B. Support space technology.

C. Criticize the technological changes.

D. Highlight the positive changes brought by

technology.

E. All the above.

Directions (11-20): In the passage given

below there are blanks, each followed by a

word given in the brackets. Every blank has

five alternative words given in options. Find

the word which best suits the respective

blank. If the given word suits the blank, mark

'no correction required' as the answer.

The widespread consternation over the rupee

hitting a 27-month low against the dollar is

unwarranted, for the Indian currency has

been among the better (###Q11###)

[hiking] currencies over the last couple of

years. While other (###Q12###) [trickling]

market currencies such as the Russian rouble

and the Brazilian real are down more than 20

per cent this year, the rupee is lower by just 6

per cent. This follows a strong performance in

2014, when the Indian currency lost just 1.2

per cent against the greenback. It is obvious

that the rupee is in a sweet spot

(###Q13###) [peculiar] to its emerging

market peers, which have been hit hard by

the (###Q14###) [ascent] in commodity

prices. India, on the other hand, has benefited

from this fall. The crash in crude prices

combined with the checks on gold imports

have helped (###Q15###) [recede] the

current account deficit to just 1.27 per cent of

GDP. Strong foreign inflows — from both

portfolio and direct investments — have

pushed India’s forex reserves to $351 billion;

we are among the few countries that

have (###Q16###) [considered] to

increase forex reserves since the middle of

last year. These reserves provide the Indian

central bank with (###Q17###)

[ammunition] to protect the rupee from short-

term volatility that may arise once the Federal

Reserve goes through with its long-anticipated

rate hike. Since the Fed has given financial

markets sufficient time to (###Q18###)

[discern] the move, a 25 basis points move is

not likely to cause too much turbulence. True,

some short-term money will flow out of the

equity markets; foreign portfolio investors

have (###Q19###) [turned] net sellers

since November. But long-term investors are

likely to stay put due to the better growth

(###Q20###) [contrariety] of Indian

companies. The superior real yield, falling

inflation and a stable rupee also make a

strong case for staying invested in Indian debt

instruments.

11. Choose the correct answer from the given

options to fill the blanks which are numbered.

A. moving B. performing

C. doing D. stagnating

E. No correction required

www.gradeup.co

4

12. Choose the correct answer from the given

options to fill the blanks which are numbered.

A. emerging B. on-going

C. suiting D. menacing

E. No correction required

13. Choose the correct answer from the given

options to fill the blanks which are numbered.

A. known B. employed

C. straight D. relative

E. No change required

14. Choose the correct answer from the given

options to fill the blanks which are numbered.

A. spree B. rise

C. plunge D. demolition

E. No correction required

15. Choose the correct answer from the given

options to fill the blanks which are numbered.

A. boost B. recover

C. maintain D. sustain

E. No correction required

16. Choose the correct answer from the given

options to fill the blanks which are numbered.

A. managed B. agitated

C. breached D. stood

E. No correction required

17. Choose the correct answer from the given

options to fill the blanks which are numbered.

A. gadget B. thing

C. product D. process

E. No correction required

18. Choose the correct answer from the given

options to fill the blanks which are numbered.

A. hinder B. assimilate

C. nullify D. ostracize

E. No correction required

19. Choose the correct answer from the given

options to fill the blanks which are numbered.

A. spanned B. produced

C. acquired D. raised

E. No correction required.

20. Choose the correct answer from the given

options to fill the blanks which are numbered.

A. features B. plight

C. matters D. prospects

E. No correction required

Directions (21-30): Which of the phrases

(A), (B), (C) and (D) given below each

sentence should replace the phrase printed

in bold type to make the sentence

grammatically correct. If the sentence is

correct as it is, mark (E), i.e., 'No correction

required' as the answer.

21. When we went to wonder of worlds, I loved

that really big old silver antique car that

was parked in the parking lot of the East India

Mall.

A. Really old silver big antique car

B. Really big silver old antique car

C. Really big silver antique car

D. Really big old silver antique car

E. No correction required

22. Each individual knows how to protected on

own life and it should be left to his discretion.

A. Protected by own

B. Protect by own

C. Protect his own

D. Protected his own

E. No change required

23. The last few decades has seen a great deal

of political instability in India.

A. Decade had seen

B. Decade has seen

C. Decades have seen

D. Decade have seen

E. No change required

24. He admired the speed with which Koti

completed the work and appreciating the

method adopted by him

A. appreciate the method being adopted

B. appreciated the method adopted

C. appreciate the method of adoption

D. appreciate the adopting method

E. No correction required

25. I had met him after the party where he had

been given an inspiring speech

A. when he had

B. where he would have

C. in which he was given

D. where he had

E. No correction required

26. After the success of our project we have been

receiving more requests than we do not

have the resources to handle them.

A. many requests but

B. most of the requests

C. more requests that

D. too many requests

E. No correction required

27. The poor Brahmin led a hand to mouthful

existence and could use any job which paid

him a little.

www.gradeup.co

5

A. handful to mouthful existence

B. hand to mouth existence

C. handing for mouthful existing

D. hand and mouth exist

E. No correction required

28. In order to earning decent living we need

to have a good job which pays a substantial

amount of money.

A. earned decency life

B. earning decency live

C. earn a decent living

D. earned decently life

E. No correction required

29. We went to the famous restaurant to eat and

were served piped hot food.

A. served piping hotter

B. serving pipe hot

C. served piping hot

D. serve pipe hotten

E. No correction required

30. When he fell down the ditch, he shouted with

all his might so that to catch someone’s

attention.

A. such that to catch

B. so as to catch

C. so that to catching

D. so then to catch

E. No correction required

Direction (31-35): In the following question,

two equations I and II are given. Solve both

the equations carefully & answer the

questions given below:

31. I.

II.

A. x < y B. x > y

C. x ≤ y D. x ≥ y

E. x = y or no relation can be established.

32. I.

II.

A. x < y B. x > y

C. x ≤ y D. x ≥ y

E. x = y or no relation can be established.

33. I.

II.

A. x < y B. x > y

C. x ≤ y D. x ≥ y

E. x = y or no relation can be established.

34. I.

II.

A. x < y B. x > y

C. x ≤ y D. x ≥ y

E. x = y or no relation can be established.

35. I.

II.

A. x < y B. x > y

C. x ≤ y D. x ≥ y

E. x = y or no relation can be established.

36. Direction: What will come in place of the

question mark (?) in the following number

series?

14, 8, 9, 14.5, 30, ?

A. 72 B. 73

C. 74 D. 75

E. 76

37. Direction: What will come in place of the

question mark (?) in the following number

series?

77, 85, 69, 101, 37, ?

A. 105 B. 125

C. 145 D. 165

E. 185

38. In the following number series, one number is

missing. What should come at the place of

missing number (?)

20, 29, 54, 103, 184, ?

A. 301 B. 302

C. 303 D. 304

E. 305

39. Direction: What will come in place of the

question mark (?) in the following number

series?

7, 8, 18, 57, ?, 1165

A. 212 B. 217

C. 232 D. 247

E. 275

40. Direction: What will come in place of the

question mark (?) in the following number

series?

5, 7, 18, 47, 103, ?

A. 155 B. 175

C. 195 D. 215

E. 235

www.gradeup.co

6

41. Direction: What should come in place of

question mark (?) in the following question?

(You do not have to calculate the exact

value.)

A. 12 B. 42

C. 92 D. 132

E. 172

42. Direction: What approximate value will come

in place of the question mark (?) in the

following question?

A. 24 B. 39

C. 36 D. 28

E. 32

43. What approximate value will come at the

place of question mark (?) in the following

question?

A. 19 B. 18

C. 21 D. 25

E. 15

44. Directions: What approximate value will

come in place of the question mark (?) in the

following question? (You are not expected to

calculate the exact value)

7441 ÷34 × 12 = ? × 9 + 110

A. 420 B. 280

C. 590 D. 350

E. 220

45. Directions: What approximate value will

come in place of question mark (?) in the

following questions? (You are not expected to

calculate the exact value.)

5466.97-3245.01+1122.99=? + 2309.99

A. 1130 B. 1000

C. 1100 D. 1030

E. 1060

Direction (46-50): Study the following

graph carefully to answer the question given

below:

Given below is the demand and production of

6 brands (in units) of a product in the year

2016.

46. If the demand for brand C product increase by

75% then to meet the demand production

should be increased by what percent?

A. 32.65% B. 48.48%

C. 57.14% D. 31.25%

E. None of these

47. Brand A increase its production to meet its

demand. With every 160 unit produced the

brand increases its price by 10%. If the

earlier price of one product was INR 5000

then find the new price of the product.

A. INR 5500 B. INR 6655

C. INR 7320.5 D. INR 8052.55

E. Cannot be determined

48. The demand for brand D product fell. The new

demand is 20% less than its production. Find

by what percentage demand fell?

A. 28% B. 30%

C. 38.88% D. 72%

E. 32%

49. Brand B decreased its price of the product to

meet its demand to its production. When the

price decreased by 12% the demand

increased by 25%. If the ratio between the

new price and new demand is 11:20 then find

the price of the product before the decrease.

A. INR 2500 B. INR 2300

C. INR 2200 D. INR 2000

E. INR 2800

50. The production of brand E and F took together

is approx. what percent of total demand of E

and F?

A. 81% B. 21%

C. 123% D. 121%

E. 23%

www.gradeup.co

7

51. Ratio between height of 2 cylinder is in the

ratio 3:5. Their volumes are in the ratio

27:80. Find ratio between their radius.

A. B.

C. D.

E. None of these

52. B is 20% efficient than A. B started the work

& do it for x days. And then B is replaced by

A. And A completed the remaining work in

x+8 days. Ratio of work done by A & B is 3:2.

In how many day A & B working together to

complete the whole work?

A. 120/12 B. 150/11 days

C. 140/13 days D. 100/33 days

E. 75/12 days

53. The time taken for covering ‘X’ Km by

downstream is equal to 'X-18' by covering

upstream. Upstream speed is 6 km/ hr less

than that of downstream. If the speed of the

boat in still water is 15 km/hr. What is the

value of 'X'.

A. 51 B. 52

C. 53 D. 54

E. 55

54. A sum of Rs. 91,000 is borrowed at 20% per

annum compounded annually. If the amount

is to be paid in two years, the amount will be?

A. 1,20,000 Rs. B. 1,25,760 Rs.

C. 1,27,526 Rs. D. 1,31,040 Rs.

E. 1,34,034 Rs.

55. If an article is marked 40% above the cost

price. If discount of x% is given on the

marked price of the article then final profit of

12% is obtained. Now if CP of a new article is

120 Rs. and x% profit is desired then what

should be the selling price of that new article?

A. 140 B. 142

C. 144 D. 146

E. 148

56. A, B and C started a business and invested in

the ratio of 3:4:5. After 4 months, A withdrew

1/12th amount of what B and C invested. If

the annual income was 9200, then what was

the share of B?

A. 3280 B. 3480

C. 3200 D. 3880

E. 4080

57. In a 40 litres mixture acetic acid and sodium

acetate are in the ratio 3:1,find the amount of

sodium acetate solution to be added to make

the ratio 2:3.

A. 40 litres B. 20 litres

C. 15 litres D. 30 litres

E. 35 litres

58. Radhika has two daughters by name Rinku

and Sindhu. The ratio of the age of Radhika

and Rinku is 3:1 and that of Rinku and Sindhu

is 8:5. Given that Rinku is six years elder to

Sindhu. Find the ratio of their ages after 12

years.

A. 29:15:12 B. 30:14:10

C. 29:14:11 D. 30:14:11

E. 30:13:11

59. Three years ago the average age of Mohan’s

family having 5 members was 17 years.

Mohan becomes father but the average age of

his family is same today. What is the present

age of baby?

A. 1 year B. 2 years

C. 3years D. 4 years

E. 5 years

60. Out of 12 persons, 11 spend Rs.2000 monthly

each. The twelfth person spends Rs.110 more

than the average spending of the 12 people.

How much money does the twelfth person

spend?

A. Rs.2200 B. Rs.2120

C. Rs.3300 D. Rs.1800

E. Rs.2010

Direction (61-65): Read the following

information carefully and answer the

questions given below.

61. Find the average of the number of female in

all the colleges except college C and E?

A. 672.8 B. 683.5

C. 750 D. 753.5

E. 602.8

www.gradeup.co

8

62. Find the average difference between the

number of male and female students in all the

colleges?

A. 312.833 B. 314.60

C. 313 D. 314.50

E. None of these

63. The number of female students in college C is

what approx. percent of the number of male

students of college A?

A. 70% B. 72%

C. 74% D. 76%

E. 77%

64. Out of total female in college E, 30% are in

Arts department which is 35% of the total

students in Arts department. Find out

approximately how much percent of male

students from E are in Arts department?

A. 20% B. 22%

C. 19% D. 23%

E. 25%

65. Find the ratio of 2/3rd of college B male

students and female students of college F.

A. 29: 11 B. 29: 33

C. 33: 29 D. 29: 22

E. 22: 29

Direction (66-70): Study the following

information carefully and answer the question

based on it.

8 persons A, B, C, D, E, F, G and H have their

birthdays on 14th and 21st of four different

months, January, March, April and June not

necessarily in the same order. A has birthday

on even date and a month which having 31

days. Number of persons between C and F is

same as the number of persons between B

and H. H’s birthday is neither on even number

date nor in a month which having 31 days. B

is not born after H. C is born before F and B,

who is not born in January. D is not born in

June. F was born on even number date. D’s

birthday is on odd number date and a month

which having 31 days. B and H were not born

in the same month. E was born after F. D is

born before G.

66. How many persons born after F?

A. 1 B. 2

C. 3 D. 4

E. 5

67. Who among the following is not born in the

month having 30 days?

A. F B. D

C. B D. H

E. E

68. Who among the following does not belongs to

the group?

A. A B. D

C. G D. B

E. H

69. How many persons have birthday between D

and E?

A. 1 B. 2

C. 3 D. 4

E. 5

70. Who among the following in not born before

F?

A. A B. D

C. C D. H

E. G

Directions (71-73): In these questions,

relationship between different elements is

shown in the statements. The statements are

followed by two conclusions. Find the

conclusion which is definitely true.

71. Statements:

A > B = C < D < E > F

Conclusions:

I. F < C

II. A>D

A. If only Conclusion I is true.

B. If only Conclusion II is true.

C. If either Conclusion I or II is true.

D. If neither Conclusion I nor II is true.

E. If both Conclusions I and II are true.

72. Statements:

A < B > C > D; A > E, D > F

Conclusions:

I. F > B

II. B > E

A. If only Conclusion I is true.

B. If only Conclusion II is true.

C. If either Conclusion I or II is true.

D. If neither Conclusion I nor II is true.

E. If both Conclusions I and II are true.

73. Statements:

A = B < C > D; E > C < F

Conclusions:

I. E > A

II. F > D

www.gradeup.co

9

A. If only Conclusion I is true.

B. If only Conclusion II is true.

C. If either Conclusion I or II is true.

D. If neither Conclusion I nor II is true.

E. If both Conclusions I and II are true.

74. Which of the following symbols should replace

the sign ($) and (#) in the given expression in

order to make the expression S > J and M ≥ J

is definitely true?

S ≥ T $ P > K ≤ N, O < M = K # J > L

A. =, < B. >, ≤

C. <, ≥ D. <, ≤

E. >, ≥

75. Which of the following symbols should replace

the $ and # in the given expression in order

to make the expression L ≥ S and N > T is

definitely true?

N > O ≤ L = P $ T, H > M ≤ T # S < N

A. >, < B. ≥, ≤

C. >, = D. <, ≤

E. >, ≥

Direction (76-80): Study the following

information carefully and answer the

questions given below:

Eight persons P, Q, R, S, T, U, V and W are

sitting in a straight line and facing north

direction. Their ages are, 12, 14, 18, 26, 29,

35 ,42 and 67. The one who is 12 years old is

4th to the left of the eldest person, who is

sitting at the end. The sum of the ages of S

and Q is equal to P. S is not the youngest

person. S and W are neighbours of Q. R’s age

is not an even number and he is older than W

and younger than U. Only 3 persons are

sitting between S and U. Only 2 persons are

sitting between Q and T, who is 29 years old.

P is not the neighbour of U. R is not sitting to

the left of Q.

76. Who among the following sits at the end?

A. T B. P

C. Q D. S

E. W

77. Who among the following sits third to the left

of W?

A. S

B. The one who is 12 years old

C. The one who is 26 years old

D. T

E. U

78. Which of the following pair is neighbour of U?

A. R and W B. S and P

C. W and V D. R and V

E. Q and R

79. Who among the following is 14 years old?

A. P B. Q

C. R D. U

E. S

80. How many persons are younger than W?

A. 1 B. 2

C. 3 D. 4

E. None

81. Direction: If 1 number is added in even digit

in the given number 9458732 and 2 is

subtracted in odd digit, then how many digit

is repeated in the newly formed number?

A. 1 B. 2

C. 3 D. 4

E. 5

82. In a certain code language ‘green grass

everywhere’ is written as ‘dik pa sok’ and ‘cow

eats grass’ is written as ‘nok ta pa’. How is

‘cow’ written in that code language?

A. nok B. ta

C. nok or ta D. Data inadequate

E. None of these

Direction (83-85): Study the following

information carefully and answer the

questions given below:

Each of the six students got different marks in

an exam. D got less marks than only B. E got

more marks than only two students. A didn’t

get less marks than E. C did not get least

marks. The one who got second lowest, got

160 marks. A got 205 marks.

83. The one who got maximum marks, got 100

marks more than C. Which of the following

mark is got by one who got maximum marks?

A. 225 B. 270

C. 260 D. 300

E. None of these

84. Which of the following is true with respect to F

as per the given information?

A. The possible marks, got by F is 165

B. F got minimum marks.

C. Only two students got more marks than F

D. F definitely got more marks than C

E. None of these

www.gradeup.co

10

85. Which of the following may be the possible

number of marks which E got?

A. 210 B. 140

C. 185 D. 159

E. None of these

Direction (86-90): Study the following

information carefully and answer the question

given below:

There are eight persons A, B, C, D, E, F, G

and H. They are born in the month (Same for

each person) of different years i.e. 1976,

1980, 1982, 1990, 1991, 1995, 2000 and

2005. There ages are considered as on the

same month of 2017. B is born in odd number

year but he is not the youngest. E is 37 years

old now. C was born in 1990. G is at least 9

years older than B. D is younger than B and

born in odd number year. A is 18 years

younger than G. F is older than D but not the

older person. Not more than 2 persons are

younger than F.

86. Who among the following is youngest person?

A. A B. C

C. D D. B

E. G

87. How many persons are older than C?

A. 2 B. 3

C. 1 D. None

E. more than 3

88. What is the difference of age between F and

E?

A. 15 years B. 10 years

C. 30 years D. 4 years

E. 12 years

89. If all the persons are arranged according to

their names according to alphabet series from

youngest to oldest then how many of them

position will remain unchanged?

A. 0 B. 1

C. 2 D. 3

E. 4

90. Who among the following is born in 1982?

A. D B. G

C. A D. C

E. H

Direction (91-96): Study the following

information carefully and answer the

questions given below:

Seven persons A, B, C, D, E, F and G attends

seminar on different dates starting from

21st June to 27th June but not necessarily in

the same order. They like different brands

laptop, Lenovo, Dell, Apple, Sony, Samsung,

Asus, and HP. They have different brands of

watch, Sonata, Rolex, Titan, Maxima, Casio,

Fossil and Diesel.

The one who has Diesel attends a seminar on

an even numbered date. E has Rolex and F

doesn’t have Maxima. G attends the seminar

on the last day. The one who has Diesel likes

either Apple or Asus laptop. A has Titan but

he doesn’t like HP and is not attend the

seminar just before or just after B. F likes Dell

and attends the seminar three days before

the one who likes Samsung. D neither likes

Samsung nor Asus laptop. The one who likes

HP attends the seminar on 25th June. B likes

Sony wearing also have Sonata watch and not

attends the seminar just before or just after

F, who attends the seminar on an odd

numbered date. The one who likes Apple

neither has Casio nor Maxima. B attends the

seminar just before D and none of them likes

HP. The one who has Casio attends the

seminar on the 2nd day.

91. Who among the following attends the seminar

on 3rd day?

A. D

B. The one who likes Dell

C. C

D. The one who has Diesel

E. The one who likes Samsung

92. Which of the following is correct combination

for D?

A. Lenovo-Casio

B. Dell-Sonata

C. Samsung-Rolex

D. Sony-Fossil

E. HP-Rolex

93. How many persons attend the seminar

between the one who likes Samsung and the

one who has Fossil?

A. 0 B. 1

C. 2 D. 3

E. 4

94. If all the persons rearranged according to

their names (alphabetically) from the starting

date then who among the following likes

Apple?

www.gradeup.co

11

A. A B. B

C. C D. D

E. E

95. Which of the following combination is correct?

A. D-Lenovo-Fossil B. E-HP-Titan

C. A-Sony-Maxima D. F-Dell-Diesel

E. A-Samsung-Titan

96. ‘B’ is related to ‘Dell’ and ‘F’ is related to

‘Rolex’ in the same way then who among the

following is related to ‘Maxima’?

A. A B. B

C. C D. D

E. E

Directions (97-98): Study the following

information carefully and answer the

questions given below:

* D is the mother of E, who is the sister of F.

* A is the father of C. B is wife of A.

* F is son of C.

97. How is E related to A?

A. Son B. Grandson

C. Granddaughter D. Daughter

E. None of these

98. How is B related to D?

A. Mother-in-law B. Sister

C. Mother D. Can’t be determined

E. None of these

Directions (99-100): Study the following

information carefully and answer the

questions given below:

Rahul starts running from point A and runs

15 km towards south. He takes a left turn and

runs 20 km. Now he runs 9 km after taking a

right turn. He finally takes a right turn and

runs 20 km and stops at point B.

99. How far is point B with respect to point A?

A. 24 km B. 40 km

C. 45 km D. 29 km

E. 49 km

100. Towards which directions was Rahul moving

before he stopped at point B?

A. North B. East

C. West D. South

E. North-West

***

The information provided here is only for refrence. It may vary from the original.

www.recruitment.guru

STATE BANK OF INDIA PAPER ON 7th JANUARY 2007

Question Paper

Section 1: Reasoning

Directions (1-5): In the questions given below, certain symbols are used with the

following meaning:

A @ B means A is greater than B.

A * B means A is either greater than or equal to B.

A # B means A is equal to B

A $ B means A is either smaller than or equal to B

A + B means A is smaller than B

Now in each of the following questions assuming the given statements to be true,

find which of the two conclusions I and II given below them is/are

definitely True?

Give answer (a) if only conclusion I is true.

Give answer (b) if only conclusion II is true.

Give answer (c) if either conclusion I and II are true.

Give answer (d) if neither conclusion I nor II is true.

Give answer (e) if both conclusions I and II are true.

1. Statements:

D + T ; E $ V ; F * T ; E @ D

Conclusions: I. D $ V II. D + F

Ans: (b)

2. Statements:

B + D ; E $ T ; T * P ; P @ B

Conclusions:

I. P $ D II. P @ D

Ans: (c)

3.Statements:

T * U ; U $ W ; V @ L ; W + V

Conclusions: 1. V @ T II. L # W

Ans: (d)

4.Statements:

P $ Q ; N # M ; M @ R ; R * P

Conclusions:

I. P + N II. Q $ M

Ans: (a)

The information provided here is only for refrence. It may vary from the original.

www.recruitment.guru

5.Statements: E * F ; G $ H ; H # E ; G @ K

Conclusions:

I. H @ K II H * F

Ans: (e)

Directions(6-11): Study the following letter-number-symbol sequence carefully

and answer the questions given below:

3 D 5 F E 3 8 $ M 2 1 K * P T @ U 9 A 7 1 £ H J 4 Q 6

6. What should come in place of the question mark (?) in the following Sequence?

5ES, MIP, ?, 1HQ

(a) TUA (b) TU7 (c) @ 91 (d) T91

(e) None of these

Ans: (b)

7.Which of the following is exactly in the midway between the eleventh from the

left end and the 7th from the right end?

(a) P (b) @ (c) T (d) U (e)

None of these

Ans: (b)

8. Which of the following is the sixth to the right of the twentieth from the right

end?

(a) 5 (b) F (c) P (d) K (e) None

of these

Ans: (c)

9. How many such digits are there in the above sequence which are immediately

preceded as well as followed by digits ?

(a) None (b) One (c) Two (d) Three

(e) more than three

Ans: (a)

10. If the first fifteen elements are written in the reverse order then which of the

following will be eighth to the left of the thirteenth element from right end ?

(a) M (b) 8 (c) $ (d) * (e)

None of these.

Ans: (a)

11. If all the consonants starting from B are given sequentially the value of even

numbers such as B=2, C=4 and so on and all the vowels are given the value of 5

each, then what will be the value of the letters of the word CUSTOM ?

(a) 92 (b) 86 (c) 82 (d) 96 (e)

None of these

Ans: (d)

12. How many such 5s are there in the following sequence that the sum of the two

immediately following digits is greater than the sum of the two immediately

The information provided here is only for refrence. It may vary from the original.

www.recruitment.guru

preceding digits ?

3 7 6 5 8 3 2 4 5 5 4 8 7 9 1 5 3 4 8 7 5 9 8 7 6 4

(a) One (b) Two (c) Three (d) Four (e) None of

these

Ans: (c)

13. If A + B means "A is the sister of B", A × B means "A is the wife of B", A ÷ B

means "A is the father of B" and A-B means "A is the brother of B"' then which

of following expresses the relationship that "t is the daughter of p"?

(a) P × Q ÷ R + S - T (b) P × Q ÷ R - T + S (c) P × Q ÷ R + T - S (d)

P × Q ÷ R + S + T (e) None of these

Ans: (b)

14.If the position of the first letter of English alphabet is interchanged with the

position of the fourteenth letter, second letter with the fifteenth letter in such a

way that M is interchanged with Z, then which of the following letters will be 9th

to the right of 17th letter from the right?

(a) F (b) E

(c) R (d) T

(e) None of these

Ans: (a)

Directions(15-18) : Read the following information and answer the questions

given below:-

(i) Seven friends P, Q, R, S, T, U and W have gathered at the Mumbai airport.

Five of them are scheduled to go to five different places-Delhi, Chennai,

Lucknow, Bangalore and Calcutta.

(ii) Five of them are executives, each specialising in viz. Administration (admn.),

Human Resource Management (HRM), Marketing ,Systems and Finance.

(iii) T, an executive is going to Chennai and is neither from finance nor

Marketing.

(iv) W is a system specialist and is leaving for Delhi. U is an executive but is not

going to one of the five places.

(v) Q is an executive from HRM but has come at the airport to see his friends.

(vi) P is an executive but not from Marketing and is flying to one of the

destinations but not to Bangalore or Calcutta.

15. Who is going to fly to Bangalore?

(a) Data inadequate (b) R (c) S (d) P (e) None of

these

Ans: (a)

16. Who among the following specialises in Marketing?

(a) S (b) P (c) U (d) Data inadequate (e) None of

these

Ans: (c)

17. R has specialisation in which of the following fields?

(a) Finance (b) Marketing (c) Either Marketing or Finance (d)

The information provided here is only for refrence. It may vary from the original.

www.recruitment.guru

None (e) All of these

Ans: (d)

18.The one who is going to fly to Chennai is

(a) Not an executive (b) From administration (c) S (d)

From Finance (e) None of these

Ans:(b)

19. How many pairs of letters are there in the word 'NURSING' which have as

many letters between them as in the alphabet?

(a) One (b) Three (c) Five (d) Six (e) None of these

Ans: (b)

Directions(20-26): Read the following information carefully and answer the

questions given below.

A famous museum issues entry passes to all its visitors for security reasons.

Visitors are allowed in batches after every one hour. In a day there are six batches.

A code is printed on entry pass which keeps on changing for every batch.

Following is an illustration of pass-codes issued for each batch.

Batch I:

clothes neat and clean liked are all by

Batch II:

by clothes neat all are and clean liked

Batch III: liked by clothes clean and neat all are and so on....

20. If pass-Code for the third batch is 'night succeed day and hard work to for ',

what will be the pass code for the sixth batch?

(a) Work hard to for succeed night and day (b) Hard work for and succeed night

to day

(c) Work hard for to succeed night and day (d) Hard work for to succeed night

and day

(e) None of these

Ans: (c)

21. If 'visit in zoo should the we time day' is the pass code for the fifth batch, 'zoo

we the should visit day time in ' will be the pass code for which of the following

batches?

(a) II (b) IV

(c) I (d) III

(e) VI

Ans: (d)

22. Sanjay visited the museum in the fourth batch and was issued a pass-code 'to

fast rush avoid not do very run'. What would have been the pass-code for him had

he visited the museum in the second batch?

(a) rush do not avoid to run very fast (b) rush not do avoid to run very fast

(c) avoid rush not do to run very fast (d) Data inadequate

The information provided here is only for refrence. It may vary from the original.

www.recruitment.guru

(e) None of these

Ans: (a)

23. Subodh went to visit the museum in the second batch. He was issued a pass-

code 'length the day equal of and night are'. However, he could not visit the

museum in the second batch as he was little late. He then preferred to visit in the

fifth batch. What will be the new pass -code issued to him?

(a) and of are night the length equal day (b) and are of night the length equal

day

(c) and of are night the equal (d) and of are the night length day

equal

(e) None of these

Ans: (a)

24. If pass-code for the second batch is 'to confidence hard you leads work and

success', what will be the pass -code for the fourth batch?

(a) leads success to you hard confidence and work (b) leads success you to hard

confidence and work

(c) leads success to you hard confidence work and (d) leads to success you hard

confidence and work

(e) None of these

Ans: (a)

25. If the pass-code issued for the last (sixth) batch is 'and pencil by all boys used

are pen'. What will be the pass-code for the first batch?

(a) pencil and pen are used by all boys (b) pen and pencil used are by all

boys

(c) pen and pencil are used by all boys (d) pencil and pen are used all by

boys

(e) None of these

Ans: (c)

26. If the pass-code for the sixth batch is 'not go the way to of out do'. What will be

the pass-code for the third batch?

(a) of do to out go not way the (b) of to do out not go way the

(c) of to go out do not way the (d) Data inadequate

(e) None of these

Ans: (e)

Directions(27-32): in each question below are given three statements followed by

four conclusions-I, II, III and IV. You have to take the given statements to be true

even if they seem to be at variance with commonly known facts. Read the all

conclusions and then decide which of the given conclusions logically follow (s)

from the given statements disregarding commonly known facts.

27.Statements

Some books are pens

All pens are chairs.

The information provided here is only for refrence. It may vary from the original.

www.recruitment.guru

Some chairs are Tables.

Conclusions I. Some books are chairs.

II . Some chairs are books.

III. All tables are chairs.

IV. Some tables are chairs.

(a) All follow ( b) Only I, II, III follow

(c) Only I, II, IV follow (d) Only II, III and IV follow

(e) none of these

Ans:(c)

28. Statements

All cars are jeeps.

All jeeps are buses.

All buses are trucks.

Conclusions I. All trucks are buses.

II. All buses are jeeps.

III. All jeeps are cars.

IV. All cars are trucks.

(a). None follows (b) All follow

(c) Only III and IV follow (d) Only IV follows

(e) None of these

Ans:(d)

29.Statements

Some trees are flowers.

Some flowers are pencils.

Some pencils are tables.

Conclusions I. Some tables are flowers.

II. Some pencils are trees.

III. Some tables are trees.

IV. Some trees are pencils.

(a) All follow (b) None follows

(c) Only I and III follow (d) Only II and IV follow

(e) None of these

Ans: (b)

30.Statements

All rods are bricks.

Some bricks are ropes.

All ropes are doors.

Conclusions

I. Some rods are doors.

II. Some doors are bricks.

III. Some rods are not doors.

IV. All doors are ropes.

(a) Only I and II follow

The information provided here is only for refrence. It may vary from the original.

www.recruitment.guru

(b) Only I, II and III follow

(c) Either I or III and II follow

(d) Either I or III and IV follow.

(e) None of these

Ans.(c)

31. Statements

Some books are pens.

Some pens are watches.

Some watches are books .

Some books are watches.

Conclusions I. Some radios are watches.

II. Some radios are pens.

III. Some watches are books

IV. Some books are watches.

(a) All follow (b) Only I and III follow

(c) Only II and IV follow (d) Only I and IV follow

(e) None of these

Ans:(e)

32. Statements

All towns are villages

No village is forest

Some forests are rivers.

Conclusions I. Some forests are villages.

II. Some forests are not villages.

III.Some rivers are not villages.

IV.All villages are towns.

(a) All follow

(b) Only either I or II follows

(c) Only either I or II and III follow

(d) None of these

(e) None of these

Ans: (e)

33. In a row of boys facing north, Sudhanshu is twelfth from his left. When shifted

to his right by four places, he becomes eighteenth from the right end of the row?

(a) 32 (b) 33

(c) 34 (d) Data inadequate

(e) None of these

Ans: (b)

34.In a certain code language PROBLEM is written as MPERLOB. How will

NUMBERS be Written in that code?

(a) SNUREMB (b) SNRUBME

(c) SNRUEMB (d) SNRUMEB

The information provided here is only for refrence. It may vary from the original.

www.recruitment.guru

(e) None of these

Ans:(c)

Directions(35-40):In each question below is given a statement followed by two

assumptions numbered I and II. An assumption is something supposed or taken for

granted. You have to consider the statement and the following assumptions and

decide which of the assumptions is implicit in the statement.

Give Answer (a) if only assumption I is implicit.

Give Answer (b) if only assumption II, is implicit.

Give Answer (c) if either I or II is implicit.

Give Answer (d) if neither I nor II is implicit.

Give Answer (e) if both I and II are implicit.

35.Statement: The Government has recently hiked the prices of diesel and petrol

to reduce the oil pool deficit.

Assumptions:

I. The amount earned by this increase may be substantial enough to reduce the

deficit.

II. There may be wide spread protests against the price hike.

Ans:(e)

36. Statement: The X passenger car manufacturing company announced a sharp

reduction in the prices of their luxury cars.

Assumptions: I. There may be an increase in the sale of their luxury cars.

II. The other, such car manufacturers may also reduce their prices.

Ans:(e)

37. Statement : A foreign film producer rendered his apology before Indian

society for misinterpreting a part of Indian epic.

Assumptions : I. Indians are very sensitive to the misinterpretation of their epic.

II. It is possible to derive wrong meaning from the epic.

Ans:(d)

38.Statement : Aswin's mother instructed him to return home by train if it rains

heavily.

Assumptions I. Aswin may not be to decide himself if it rains heavily.

II. The trains may ply even if it rains heavily.

Ans:(b)

39. Statements : The Government of India decided to start a track II dialogue with

its neighbour to reduce tension in the area.

Assumption:

I. The neighbouring country may agree to participate in the track II dialogue.

II. The people involved in track II dialogue may be able to persuade their

respective Governments.

Ans:(e)

The information provided here is only for refrence. It may vary from the original.

www.recruitment.guru

40. Statements :The host in one of the popular T.V programmes announced that

the channel will contact the viewers between 9.00 a.m. to 6.00 p.m on weekdays

and the lucky ones will be given fabulous prizes.

Assumptions: I. The people may remain indoors to receive the phone call.

II. More people may start watching the programme.

Ans:(e)

Directions(41-45): In making decisions about important questions, it is desirable

to be able to distinguish between "Strong" arguments and "Weak" arguments are

those which are both important and directly related to the

question. "Weak" arguments are those which are of minor importance and also

may not be directly related to the question or may be related to a trivial aspect of

the questions.

Instructions: Each question below is followed by a statement and two arguments

numbered I & II. You have to decide which of the argument is

a "Strong" argument and which is a "Weak" argument.

Give answer (a) if only argument I is strong.

Give answer (b) if only argument II is strong.

Give answer (c) if their I or II is strong.

Give answer (d) if neither I nor II is strong.

Give answer (e) if both I and II are strong.

41.Statement: should the habit of late coming in educational institutions be

checked?

Arguments: I. No. Until it affects the work.

II. Yes. Discipline must be maintained.

Ans:(b)

42.Statement: Should seniority be the only criterion for the promotion?

Arguments: I. No. All the senior employees are not interested in promotion.

II. Yes. Otherwise senior employees do feel humiliated.

Ans:(d)

43. Statement: Should children be prevented completely from watching

television?

Arguments: I. No. We get vital information regarding education through television.

II. Yes. It hampers the study of children.

Ans:(e)

44. Statement: Should trade unions be banned completely?

Arguments: I. No. This is the only way through which employees can put their demands before

management.

II. Yes. Employees get their illegal demands fulfilled through these unions.

Ans:(a)

The information provided here is only for refrence. It may vary from the original.

www.recruitment.guru

45. Statement: should women be given equal opportunity in matter of

employment in every field?

Argument:

I. Yes. They are equally capable.

II. They have to shoulder household responsibilities too.

Ans:(a)

Directions(46-50): In each question below is given a statement followed by two

conclusions numbered I and II.You have to assume everything in the statement to

be true, then consider the two conclusions together and decide which of them

logically follows beyond a reasonable doubt from the information given in the

statement.

Give answer (a) if only conclusion I follows.

Give answer (b) if only conclusion II follows.

Give answer (c) if either I or II follows.

Give answer (d) if neither I nor II follows, and

Give answer (e) if both I and II follow.

46. Statement: The cabinet of State 'X' took certain steps to tackle the milk glut in

the state as the cooperatives and Government diaries failed to use the available

milk-A news report

Conclusion: I. The milk production of State 'X' is more than its need.

II. The Government and co-operative diaries in State 'X' are not equipped in terms

of resources and technology to handle such excess milk.

Ans:(e)

47. Statement: It has been decided or the Government to withdraw 33% of the

subsidy on cooking gas from the beginning of next month-A spokesman of the

Government.

Conclusions: I. People now no more desire of need such subsidy from Government as they can

afford increased price of the cooking gas.

II. The price of the cooking gas will increase at least by 33% from the next month.

Ans:(d)

48. Statement: "The Government will review the present policy of the diesel price

in view of further spurt in the international oil prices"-A spokesman of the

Government.

Conclusions: I. The Government will increase the price of the diesel after the imminent spurt in

the international oil [prices.

II. The Government will not increase the price of the diesel even after the

imminent spurt in the international oil prices.

Ans:(c)

49.Statement: My first and foremost task is to beautify this city-if city 'X' and Y

can do it- why can't we do it-statement of Municipal Commissioner of City 'Z' after

taking over charge.

The information provided here is only for refrence. It may vary from the original.

www.recruitment.guru

Conclusions: I. The people of city 'Z' are not aware about the present state of their city.

II. The present commissioner has worked in city 'X' and Y and has good

experience of beautifying cities.

Ans:(d)

50.Statement: Women's Organisations in India have welcomed the amendment of

the Industrial Employment Rules 1946 to curb sexual harassment at the work

place.

Conclusions:

I. Sexual harassment of women at work place is more prevalent in India as

compared to other developed countries.

II. Many organisations in India will stop recruiting women to avoid such problems.

Ans:(d)

Directions (51-55): Each of the questions below consists of a question and two

statements numbered I and II given below it. You have to decide whether the data

provided in the statements numbered I and II given below it. You have to decide

whether the data provided in the statements are

sufficient to answer the questions. Read both the statements and -

Give Answer (a) if the data in statement I alone are sufficient to answer the

question, while the data in statement II alone are not sufficient to answer the

question.

Give answer (b) if the data in statement II alone are sufficient to answer the

question, while, the data in statement I alone are not sufficient to answer the

question.

Give answer (c) if the data either in the statement I alone or in statement II alone

are sufficient to answer the question.

Give answer (d) if the data even in both statement I and II together are not

sufficient to answer the question.

Give answer (e) if the data in both statements I and II together are necessary to

answer the question.

51. In which direction is Ravi facing?

I. Asok is to the right of Ravi

II. Samir is sitting opposite of Asok facing north.

Ans:(d)

52. How M is related to

I. The sister of K is the mother of N who is daughter of M .

II. P is the sister of M.

Ans: (e)

53. Is D brother of T?

I. T is the sister of M and K.

II. K is the brother of D.

Ans: (d)

The information provided here is only for refrence. It may vary from the original.

www.recruitment.guru

54. How many sons does P have?

I. S and T are brothers of M.

II. The mother of T is P who has Only One daughter.

Ans: (e)

55. Who is the tallest among P, Q, R, S and T?

I. R is taller than Q and T.

II. T is taller than S and P and S is taller than Q and R.

Ans: (b)

Directions (56-60) : Below is given a passage followed by several possible

inferences which can be drawn from the fact stated in the passage. You have to

examine each inference separately in the context of the passage and decide up on

its degree of truth or falsity.

Mark answer (a) if the inference is "definitely true" i.e., it properly follows from

the statement of facts given.

Mark answer (b) if the inference is "probably true" though not "definitely true" in

the light of the facts given.

Mark answer (c) if the data are inadequate i.e, from the facts given you cannot say

whether the inference is likely to be true or false.

Mark answer (d) if the inference is "probably false" in the light of the facts given.

Mark answer (e) if the inference is "definitely false" i.e, it cannot possibly be

drawn from the facts given or it contradicts the given facts.

With the purpose of upliftment of Gonda district in Uttar Pradesh, a new

formula way evolved for practical success in several fields, such as, irrigation,

animal husbandry, dairy farming, moral uplift and creation of financial resources.

Small farms were clustered for irrigation by one diesel pump which could irrigate

about 20 acres of land. Youth were prompted to take loans from the banks for

purchase of engine pumps to be supplied to the farmers on rent. This formula

worked so well that the villages in Gonda district were saturated with irrigation

facilities. Cattle rearing was linked with multiple cropping .

Most of the targets fixed for different areas were achieved, which was an

unusual phenomenon. This could be possible only because of right motivation,

participation and initiative of the people. Imagination and creativity combined

together helped in finding out workable solutions to the problems of the

community.

56. There was no problem and complaint of people residing in entire Gonda

district, before the beginning of the project.

Ans: (e)

57. Purchasing of engine pumps by individual farmers may be beyond their

affordable limits.

Ans: (a)

58. Earlier farming was not basically one of the professions of people in Gonda

district.

Ans: (e)

The information provided here is only for refrence. It may vary from the original.

www.recruitment.guru

59.By using the same formula upliftment of any other district is possible.

Ans: (a)

60.There are very few people who can motivate others in the right direction.

Ans: (c)

Directions (61-65)

In each of the following questions, there are two sets of figures. The figures on the

top are problem figures (four figures and one question marked space) and those on

the bottom are Answer Figures indicated by numbers a, b, c, d and e. A series is

established if one of the five answer figures is placed in the ‘question marked

space’. Choose the right answer figure

61)

Ans (d)

62)

Ans (a)

The information provided here is only for refrence. It may vary from the original.

www.recruitment.guru

63)

Ans (b)

64)

Ans (c)

65)

The information provided here is only for refrence. It may vary from the original.

www.recruitment.guru

Ans (a)

Directions (66-70) In each of the following questions, a series begins with

unnumbered figure on the extreme left. One of the five numbered figures in the

series does not belong to the series. The two unnumbered figures one each on

extreme left and extreme right fit into the series. You have to take as many aspects

into account as possible of the figures in the series and find out the one and only

one of the five numbered figures which does not fit into the series. The number of

that figure is the answer

66)

Ans (d)

67)

Ans (e)

The information provided here is only for refrence. It may vary from the original.

www.recruitment.guru

68)

Ans (e)

69)

Ans (a)

70)

Ans (d)

71) Directions (71-75) In each of the following questions, which one of the five

answer figures on the right should come after the problem figures on the left, if the

sequences were continued?

The information provided here is only for refrence. It may vary from the original.

www.recruitment.guru

Ans (b)

72)

Ans (c)

73)

The information provided here is only for refrence. It may vary from the original.

www.recruitment.guru

Ans (a)

74)

Ans (c)

75)

The information provided here is only for refrence. It may vary from the original.

www.recruitment.guru

Ans (e)

Section 2: Quantitative Aptitude

76. The difference between a two digit number and the number obtained by

interchanging the positions of its digits is 36. What is the difference between the

two digits of that number?

(a) 4 (b) 9

(c) 3 (d) Cannot be determined

(e) None of these

Ans: (a)

77. By how much is two-fifth of 200 greater than three-fifth of 125?

(a) 15 (b) 3

(c) 5 (d) 30

(e) None of these

Ans: (c)

78. The area of a rectangular field is 460 square metres. If the length is 15 percent

more than the breadth, what is breadth of the rectangular field?

(a) 15 metres (b) 26 metres

(c) 34.5 metres (d) Cannot be determined

(e) None of these

Ans: (e)

79. Deepa brought a calculator with 30% discount on the listed price. Had she not

got the discount, she would have paid Rs.82.50 extra. At what price did she buy the

calculator?

(a) Rs. 192.50 (b) Rs. 275

(c) Rs.117.85 (d) Cannot be determined.

(e) None of these.

Ans: (a)

The information provided here is only for refrence. It may vary from the original.

www.recruitment.guru

80. How many different ways can be letters of the word SOFTWARE be arranged

in such a way that the vowels always come together?

(a) 13440 (b)1440

(c) 360 (d) 120

(e) None of these

Ans: (e)

81. A motor starts with the speed of 70 kmph with its speed increase in every two

hours by 10kmph. In how many hours will it cover 345 Kms?

(a) 2 1/4 Hours (b) 4 1/2 Hours

(c) 4 Hours 5 Minutes (d) Cannot be determined

(e) None of these.

Ans: (b)

82. What will be the cost of gardening 1 metre broad boundary around a

rectangular plot having perimeter of 340 metres at the rate of Rs. 10per Square

Metre?

(a) Rs. 3,400 (b) Rs. 1,700

(c) Rs. 3,440 (d) Cannot be determined

(e) None of these.

Ans: (c)

83. In how many different ways a group of 4 men and 4 women be formed out of

7 men and 8 women ?

(a) 2450 (b) 105

(c) 1170 (d) Cannot be determined

(e) None of these.

Ans: (a)

84. A certain amount earns simple interest of Rs. 1,750/- after 7 years. Had the

interest been 2% more, how much more interest would it have earned?

(a) Rs. 35 (b) Rs. 350

(c) Rs. 245 (d) Cannot be determined

(e) None of these.

Ans: (d)

85.Seats for Maths, Physics and Biology are in the ratio of 5:7:8 respectively.

There is a proposal to increase these seats by 40%, 50% and 75% respectively.

What will be the respective ratio of increased seats?

(a) 2:3:4 (b) 6:7:8

(c) 6:8:9 (d) Cannot be determined

(e) None of these.

Ans: (a)

Directions 86-90 Study the following information to answer these questions

86. Financial information about a company for 2 years

The information provided here is only for refrence. It may vary from the original.

www.recruitment.guru

Which of the following can be inferred from above?

a) The profit in both the years was same because the revenue and expenditure

were also the same

b) Inspite of revenue being less in 2007, the profit was same as 2006 because

the expenditure also was correspondingly less

c) The profit and revenue in 2007 were slightly less than that of 2006

d) The profit is entirely related to revenue

e) The expenditure is directly related to revenue

Ans (b)

87.

Food type % of fat per 100 gms

A 28

B 20

C 13

Which of the following diets would supply the most fat?

a. 200 gms of A and 300 gms of C

b. 200 gms of B and 200 gms of A

c. 100 gms of G and 300 gms of A

d. 300 gms of C and 100 gms each of A and B

e. 50gms each of A, B and C

Ans (c)

2006 2007

Revenue

Expenditure

Profit

The information provided here is only for refrence. It may vary from the original.

www.recruitment.guru

88. A=x% of y, B=y% of x. Which of the following is true based on above?

a) A is greater than B

b) A is smaller than B

c) Relationship between A and B cannot be determined

d) If x is smaller than y then A is greater than B

e) None of these

Ans (e)

89. Percentage of male, female, literate and illiterate

What is the difference between number of Literate male and Literate female

a) 75,000

b) 1,500

c) 5,000

d) 500

e) None of these

Ans (c)

90. x= a__

a – 1

y= 1__

a – 1

a) x is greater than y

b) y is greater than x only if a<1

c) x is equal to y

8%

35%

24%

33%

N=2,50,000

Illiterate female

Literate male

Illiterate male

Literate female

The information provided here is only for refrence. It may vary from the original.

www.recruitment.guru

d) x is equal to y only if a<1

e) x is greater than y only if a<1

Ans (a)

Directions (91-95)In each of the following questions, two equations are given. You

have to solve them and

a) if p < q

b) if p > q

c) if p ≤ q

d) if p ≥ q

e) if p = q

91. I. p2-7p = -12

II. q2 - 3q + 2= 0

Ans (b)

92. I. 12p2-7p=1

II. 6q2 - 7q + 2= 0

Ans (a)

93 I. p2 + 12p + 35= 0

II. 2q2 + 22q + 56= 0

Ans (c)

94 I. p2-8p + 15 = 0

II. q2 - 5q = -6

Ans (d)

95 I. 2p2 + 20p + 50 = 0

II. q2 = 25

Ans (c)

Directions (96-100) Study the following graph to answer the given questions

Production of 2 companies A & B over the years (Production in Lakh Units)

The information provided here is only for refrence. It may vary from the original.

www.recruitment.guru

96. For Company A, what is the percent decrease in production from 2000 to

2001?

a) 75

b) 50

c) 25

d) 10

e) None of these

Ans (c)

97. In 2007, the production of Company B is approximately what percent of that of

in 2006?

a) 60

b) 157

c) 192

d) 50

e) 92

Ans (b)

0

1

2

3

4

5

6

7

8

9

10

11

12

13

2000 2001 2002 2003 2004 2005 2006 2007

A

B

The information provided here is only for refrence. It may vary from the original.

www.recruitment.guru

98. For Company A, in which year is the percentage increase/ decrease in the

production from the previous years is the highest?

a) 2007

b) 2008

c) 2005

d) 2002

e) None of these

Ans (d)

99. What is the difference in the total production of the two companies for the

given years?

a) 27,00,000

b) 31,00,000

c) 2,70,000

d) 3,10,000

e) None of these

Ans (a)

100. Which of the following is the closest average production in lakh units of

Company B for the given years?

a) 4.2

b) 3.5

c) 4.3

d) 3.75

e) 3.9

Ans (e)

The information provided here is only for refrence. It may vary from the original.

www.recruitment.guru

Directions 101-105 Study the following information to answer the given

questions.

Percentage of students in various courses (A, B, C, D, E, F) and percentage of

girls out of those

Total students: 1,200 (800 girls + 400 boys)

101. For course D, what is the respective ratio of boy and girls?

a) 3:4

b) 4:5

c) 3:5

A 20%

B 15%

C 5% D

35%

E 12%

F 13%

Percentage of various courses

A 30%

B 10%

C 2%

D 30%

E 14%

F 14%

Percentage of Girls in courses

The information provided here is only for refrence. It may vary from the original.

www.recruitment.guru

d) 5:6

e) None of these

Ans (a)

102. For which pair of courses is the number of boys the same?

a) E & F

b) A & D

c) C & F

d) B & D

e) None of these

Ans (c)

103. For course E, the number of girls is how much per cent more than the boys for

course E?

a) 250

b) 350

c) 150

d) 80

e) None of these

Ans (a)

104. For which course is the number of boys the minimum?

a) E

b) F

c) C

d) A

The information provided here is only for refrence. It may vary from the original.

www.recruitment.guru

e) None of these

Ans (d)

105. How many girls are there in course C?

a) 44

b) 16

c) 40

d) 160

e) None of these

Ans (b)

Directions (106-110): In each of these questions a number series is given . Only

one number is wrong in each series. You have to find out the wrong number.

106. 10 15 24 35 54 75 100

(a) 35 (b) 75

(c) 24 (d) 15

(e) 54

Ans: (a)

107. 1 3 4 7 11 18 27 47

(a) 4 (b) 11

(c) 18 (d) 7

(e) 27

Ans: (e)

108. 3 2 3 6 12 37.5 115.5

(a) 37.5 (b) 3

(c) 6 (d) 2

(e) 12

Ans: (e)

109. 2 8 32 148 765 4626 32431

(a) 765 (b) 148

(c) 8 (d) 32

(e) 4626

Ans: (d)

The information provided here is only for refrence. It may vary from the original.

www.recruitment.guru

110. 2 3 11 38 102 229 443

(a) 11 (b) 229

(c) 102 (d) 38

(e) 3

Ans: (b)

Directions(111-115): Each of the questions below consists of a questions and two

statements numbered I and II given below it. You have to decide whether the data

provided in the statements are sufficient to answer the questions. Read both the

statements and-

Give answer (a) if the data in statement I alone are sufficient to answer the

questions, while the data in statement II alone are not sufficient to answer the

question.

Give answer (b) if the data in statement II alone are sufficient to answer the

question, while the data in statement I alone are not sufficient to answer the

question.

Give answer (c) if the data either in statement I alone or in statement II alone are

sufficient to answer the questions.

Give answer (d) if the data even in both the statement I and II together are not

sufficient to answer the question.

Give answer (e) If the data in both the statements I and II together are necessary

to answer the question.

111. By selling a product for Rs.100/- how much profit was earned ?

(I) 20% profit would have been earned if it had been sold for Rs.90/-

(II) The profit was one-third of the purchase price.

Ans: (c)

112. A train crosses another train running in the opposite direction in x seconds.

What is the speed of the train?

(I). Both the trains are running at the same speed .

(II) The first train is y cm long.

Ans: (d)

113. The difference between the two digits of a number is 6. What is the number ?

(I). The digit at the unit place is bigger than the other digits .

(II). The sum of the two digits is 12.

Ans: (e)

114. X, Y and Z are integers. Is X an odd number?

(I). An odd number is obtained when X is divided by 5.

(II) (X + Y) is an odd number.

Ans: (a)

115. What is the capacity of a cylindrical tank?

(I) Radius of the base is half of its height which is 28 metres.

(II) Area of the base is 616 sq. metres and its height is 28 metres.

Ans: (c)

The information provided here is only for refrence. It may vary from the original.

www.recruitment.guru

Directions (116-120): Study the following table to answer the given questions.

Percentage of Marks Obtained by Seven Students in Six Subjects

English History Computers Maths Sciences Economics

Student (60) (40) (130) (150) (120) (80)

Meera 100 80 50 90 90 60

Subodh 80 70 80 100 80 40

Kunal 90 70 60 90 70 70

Soni 60 60 65 80 80 80

Richu 50 90 62 80 85 95

Irene 40 60 64 70 65 85

Vijay 80 80 35 65 50 75

116. What are the total marks obtained by Meera in all the subjects?

(a) 448 (b) 580 (c) 470 (d) 74.67 (e) None of these

Ans: (a)

117. What are the average marks obtained by these seven students in History ? (

rounded off to two digits)

(a) 72.86 (b) 27.32 (c) 24.86 (d) 29.14 (e) None of

these

Ans: (d)

118.How many students have got 60% or more marks in all the subjects?

(a) One (b) Two (c) Three (d) None (e) None of

these

Ans: (b)

119.What is the overall percentage of Kunal?

(a) 64 (b) 65 (c) 75 (d) 64.24 (e) None of

these

Ans: (c)

120. In which subject is the overall percentage the best?

(a) Maths (b) Economics (c) History (d) Science (e) None

of these.

Ans: (a)

Directions (121-125): Find out the approximate value which should replace the

question mark (?) in the following questions, (You are not expected to find out the

exact value)

121.95 3.7 ÷95 0.9989 =95?

(a) 1.9 (b) 3 (c) 2.99 (d) 3.6 (e) 2.7

Ans: (e)

122.√1000+ 3.001 of 1891.992 =?

4.987

The information provided here is only for refrence. It may vary from the original.

www.recruitment.guru

(a) 2500 (b) 1230 (c) 1640 (d) 1525 (e) 2130

Ans: (b)

123. 0.0004÷0.0001×36.000009=?

(a) 0.10 (b) 1.45 (c) 145 (d) 14.5 (e) 1450

Ans: (e)

124. 137% of 12345 = ?

(a) 17000 (b) 15000 (c) 1500 (d) 14300 (e) 900

Ans: (a)

125. 3739 + 164 ×27 = ?

a) 105400 (b) 4000 (c) 8200 (d) 690 (e) 6300

Ans: (c)

Section 3: English Language

English Language

Directions(126- 135) Read the following passage to answer the given questions.

Some words have been printed in bold to help you to locate them while answering

some of the questions.

We tend to be harsh on our bureaucracy, but nowhere do citizens enjoy dealing

with their government. They do it because they have to. But that doesn't mean that

the experience has to be dismal. Now there is a new wind blowing through

government departments around the world, which could take some of this pain

away. In the next five years it may well transform not only the way public services

are delivered but also the fundamental relationship between government and

citizens. Not surprisingly, it is the Internet that is behind it. After e-commerce and

e-business, the next revolution may be e-governance.

Examples abound. The municipality of phoenix, Arizona, allows its citizens

to renew their car registrations, pay traffic fines, replace lost identity cards etc.

Online without having to stand in endless queues in a grubby municipal office. The

municipality is happy because it saves $5 a transaction-it costs only $1.60 to

process an online transaction versus $6.60 to do it across the counter. In Chile,

people routinely submit their income tax returns over the Internet, which has

increased transparency, drastically reduced the time taken and the number of errors

and litigation with the tax department. Both tax payers and the revenue department

are happier.

The furthest ahead, not surprisingly, is the small, rich and entrepreneurial civil

service of Singapore, which allows citizens to do more functions online than any

other. As in many private companies, the purchasing and buying of Singapore's

government departments is now on the web, and cost benefits come through more

competitive bidding, easy access to global suppliers and time saved by online

processing of orders. They can post their catalogues on their site, bid or contracts

submit in voices and check their payment status over the Net.

The most useful idea for Indian municipalities is Gov works a private sector-

run site that collects local taxes, fines, and utility bills for 3,600 municipalities

The information provided here is only for refrence. It may vary from the original.

www.recruitment.guru

across the United States. It is a citizen's site, which also provides information on

Government jobs, tenders, etc. The most ambitious is the British Government,

which has targeted to convert 100 percent of its transactions with its citizens to the

internet by 2005.

Cynics in India will say, 'Oh, e-government will never work in India. We are

so poor and we don't have computers.' But they are wrong! There are many

experiments afoot in India as well. Citizens in Andhra Pradesh can download

government forms and submit applications on the Net without having to bribe

clerks. In many districts, land records are online and this has created transparency.

Similarly, in Dhar district to Madhya Pradesh, villagers have begun to file

applications for land transfers and follow their progress on the Net. In seventy

village in the Kolhapur and Sangli district in Maharashtra, internet booths have

come up where farmers can daily check the market and rates of agricultural

commodities in Marathi, along with data on agricultural schemes, information on

crop technology, when to spray and plant their crops and bus and railway time

tables. They also find vocational guidance in jobs, application for ration cards,

kerosene/ gas burners and a land record extracts with details of land ownership.

Sam Pitroda's World Tel, Reliance Industries and the Tamil Nadu Government

are jointly laying 3000km of optic fibre cables to create a Tamil network which

will offer ration cards, school College and hospital admission forms, land records,

and pension records. If successful, World Tel will expand the network to Gujarat,

Karnataka and West Bengal. In Kerala, all the villages are getting linked online to

the district headquarters, allowing citizens to compare the development priorities

of their village with other villagers in the state.

Many are still skeptical of the real impact because so few Indians have

computers. The answer lies in interactive cables. TV and in Internet kiosks.

Although India has only five million computers and thirty-eight million telephones,

it has thirty-four million homes with cable TV and these are growing eight percent

a year. By 2005 most cable homes will have access to the internet from many of

the 700000 local STD/PCO Booths. Internet usage may be low today, but it is

bound to grow rapidly in the future, and e-governance in India may not be a dream.

126. According to the passage, which country has the most ambitious plan for e-

governance ?

(a) USA (b) Chile

(c) Singapore (c) India

(e) UK

Ans: (c)

127. Gov Works in which of the following countries ?

(a) India (b) UK

(c) Chile (d) Singapore

(e) None of these

Ans: (e)

128. Choose the word that is opposite in meaning of the word "dismal" as used in

the passage.

(a) grim (b) approve

(c) pleasing (d) better

The information provided here is only for refrence. It may vary from the original.

www.recruitment.guru

(e) enrich

Ans: (c)

129. How can India overcome low penetration of computers for e-governance ?

(a) By manufacturing more computers

(b) Through cable TV and Internet Kiosks

(c) By opening more STD/ PCO booths

(d) By making the Internet free

(e) By putting more services on internet

Ans: (e)

130. Which of the following has not been one of the effects of submitting income

tax return over internet in Chile ?

(a) Reduction of legal cases

(b) Reduction in errors

(c) Increase in transparency

(d) Increase in no. of returns

(e) Reduction in time taken

Ans: (d)

131. Choose the word that is same in meaning as the word "abound" as used in the

passage.

(a) around (b) proliferate

(c) flourish (d) plentiful

(e) few

Ans: (d)

132.Choose the word that is same in the meaning as the word "post" as used in the

passage

(a) deliver (b) send

(c) put up (d) drop out

(e) later

Ans: (b)

133. According to the passage which country is at present the most advanced in e-

governance ?

(a) Singapore (b) Chile

(c) India (d) USA

(e) UK

Ans: (a)

134. In which direction is the new wind blowing ?

(a) More and more interaction of citizens with government through internet

(d) Outsourcing the work of infrastructure creation for internet

(c) Increasing the penetration of computers in rural areas

(d) Integrating e-commerce, e- business and e-governance

(e) Introducing e-governance programmes in schools and colleges.

Ans: (a)

The information provided here is only for refrence. It may vary from the original.

www.recruitment.guru

135. According to the passage, what is the annual growth rate of computer in India

?

(a) 8% (b) 5%

(c) 0.5% (d) Not mentioned

(e) None of these

Ans: (d)

Directions (136-140): In each of the following questions four words are given of

which two words are most nearly the same or opposite in meaning. Find the two

words which are most nearly the same or opposite in meaning and indicate the

number of the correct letter combination.

136. (A) Proximate (B) Elevated (C) Nimble (D) Agile

(a) A-B (b) B-C (c) A-C (d) B-D (e) C-D

Ans: (e)

137. (A) Notion (B) Symbol (C) Concept (D) Message

(a) A-B (b) B-C (c) A-C (d) C-D (e) B-D

Ans: (e)

138. (A) Limpid (B) Luscious (C) Acrid (D) Benign

(a) A-B (b) B-C (c) C-D (d) B-D (e) A-C

Ans: (b)

139. (A) Asinine (B) Furious (C) Fortunate (D) Ridiculous

(a) A-D (b) A-B (c) A-C (d) B-C (e) B-D

Ans: (a)

140.(A) Companion (B) Amateur (C) Adept (D) Adherent

(a) B-C (b) B-D (c) C-D (d) A-B (e) A-D

Ans: (e)

141. (A) Squander (B) Disunite (C) Flicker (D) Preserve

(a) A-C (b) B-C (c) A-D (d) C-B (e) B-D

Ans: (c)

142.(A) Mitigate (B) Acquiesce (C) Relinquish (D) Duplicate

(a) A-C (b) B-D (c) A-B (d) C-D (e) B-C

Ans: (e)

143.(A) Fable (B) Legend (C) Portrayal (D) Contract

(a) A-C (b) B-C (c) B-D (d) A-B (e) C-D

Ans: (d)

144.(A) Occurrence (B) Pretence (C) Profusion (D) Extravagance

(a) B-D (b) C-D (c) B-C (d) A-C (e) A-B

Ans: (b)

The information provided here is only for refrence. It may vary from the original.

www.recruitment.guru

145.(A) Excellent (B) Passionate (C) Apathetic (D) Discrepant

(a) A-B (b) A-C (c) A-D (d) B-C (e) C-D

Ans: (d)

Directions(146-150): Rearrange the following eight sentences (A), (B), (C), (D),

(E), (F), (G) and (H) in the proper sequence to form a meaningful paragraph; then

answer the questions given below them.

(A) But I always felt somewhere in my mind that I loved acting

(B) He never wanted me to be an actor, as he didn't look upon theatre or acting as

respectable vocation

(C) Firstly there was no tradition of theatre in my family

(D) I am talking specifically of acting, not theatre in general.

(E) My parents were old-fashioned

(F) I will answer all your queries a little elaborately

(G) Let alone theatre, arts in general had no place of respect in my family

(H) My father was a government servant

146. Which of the following will be the LAST sentence?

(a) G (b) C (c) H (d) D (e) E

Ans. (d)

147. Which of the following will be the FIRST sentence?

(a) A (b) B (c) D (d) C (e) F

Ans. (e)

148. Which of the following will be the SIXTH sentence?

(a) B (b) C (c) D (d) D (e) E

Ans. (a)

149. Which of the following will be the FOURTH sentence?

(a) D (b) E (c) A (d) B (e) G

Ans. (b)

150. Which of the following will be the THIRD sentence?

(a) H (b) E (c) G (d) B (e) A

Ans. (c)

Directions (151-155) : Read each sentences to find out whether there is any error

in it. The error, if any, will be in one part of the sentence. The number of the part is

the answer. If there is no error, the answer is 'e'. (Ignore errors of punctuation, if

any)

151. Many animals and plants live in water (a)/ but not in the same kind of water

(b)/ because not all water is the same. (c)/ Sea water, for instance, contains a lot of

salt, fresh water contains very little. (d)/ No error (e)

Ans. (a)

152. A sparrow has made a nest in Kesho's house (a)/ and had laid eggs. Both

Kesho and his sister Shyama (b)/ watched the nest for hours every day. (c)/ Even

The information provided here is only for refrence. It may vary from the original.

www.recruitment.guru

meal times were forgotten. (d)/ No error (e)

Ans. (b)

153. A skilful advertiser may be able to create (a)/ practically a monopoly for

himself. (b)/ not because his product is superior to (c)/ but because he has

succeeded in inducing people to believe that it is (d)/ No error (e)

Ans. (b)

154. Whatever may be the origin of speech, (a)/ we can be certain that man did not

began (b)/ to feel the need to speak (c)/ until he began to live in communities. (d)/

No error (e)

Ans. (b)

155. Each animal of the same species (a)/ looks for the same sort of food. (b)/

Also, there may be other animals of different species (c)/ competing with the same

food. (d)/ No error (e)

Ans. (e)

Directions (156-160) : Pick out the most effective pair of words from the given

pair of words to make the sentence/s meaningfully complete.

156. Of all the problems that have .........................human beings since the

beginning of recorded history, perhaps the most significant has been the ...............

of their own nature

(a) encountered.....................importance (b) perplexed..................value

(c) questioned................ scope (d) confronted.............riddle (e)

directed...............issue

Ans. (d)

157. Self-concept and self-esteem are crucial to personal and professional

effectiveness because if they are not fully ................I may act in mystifying and

sometimes..............ways

(a) positive................destructive (b) developed...............proactive

(c) nurtured.............reactive (d) nourished........listless (e)

devised..........vulnerable

Ans. (c)

158. A key factor in..................an effective and fulfilling life in our complex

society is the ability to.............control over our actions.

(a) achieving...........exert (b) obtaining.............energize (c)

projecting...................restrain

(d) providing..............admonish (e) expressing.............withhold

Ans. (a)

159. A theory is..................evaluated to the degree that its concepts...................to

themselves a verification by independent investigators

(a) critically.............pertain (b) positively.................lend (c)

obviously...............yield

The information provided here is only for refrence. It may vary from the original.

www.recruitment.guru

(d) alarmingly..................introduce (e) delightfully...........submit

Ans. (a)

160. It is often the .............that everyone agrees about objectives; the

disagreement.....................about exactly how they are to be achieved

(a) view.......................follows (b) situation...................talks (c)

matter................projects

(d) happening.............matters (e) case...............arises

Ans. (e)

Directions (161-165) : Which of the phrases (a), (b), (c) and (d) given below

should replace the phrase given in bold in the following sentence grammatically

correct.. If the sentence is correct as it is & no correction is required, mark (e) as

the answer.

161. He went to the blast site to save his colleague who was stranding there

(a) had been stranded (b) had to strand (c) was being stranding (d) was been

stranding (e) No correction required

Ans. (a)

162. When I receive the letter, the date for the interview was already over

(a) I received the letter (b) the letter I had received (c) I had to receive letter (d) I

was receiving

(e) No Correction required

Ans. (a)

163. Everyone is requested to attend the dinner party hosted by the president

(a) is requesting to (b) is requested to (c) will request to (d) is to requested

to (e) No correction required

Ans. (e)

164. The chairman approved the recommendations to the committee with partial

modifications

(a) by the committee for (b) of the committee with (c) to the committee (d) at the

committee in

(e) No correction required

Ans. (b)

165. Had the opportunity been given to him he will have proved successful

(a) will have to prove (b) will be proving (c) would have proved (d) would

have proven (e) No correction required

Ans. (c)

(Directions 166-170) : In the following passage there are blanks each of which

has been numbered. These numbers are printed below the passage and against

each five words are suggested, one of which fits the blank appropriately. Find out,

the appropriate word in each case.

In an upper class drawing room in Mumbai, a dozen or so (166) the city's

intelligentsia is (167) on the dregs of what was a live-wire party an hour ago.

The information provided here is only for refrence. It may vary from the original.

www.recruitment.guru

Poets, editors, writers, filmmakers, all card-carrying members of the rich and

powerful, not at this late hour, on their ninth drink, are (168) in what Indians are

best known to do: Media-bashing, armchair criticism and name-calling.

Inevitably the talk turns to Tehelka, and its managing director and editor-in-chief,

Tarun Tejpal, and the party quickly degenerates into an orgy of guilt and shame

and that peculiar habit of the Indian intelligentsia : passing the buck, with every

one present attacking his neighbour for not supporting Tejpal more. It is a trait

Tejpal is (169) with and has been telling me about in the weeks preceding this

interview.

"Often at parties when someone (170) me 'great job, carry on what you are doing',

and showers with me staggering praise, staggering love, staggering (171) just smile

and let it pass as i don't want to score a brownie point and say, 'and what about

your boss, what will you do?"

This lightly-tossed sentence with the slightest hint of a dark chuckie is the only

time I have been Tejpal allow for bitterness, though I have been (172) him for

weeks for sings of it. Because for a man who has been the victim of the most

(173) government witch-hunts in recent years, Tejpal is astonishingly devoid of

(174) . You expect him to rage, to spew fire, to heap scorn and anger at his

enemies, and instead, what you encounter, is a man in whose eyes you see only

compassion and a weary understanding of the nature of the beast.

The abiding irony in all this is that unlike his armchair supporters in the middle

class, who rave and rant on his behalf, but do nothing else, Tejpal displays

forbearance. " The fact that I am essentially a literary animal and that my entire

sensibility has been shaped by literature has helped. I'm not a black-and-white

person. I'm a (175) who understand the greys", he says to me at a coffee shop, over

tea.

166. (a) of (b) in (c) into (d) off (e) from

167. (a) feed (b) feeds (c) fed (d) feeding (e) feeded

168. (a) mixing (b) lending (c) scaring (d) indulging (e) pushing

169. (a) ordinary (b) familiar (c) strange (d) free (e) routine

170. (a) bold (b) said (c) ask (d) say (e) tells

171. (a) delight (b) contempt (c) worship (d) affection (e) admiration

172. (a) lure (b) teasing (c) baiting (d) harassing (e) matching

173. (a) gentle (b) good (c) various (d) vicious (e) wicked

174. (a) forbearance (b) anger (c) wrath (d) forgiveness (e) gale

The information provided here is only for refrence. It may vary from the original.

www.recruitment.guru

175. (a) boy (b) guy (c) youth (d) human being (e)

follow

Section 4: General Awareness

176. On which of the following dates was the National Centre for Promotion of

Employment for Disabled People-Shell Helen Keller Awards-2007 presented?

(a) December 1, 2007 (b) December 2, 2007 (c) December 3, 2007 (d) December

4, 2007 (e) None of these

Ans. (b)

177. The nuclear power plant of 220 MW which was recommissioned on

December 2, 2007, after 'en-ass coolant channel replacement (EMCCR)'

is______________

(a) Kalpakam (b) Kota (c) Narora (d) Rawatbhatta (e)

None of these

Ans. (c)

178. The former South African cricketer who, on December 4, 2007 signed two-

year contract with Board of Control for Cricket in India (BCCI) coach the Indian

cricket team is ______________

(a) Boeta Dippenar (b) Jack Russel (c) Allan Donald (d)

GaryKirsten (e) None of these

Ans. (d)

179. Which of the following Indian cities hosed the East Asia Gender Equality

Ministerial Meeting, in December 2007, attended by ministerial level

representatives from several countries and various United Nations agencies?

(a) New Delhi (b) Chennai (c) Kolkata (d) Mumbai (e) None of

these

Ans. (a)

180. South Africa's former Deputy President who, on December 19, 2007 defeated

Thabo Mbeki, to become the new President on African National Congress, at a

party conference the northern town of Polokwane is

(a) Susan Booysen (b) Nelson Zulahe (c) Jacob Zuma (d) Momir Williams

Junior (e) None of these

Ans. (c)

181. Which of the following countries, for the first time, attended a summit the six

Arab countries belonging the Gulf Cooperation Council (GCC) in Doha, Qatar, in

December 2007?

(a) Egypt (b) Syria (c) Iran (d) Iraq (e) None of these

Ans. (c)

182. United Russia which won the Russian Parliamentary elections in December

2007, captured_____________ seats the 450-seat Duma, the Lower House of

Russia's Parliament.

The information provided here is only for refrence. It may vary from the original.

www.recruitment.guru

(a) 290 (b) 315 (c) 367 (d) 410 (e) None of these

Ans. (b)

183. The 4300-year-old Chinese oil capital of China's oldest Liangzhu Culture,

which was dug up by archaeologists in December 2007 is located in eastern

China's _______________ province

(a) Fujian (b) Zhejiang (c) Anhui (d) Jiangsu (e) None of these

Ans. (b)

184. With which of the following companies did Reliance Industries, on December

4, 2007 sign a Memorandum of Understanding (MoU) for joint-co-operation in

Petrochemicals?

(a) Oil India Ltd (b) Indian Oil Corporation (c) GAIL (India) Ltd (d) Hindustan

Petroleum Corporation Ltd

(e) None of these

Ans. (c)

185. The Central Government, in December 2007, allowed ______________ based

Dolce & Gabbana and India's realty firm DLF to set up a joint venture in single

brand retailing of lifestyle products

(a) Italy (b) France (c) Britain (d) USA (e) None of these

Ans. (a)

186. Which of the following companies, on December 4, 2007 signed a 'large

Account Reseller Agreement' with Microsoft Corporation India to provide

integrated technology solution to Indian enterprises?

(a) Dell (b) Lenovo (c) Compaq (d) Toshiba (e) None of

these

Ans. (a)

187. Which of the following Indian Companies, in December 2007, opened 160

stores to its specialty beauty, health and wellness retail business subsidiary, H & B

Stores, under the new brand name "new-u"?

(a) Reliance (b) Dabur (c) Unilever (d) Subhiksha (e) None of

these

Ans. (b)

188. Whom did Neha Aggarwal of Delhi beat in the finals to win the youth girls'

crown in the national youth and junior table tennis championship in Indore on

December 21, 2007?

(a) Madhurika Patkar (b) Ashelsha Bodas (c) Priti Mokashi (d) Pooja

Sahasrabudhe (e) None of these

Ans. (d)

189. The tennis player from Andhra Pradesh who, on December 22, 2007, beat

Divij Sharan of Delhi, to win the men's singles title in the 2007 National grasscourt

championship in Kolkata is :

(a) J. Vishnu Vardhan (b) Vinod Sridhar (c) V. M. Ranjeeth (d) N. Sriram

The information provided here is only for refrence. It may vary from the original.

www.recruitment.guru

Balaji (e) None of these

Ans. (a)

190. The 30-year-old Armyman form Maharashtra who, on December 23, 2007

won the men's event in the Mawana Sugars Indian Open Marathon in New Delhi is

___________

(a) Angad Kumar (b) Rashpal Singh (c) Satya Prakesh (d) Birender Singh

Chahar (e) None of these

Ans. (c)

191. Which of the following countries in December 2007, won the Davis Cup

(tennis) in Portland (U. S.) after a long gap of twelve years?

(a) Russia (b) United States of America (c) Sweden (d) Germany (e)

None of these

Ans. (b)

192. Miss Angola who came first runner-up in the Miss World 2007 pageant held

in Sanya, China, in December 2007 is

(a) Moran Gordillo (b) Micaela Reis (c) Sandy Dudette (d) Adyshe

Barbara (e) None of these

Ans. (b)

193. Which of the following South American Countries on December 1, 2007

threatened to halt the OPEC nation's oil sales to the United States is

(a) Venezuela (b) Peru (c) Chile (d) Brazil (e) None of

these

Ans. (a)

194. The renowned sitar maestro of Dharwad, contemporary of the legendry

Hindustani vocalist the late Ustad Abdul Kareem Khan Saheb, who passed away in

Belgaum on December 2, 2007 is__________

(a) Maznoo Mia (b) Allah Rakha Khan (c) Bale Khan (d) Aftab

Rasool (e) None of these

Ans. (c)

195. Who was elected President of the Ramakrishna Math and Mission at a

meeting of the Board of Trustees of the Match and the governing body of Mission

in Kolkata on December 3, 2007?

(a) Atmasthananda (b) Ramananda (c) Parmananda (d)

Vijnananada (e) None of these

Ans. (a)

196. The Executive Director of the Global Fund to Fight AIDS, Tuberculosis and

Malaria, who on December 20, 2007 renewed its grant to India, worth more than $

100 million for the next three years till 2010 is __________

(a) Michael Kazatchleine (b) Francois Dippennar (c) Alex Mulromy (d)

Deniel Mozzart (e) None of these

Ans. (a)

The information provided here is only for refrence. It may vary from the original.

www.recruitment.guru

197. "Hand-in-Hand 2007" the first-ever joint Sino-Indian anti-terrorism training

exercise was held at the Kuming Military academy in China in December 2007.

Kuming is the capital of south-western ____ province

(a) Hunan (b) Yunnan (c) Shandong (d) Zheziang (e) None of

these

Ans. (b)

198. The ten-year old Indian boy who, in December 2007, emerged as the youngest

champion in the history of 'Junior Mastermind; a BBC Quiz show is ___________

(a) Swapnil Shankar (b) Madhav Prasoon (c) David Varghese (d) Krishna

Routrey (e) None of these

Ans. (c)

199. Bharatiya Janata Party came back to power for the consecutive ___________

time in Gujarat on December 23, 2007 the day when the result of the Assembly

elections were declared?

(a) second (b) third (c) fourth (d) fifth (e) None of these

Ans. (c)

200. Whom did Nick Clegg replace to become the new leader of Great Britain's

Liberal Democratic Party in London on December 18, 2007?

(a) Richard Keen (b) Marshall Daniel (c) Menzies Champbell (d) Alistaire

Harrod (e) None of these

Ans. (c)

201. Right-wing South Korean businessman who, on December 19, 2007 defeated

left-of-centre Chung Dong-young to become the new President of South Korea is

(a) Kim Yoon-Ok (b) Lee Myung-bak (c) Ahn Sang-sao (d) Mile Jen-

foo (e) None of these

Ans. (b)

202. An earthquake of 6.8 magnitude on the Richter scale struck New Zealand's

North Island on December 20, 2007 causing massive damage in the city of

___________

(a) Auckland (b) Hamilton (c) Napier (d) Gisborne (e) None of

these

Ans. (d)

203. The passport-for Schengen Area of Europe saw its biggest-ever enlargement

on December 21, 2007 when____________ new European Union (E. U.) members

began to implement the Schengen Agreement

(a) five (b) seven (c) nine (d) eleven (e) None of these

Ans. (c)

204.GE Energy, on December 21, 2007 bagged its largest order for supply of

Jenbacher gas engines to support a major rural electrification initiative in

(a) Kenya (b) Bangladesh (c) Somalia (d) Ecquador (e) None of

these

Ans. (b)

The information provided here is only for refrence. It may vary from the original.

www.recruitment.guru

205. In December 2007, Nasdaq opened an office in the Asian capital-city of

_________aimed at stepping up efforts at attracting more firms to list on the

exchange and forging deeper ties with bourses and regulators

(a) Seoul (b) Tokyo (c) Beijing (d) Manila (e) None of these

Ans.(c)

206. In which of the following capital cities, of Gulf Cooperation Council

countries, did Union Bank of India open its representative office in December

2007?

(a) Riyadh (b) Doha (c) Kuwait City (d) Abu Dhabi (e) None of

these

Ans. (d)

207. The _________ based Prudential Finance (INC (PFI), on December 4, 2007

announced an asset management joint venture with real estate giant DLF with

which it already has a life insurance partnership

(a) Canada (b) U. S. A. (c) South Africa (d) Singapore (e) None of

these

Ans. (b)

208. Which of the following teams was defeated by Chennai Superstars to win the

ICL (Indian Cricket League) Twenty-20 final in Panchkula on December 16,

2007?

(a) Chandigarh Lions (b) Kolkata Tigers (c) Hyderabad Heroes (d)

Delhi Jets (e) None of these

Ans. (a)

209. Who, among the following golfers, won the Target World Challenge Cup in

Thousand Oaks on December 16, 2007?

(a) Zach Johnson (b) Lee Westwood (c) Tiger Woods (d) Henrik

Stenson (e) None of these

Ans. (c)

210. The American cyclist, stripped of his 2006 Tour de France title for doping,

who was banned from all competitions in France by the country's anti-doping

agency AFLD, on December 19, 2007 is ___________

(a) Fabio Lefebore (b) John Carragher (c) Desmond Capello (d) Floyd

Landis (e) None of these

Ans. (d)

211. Whom did Zubin Kumar beat in the finals to win the youth boys' crown in the

National youth and junior table tennis championship in Indore on December 21,

2007?

(a) Devesh Karia (b) G. Sathiyan (c) Raj Mondal (d) Satyajit

Prasanna (e) None of these

Ans. (a)

212. The French Minister for Foreign and European Affairs who visited India in

December 2007 to prepare the ground for French President Nicolas Sarkozy's visit

The information provided here is only for refrence. It may vary from the original.

www.recruitment.guru

in January 2008 is

(a) Bernard Kouchner (b) Andrei Strahler (c) William Tarbuck (d)

Philip Randell (e) None of these

Ans. (a)

213. Hollywood star in box-office hits like 'Monster-in-Law', fitness guru and

activist who turned seventy years old on December 21, 2007 is ___________

(a) Olivia Silverstone (b) Lisa Marigold (c) Jane Fonda (d) Elizabeth

Turner (e) None of these

Ans. (c)

214. Which of the following countries on December 23, 2007 conclude the annual

central rural work conference with a pledge to improve rural infrastructure,

promote stable development of agriculture and facilitate a sustained income growth

for farmers in the year 2008?

(a) Brazil (b) China (c) Canada (d) Argentina (e) None of these

Ans. (b)

215. The former British Prime Minister who, in December 2007, was welcomed

into Britain's Catholic community after his/ her conversion to the Catholic Church

is___________

(a) John Major (b) Margaret Thatcher (c) Rowan Williams (d) Tony

Blair (e) None of these

Ans. (d)

216. After 16 months of marathon hearing the inquiry into the 1985 Kanishka

bombing ended in Toronto on December 13, 2007. The bombing of the Air India

flight killed _________ people

(a) 117 (b) 233 (c) 329 (d) 409 (e) None of these

Ans. (c)

217. Which of the following countries, on December 17, 2007, expressed its

interest in investments by Indian corporates for construction and promotion of

budget hotels in the island nation?

(a) Sri Lanka (b) Mauritius (c) Maldives (d) Tuvalu (e)

None of these

Ans. (a)

218. The former managing editor of SAGE India and longtime Doordarshan news

reader who died in Landour (above Mussorie) on December 16, 2007 is

(a) Pankaj Singh (b) Rajesh Sharma (c) Mohanlal Chaturvedi (d)

Tajeshwar Singh (e) None of these

Ans. (d)

219. The editor of Jaipur-based monthly journal, "Sharad Krishi" who was on

December 18, 2007 presented the prestigious Atmaram Prize for developing

scientific and technical literature in Hindi is

(a) Mahendra Madhup (b) Anupam Shrivastava (c) Dhirendra Singh

The information provided here is only for refrence. It may vary from the original.

www.recruitment.guru

Parmar (d) Gaurav Bainsala (e) None of these

Ans. (a)

220. The Supreme Court of ___________ sentenced the former President, Alberto .

K. Fujimori, on December 11, 2007 sentenced the former President K. Fujimori to

six years in prison for ordering an illegal search as his government was collapsing

in 2000

(a) Chile (b) Peru (c) Colombia (d) Mexico (e) None of these

Ans. (b)

221. The United States and China, in December 2007, held the Sino-U. S. Strategic

Economic Dialogue (SED) in

(a) Beijing (b) New York (c) Oslo (d) Seattle (e) None of these

Ans. (a)

222. Thousands of workers in which of the following countries, on December 15,

2007 march through the national capital to protest against the pressures on life and

fuel and food prices due to the failure of the political leaders to form a government

after the June 2007 general elections?

(a) Slovakia (b) Czech Republic (c) Romania (d) Belgium (e)

None of these

Ans. (d)

223. The well-known British born science fiction writer and author of the book

"2001 : A Space Odyssey" who was honoured by the Sri Lankan government, on

December 16, 2007 for his contribution to the world of space exploration

is___________

(a) Daniel Bald (b) Arthur Clarke (c) Anthony Cordesman (d) Boris

Schlesinger (e) None of these

Ans. (b)

224. Which of the following car companies in India announced a special savings

scheme________ 'Happy Customer Offer' - in December 2007?

(a) Maruti Suzuki (b) Tata (c) Ford (d) Hyundai (e) None of these

Ans. (a)

225. The Eleventh Plan Document approved by the National Development Council

(NDC) on December 19, 2007 envisages a growth of _____ percent during 2001-

12 the terminal year of plan period.

(a) Eight (b) Nine (c) Ten (d) Eleven (e) None of these

Ans. (c)

The information provided here is only for refrence. It may vary from the original.

www.recruitment.guru

Explanations

(1-5): @ = > * = > # = =

$ = < + = <

1. Statements

D + T, E $ V, F * T, E @ D

After conversion

D < T, E < V, F > T, E > D

Or V > E > D < T < F

Conclusion

I . D$ V = D , < V : Not true

D is similar than V.

II. D + F + = D < F : True

2. Statements

B + D, E $ T, T * P, P @ B

After conversion

B < D, E < T, T > P, P > B

Or, E < T > P > B < D

Conclusion

I . P $ D = P < D : Not true

II. P @ D = P > D : Not true

While considering the relation between two entities three possibilities exist: greater

than, equal to or smaller than.

Therefore either I or II is true.

3. Statements

T * U, U $ W, V @ L, W + V

After conversion

T> U, U < W, V > L, W< V

Or T > U < W < V > L

Conclusion

I. V @ T = V > T : Not true

II. L # W = L > W : Not true

4. Statements

P $ Q, N # M, M @ R, R * P

After conversion

P < Q, N = M, M > R, R > P

Or N = M > R > P > Q

Conclusion

I. P + N = P < N : Not true

II. Q $ W = Q < M : Not true

5. Statements

E * F, G $ H, H # E,G @ K

After conversion

E > F, G < H, H = E, G > K

Or , K < G <H = E > F

Conclusion

I. H @ K = H > K : Not true

II. H $ F = H > K : Not true

The information provided here is only for refrence. It may vary from the original.

www.recruitment.guru

(6-11):

(i). There are altogether 27 elements in the above sequence.

(ii). There are only 14 letter in the above sequence.

(iii) There are only 9 digits in the above sequence.

(iv) There are only 4 symbols in the above sequence.

(v) The middle terms of the sequence is P.

6. 5 +6 M +6 T +6 1

E +6 J +6 U +6 H

$ +6 P +6 7 +6 Q

Therefore, ? = TU7

7. 11th from left and =1

7th from right end = 1

Remaining elements between I and 1:

K * P T @ U 9 A 7

Middle Term

8. 6th to the right of20th from the right end means 20-6 =14th from the right end.

There are altogether 27 elements in the above sequence and the middle term, i.e.,

Therefore, our required answer is option.

9. Digit Digit Digit

There is no such combination.

10. 8th to the left of 13th element from right end means 13 + 8 =21st from the right end.

21st element from the right end is equivalent to 28 – 21 = 7th elements from the left and

vice-versa.

1st 15th

2nd 14th

3rd 13th

4th 12th

5th 11th

6th 10th

7th 9th

Therefore, required element would be 9th from the left in the original sequence.

9th from left --> M.

11. According to question,

A

5

B

2

C

4

D

6

E

5

F

8

G

10

H

12

I

5

J

14

K

16

L

18

M

20

N

22

O

5

P

24

Q

26

R

28

S

30

T

32

U

5

V

35

W

36

X

38

Y

40

Z

42

Now,

The information provided here is only for refrence. It may vary from the original.

www.recruitment.guru

C U S T O M

4 + 5 + 30 +32 + 5+20 = 96

12.

Such combinations are :

13. Option (1)

P x Q ÷ R + S – T

P x Q --> P is the wife of Q

Q ÷ R --> Q is the father of R

R + S * R is the sister of S

S – T --> S is the brother of T

Note. The sex of T is not known

Deductions

(i) Q is the husband of P

(ii) P is the mother of R, S and T

(iii) Q is the father of R,S and T

(iv) R is the sister of S and T

(v) S is the brother of R and T

Option (2) P x Q --> P is the wife of Q

Q ÷ R --> Q is the father of R R- T --> R is the brother of T

T + S --> T is the sister of S

Note: The sex of S is not known

Deductions

(i) Q is the husband of R

(ii) P is the mother of R,S and T

(iii) Q is the father of R, S and T

(iv) T is the daughter of P and Q

14. According to questions, the new sequence would be:

N O P Q R S T U V W X Y Z A B C D E F G H I J K L M

9th to the right of 17th letter from the right means (17-9) =8th letter from the right.

8th letter from right = F

(15-18): On the basis of given information and conclusion as well as sub-conclusion drawn

from them we can construct the following chart:

P Field Destination

P Finance Lucknow

a + b < x + y

5

2 + 4 < 5 + 4

5

4+ 5 < 4 + 8

5

8+ 7 < 9 + 8

5

The information provided here is only for refrence. It may vary from the original.

www.recruitment.guru

Q HRM _____

R ____ Calcutta

Bangalore

S ____ Calcutta

Bangalore

T Administration Chennai

U Marketing ____

W System Delhi

19. N U R S I N G

There are three such pairs.

(20-26): On careful analysis of the given passcodes it is evident that the last word of the

previous code becomes the first word for the next batch and the first and the second words

shifted to the second and the third positions respectively. Again the seventh and the sixth words

occupy the fourth and fifth positions respectively and then the third, the fourth and the fifth

words of the previous code are written in the same order in the passcode for the next batch.

Passcode for Batch I

1 2 3 4 5 6 7 8

Cloths neat and clean liked are all by

Passcode for Batch II

8 1 2 7 6 3 4 5

By cloths neat all are and clean liked

1 2 3 4 5 6 7 8

Passcode for Batch III.

8 1 2 7 6 3 4 5

Liked by cloths clean and neat all are

1 2 3 4 5 6 7 8

Passcode for Batch IV

8 1 2 7 6 3 4 5

Are liked by all neat cloths clean and

1 2 3 4 5 6 7 8

The information provided here is only for refrence. It may vary from the original.

www.recruitment.guru

Passcode for Batch V

1 2 3 4 5 6 7 8

And are liked clean cloths by all neat

1 2 3 4 5 6 7 8

Passcode for Batch VI

1 2 3 4 5 6 7 8

Neat and are all by liked clean cloths

Now on the basis of Above analysis we can summarise the procedure for getting passcode for

each subsequent batch with reference to the passcode fro batch I in the following manner.

Passcode for Batch I

1 2 3 4 5 6 7 8

Clothes neat and clean liked are all by

Passcode for Batch II

8 1 2 7 6 3 4 5

By cloths neat all are and clean liked

Passcode for Batch III

5 8 1 4 3 2 7 6

Liked by cloths clean and neat all are

Passcode for Batch IV

6 5 8 7 2 1 4 3

Are liked by all neat cloths clean and

Passcode for Batch V

3 6 5 4 1 8 7 2

And are liked clean cloths by all neat

The information provided here is only for refrence. It may vary from the original.

www.recruitment.guru

Passcode for Batch VI

2 3 6 7 8 5 4 1

Neat and are all by liked clean cloths

Thus, we can write any required step i.e., passcode for any batch from the given passcode

directly.

20. Passcode for Batch III

5 8 1 4 3 2 7 6

Night succeed day and hard work to for

Passcode for Batch VI

2 3 6 7 8 5 4 1

Work hard for to succeed night and day

21. Passcode for Batch V

3 6 5 4 1 8 7 2

Visit in zoo should the we time day

Given Passcode

5 8 1 4 3 2 7 6

Zoo we the should and day time in clearly, this is the passcode for Batch III

22. Passcode for Batch IV

6 5 8 7 2 1 4 3

To fats rush avoid not do very run

Passcode for Batch II

8 1 2 7 6 3 4 5

Rush do not avoid to run very fast.

23. Passcode for Batch II

8 1 2 7 6 3 4 5

Length the day equal of and night are

Passcode for Batch V

3 6 5 4 1 8 7 2

The information provided here is only for refrence. It may vary from the original.

www.recruitment.guru

And of are night the length equal day

24. Passcode for Batch II

8 1 2 7 6 3 4 5

To confidence hard you leads work and success

Passcode for Batch IV

6 5 8 7 2 1 4 3

Leads success to you hard confidence and work

25. Passcode for Batch VI

2 3 6 7 8 5 4 1

And pencil are used are pen

Passcode for Batch I

1 2 3 4 5 6 7 8

Pen and pencil are used by all boys

26. Passcode for Batch IV

6 5 8 7 2 1 4 3

Not go the way to of out do

Passcode for Batch III

5 8 1 4 3 2 7 6

Of to do out go not way the

27. First and third premises are particular Affirmative, i.e., I-type.

Second premise is Universal Affirmative (A-type).

1. Some books are pens

2. All pens are chairs.

We know that,

I + A = I-type conclusion

Therefore, our derived conclusion would be:

“Some books are chairs”

This is the conclusion I.

Conclusion II is the conversion of our derived conclusion.

The information provided here is only for refrence. It may vary from the original.

www.recruitment.guru

Two out of three premises are Particular and hence, Universal conclusion is invalid. That

is, conclusion III does not follow.

Conclusion IV is the conversion of the third premise. Therefore, only conclusion I, II and

IV follow.

28. All the three premises are Universal affirmative (A-type).

1. All cars are jeeps.

2. All jeeps are buses.

We know that

A + A = A-type conclusion

Therefore, our derived conclusion would be:

“All cars buses’.

There is no such conclusion

Now,

All cars are buses.

All buses are trucks

We know that,

A + A = A-type conclusion

Thus, our derived conclusion would be:

“All cars are trucks”:

This is the conclusion IV.

Again,

All jeeps are buses.

All buses are trucks.

We know that,

A + A = A-type conclusion

Thus, our derived conclusion would be:

“All jeeps are trucks”

Therefore, only conclusion IV follows.

Thus, our required answer is option (d)

VENN -DLAGRAM

Tr

u

ck

B

us Je

e

p

C

ar

The information provided here is only for refrence. It may vary from the original.

www.recruitment.guru

29. All the three premises are Particular Affirmative (I-type). Therefore, no conclusion can be

derived from these premises.

Now look for any conversion and/or implication: There is no such conclusion.

30. First premises is Universal Affirmative (A-Q type).

Second premises is Particular Affirmative (I-type).

Third premises is Universal Affirmative (A-type).

Some bricks are ropes.

All ropes are doors.

We know that,

I +A = I-type conclusion.

Conclusion: Some bricks are doors. It is conversion of conclusion II.

Conclusion I and II from complementary pair. Therefore either conclusion I or II follows.

Therefore, our required answer is option(c).

31. First and second premises are Particular Affirmative i.e., I-type. Third premise is Universal

Affirmative (A-type).

Conclusion I is the conversion of the third premise.

Second and third premises are relevant for the conclusion II. Thus,

Some pens are watches.

All watches are radios.

We know that,

I + A =I type conclusion.

Conclusion : Some pens are radios .

Conclusion II is the conversion of this conclusion.

Therefore, only conclusion I and II follow.

VEENN DLAGRAM

Pen Book

W

atc

he

s

Ra

dio

s

The information provided here is only for refrence. It may vary from the original.

www.recruitment.guru

32. First premise Universal Affirmative (A-type)

Second premise Universal Negative (E-type)

Third premise Particular Affirmative (I-type)

All towns are villages.

No village is forest.

We know that,

A + E = E type conclusion

Conclusion: No town is forest

There is no such conclusion

All towns are villages.

No village is forest.

We know that,

E + I= O I-type conclusion

Thus, our derived conclusion would be :

“Some rivers are not villages.”

This is the conclusion III.

Again.

No town is forest.

Some forests are rivers.

We know that,

E+I = O1- type conclusion

Conclusion: Some rivers are not towns.

There is no such conclusion.

Conclusion II is the conversion of the second premise.

Therefore Conclusion II and III follow.

33. 11 Boys 12th

After shifting his place

The information provided here is only for refrence. It may vary from the original.

www.recruitment.guru

11 Boys 12 13 14 15 16

18th 17 Boys

Total Number of boys in the row

= (16 + 18) -1 = 33

34. 1 2 3 4 5 6 7

P R O B L E M

It has been coded as

7 1 6 2 5 3 4

M P E R L O B

Similarly

1 2 3 4 5 6 7

N U M B E R S

It code would be:

7 1 6 2 5 3 4

S N R U E M B

35. Both the assumptions are implicit in the statements. It is clearly mentioned in the statements that the Government has hiked the prices of diesel and petrol to reduce the oil pool deficit. Whenever the prices of commodities are hiked, generally people raised the voice against such measure. 36. The price any product is lowered assuming that its demand will increase. Therefore, assumption I is implicit in the statement. 37. From the content of the statement it is clear that both the assumptions are implicit in the statement. 38. Only assumption II is implicit in the statement. If Aswin’s mother asked his son to return home by train if it rains heavily, it implies that the trains would ply even if it rains heavily. 39. Both the assumptions are implicit in the statement. 40. Both the assumptions are implicit in the statements. 41. Only argument II is strong. 42. Neither of the argument is strong. 43. Only argument I is strong. Now a days television is an, essential means to provide useful academic information. Therefore, it is not desirable to prevent children from watching selected programmes on television. For the same reason, argument II is invalid. 44. Only argument I is strong. It is true that employees put their genuine demands before the management through the trade unions. Therefore, it is not judicious to ban the trade unions

The information provided here is only for refrence. It may vary from the original.

www.recruitment.guru

completely. It is true that employees compel the management in some instances to concede their some illegal demands through the unions but the solution suggested is not appropriate. 45. Only argument I is strong. Women are equally capable and they should be given equal opportunity in matter of employment. Argument II makes no point. 46. Both the conclusions logically follows from the information given in the statement. The term milk glut clearly indicates that the milk production of State ‘X’ is more than its requirement. Again, it is clearly mentioned that the Government and co-operative dairies in State ‘X’ failed to use the available milk it implies that such dairies State ‘X’ are not equipped properly. 47. Neither conclusion I nor conclusion II follows. The subsidy on any item cannot be 100 per cent. It is mentioned in the statement the Government has decided to withdraw 3 per cent of “the subsidy” on cooking gas and not 33 per cent of the actual price of the cooking gas. Therefore, it is erroneous to assume that the price of the cooking gas will increase at least by 33 per cent. Any policy of the Government affects all sections of the society and it cannot be assumed that poor people do not need subsidy. Thus, neither of the conclusion follows. 48. From the statement it is clear that the Government will either increase the price of diesel or will not increase the price of diesel. The deficit on this count can be adjusted by some other means. Therefore, after the review of the present policy of the diesel price in view of further spurt in the international oil prices, the Government is left with only two options to increase or not to increase the price of diesel. Therefore, either conclusion I or conclusion II follows. 49. Neither of the conclusions follows. It is clear from the statement that the City ‘Z’ requires beautification but this does not imply that the people of that city are unaware about the state of ugliness of their city. Therefore, conclusion I does not follow. The new Municipal Commissioner asserted that the task of beautification of City ‘Z’ could also be accomplished if such has happened in City ‘X’ and City ‘Y’ . From this it cannot be derived that the Municipal Commissioner has served in City ‘X’ and City ‘Y’ while asserting something one may quote the works of another person. Hence conclusion II is also not valid. 50. Neither conclusion I nor conclusion follows. In the absence of harsh punitive measures the cases of sexual harassment may take place anywhere. Therefore, it is erroneous to assume that sexual harassment of women at work place is more prevalent in India as compared to other developed countries. The statement exhorts that amendment in the Act is meant to curb sexual harassment at the work place and it cannot be taken as the imposition of restriction on recruitment of women, In order to avoid pain in arms we cannot chop our arms. 51. It is not given in the statement I That Ashok or Ravi is facing towards which direction. Therefore, it is not possible to determine the direction of right side of Ravi. From statement II Samir is sitting opposite of Ashok facing north. It implies that Ashok is facing toward south. From both the statements Ashok is facing toward south and is sitting to the right of Ravi. It implies that Ravi is facing toward south or north. Therefore, required answer is (d). 52. From statement I The mother of N is the daughter of M. The mother of N is the sister of K. Therefore, the mother of N and K are the children or M. But it is not clear whether M is the father or mother of K. From statement II P is the wife of M. Therefore, M is the husband of P. There is no information about K in statement II. From both the statements it is clear that M is the father of K.

The information provided here is only for refrence. It may vary from the original.

www.recruitment.guru

53. From statement I T is the sister of M and K. There is no information about D in the statement I. It is not clear whether M and K are males or females. Thus, the data in statement I alone are not sufficient to answer the question. From statement II K is the brother of D. There is no information about T in the statement U. We can’t determine the sex of D even with the help of data given in the statement I and II. Therefore, our required answer is option(d). 54. There is no information about P in the statement. Therefore, the data in statement I alone are not sufficient to answer the question. But it is clear that M,S and T are brothers and or sister to one another. From statement II P is the mother of T and she has only one daughter. It implies that T is the daughter of P. From statement I and II P has two sons S and T,M is the daughter of P. Therefore the data given in both the statement I and II are necessary to answer the question 55. From statement I R > Q, T There is no information P and S in the statement I. Thus, the data given in the statements I alone are not sufficient to answer the question. From statement II T > S, P S > Q, R T > S > Q, R

P It is clear that T is the tallest among them. Thus, the data in the statement II along are sufficient to answer the question, while the data in the statement I alone are not sufficient to answer the question. 61. The movement and changes of designs from Problem Figure (a) to (b) can be shown as: New design From Problem Figure (d) to (e) New design From Problem Figure (b) to (c) similar changes would occur as that have been occurred from Problem Figure (d) to (e) and new design (p) would appear at the lower middle position because starting from Problem Figure (b) a new design appears in the clockwise direction in the subsequent figures. 62. From Problem Figure (d) to (e) the lower design is reversed laterally while the other design moves to the opposite side. Similar changes would occur from Problem Figure(b) to (c) .

• • •

• • •

• • •

• • •

The information provided here is only for refrence. It may vary from the original.

www.recruitment.guru

63. In each subsequent figure all the designs move one step in clockwise direction and one of the designs gets paired. 64. In each, subsequent figure the plane of designs rotates through 45° anticlockwise. From Problem Figure (d) to (e) the fourth design from right is shifted to the first position while all other designs move one step and the last design is replaced by a new design. Similar changes would occur from Problem Figure (b) to (c) . 65. Each of the arcs rotates 90° clockwise or anticlockwise and 180° alternately. 66. The movement of designs from unnumbered figure to figure (a) can be shown as:

New design The movement of design from figure (a) to figure (b) can be shown as: New design These two steps are repeated alternately . In figure. There should be (P) instead of (S). 67. The last figure is inverted form of the first figure. Figure (d) is the inverted form of figure (b). Therefore, figure (e) should be inverted form of figure (a). 68. In the first step one line segment is deleted from the upper figure while the lower figure is inverted. In the next step one line segment is deleted from the lower figure while the upper figure is inverted. These two steps are repeated alternately. In the figure (c) both the figures are inverted. 69. The triangle is rotating respectively 45°, 90°, 135°, 180°, 225°,.. in anticlockwise direction while the arc is inverted in the next figure. In figure (a) triangle has been rotated through 90° anticlockwise instead of 90° clockwise. 70. The line segments are added in the following manner: 0 1 1 2 2 3 3 4 4 5 5 6 Upper 1 1 0 1 1 1 Side Lower 1 2 1 1 2 0 Side Therefore, figure (d) is the wrong in the series. 71. In the subsequent figure respectively one, two, three… design (s) is/are inverted. 72. In the subsequent figure respectively two, three, two, four… line segments are deleted. 73. In each subsequent figure all the designs move half step in anticlockwise direction, two designs interchange positions and one of these two, designs is replaced with a new design. 74. The complete design is inverted in the next figure and one design is added in every figure. 75. From Problem Figure (a) to (b) the following changes occur:

• • •

• • •

• • •

• • •

• •

• • •

The information provided here is only for refrence. It may vary from the original.

www.recruitment.guru

Similar changes would occur from Problem Figure (5) to Answer Figure. 76. Let the unit’s digit be x and ten’s digit be y. Also let y > x Number = 10y + x And number obtained by interchanging the digits. = 10x + y :. 10y + x - 10x – y = 36 = 9y -9x =36 = 9(y –x) =36 = y-x =4 77. Required difference = 2 or 200 – 3 of 125 5 5 = 80- 75 = 5 78. Let the breadth of rectangular field be x metres.

:. Length = x x 115

100 = 23x metres 20 Now, Length x Breadth = Area

= 23x x x = 460

20

= x2 = 640 x

20

23

= x2 =20x 20

x √ 20 x 20 = 20metres

79. Let the listed price be Rs. x :. Discount = 30% of x = 30 x = Rs. 3x 100 10 According to the question, 3x = 82.5 10 = x = 82.5 x 10 = Rs. 275 3 :. Required cost price of calculator = 70% of 275 = Rs 70 x 275 = Rs .192.50 100 80. There are 8 letters in the word “SOFTWARE”, including 3 vowels (O, A, E) and 5 consonants ( S, F,T,W,R). Considering three vowels as one letter, we have six letters which can be arranged in 6P6 = 6! Ways . But corresponding, the vowels can be put together in 3! Ways. :. Required number of words = 6! x 3! = 4320 81. Initial speed of motor =71 kmph.

The information provided here is only for refrence. It may vary from the original.

www.recruitment.guru

Distance covered in first 2 hours = 2 x 70 = 140kms. For next two hours speed of motor = 80kmph :. Distance covered in first 4 hours = 140 + 160 =300 Remaining distance =345 – 300 = 45km This distance will be covered will be covered at the speed of 90 kmph. :. Time taken = 45 = 1 hour 90 2

:. Total time = 4 + ½ = 4 ½ hours 82. Total area of rectangular plot to be gardened = 340 x 1 + 4 x 1 = 344 sq. m :. Total cost = 344 x 10 = Rs. 3440 83. 4 men out of 7 men and 4 women out of 8 women can be chosen in 7C4 x 8C4 ways

=7 x 6 x 5 x 4 x 8 x 7x 6 x 5 1 x 2 x 3 x4 1 x 2 x 3 x4 = 35 x 70= 2450 84. Let the principal be Rs x and rate of interest be r% Case I:

x x r x 7 =1750 100 = xr = 1750 x 100 7 = Rs. 25000 Case II: S.I. = x x (r+2)x 7 100 Which cannot be determined with the help of given information. 85. Let the initial seats for Maths, Physics and Biology be 5x, 7 x and 8x respectively. Now, new seats for Maths, = 5x x 140 100

For Physics .= 7x x 150

100

And for Biology = 8x x 175 100 :. Required ratio = 5x x 140 : 7x x 150 : 8x x 175 100 100 100 = 5 x 140 : 7 x 150: 8 x 175

The information provided here is only for refrence. It may vary from the original.

www.recruitment.guru

= 2 : 3 : 4 87. Amount of fat in : 200 gms of A and 300 gms of C

= 200 x 28 + 300 x 13 100 100 = 56 + 39 = 95 gms 200 gms of B and 200 gms of A = 40 + 56 =96 gms 100 gms of C and 300 gms of A = 13 + 84 = 97 gms 300gms of C and 100gms each of A and B =39 + 28 +20 =87 gms 150gms each of A, B and C = 28 x 3 + 20 x 3 + 13 x 3 2 2 2 = 42 +30 + 19.5 =91.5 gms Hence option (c) is our answer. 88. A =y% of x = xy 100 B =y% of x = xy 100 :. A = B 89. Number of literate males =35% of 2,50,000 Number of literal females =33% of 2,50,000 :. Required difference = (35 -33)% of 2,50,000 = 2% of 2,50,000 = 5,000 90. We have, x – y = a - 1 a- 1 a – 1 = a – 1 =1

a - 1 Clearly x > y 91. (I) p2 - 7p = -12 = p2 – 4p +12 = 0 = p2 -4p 3p + 12=0 = p(p-4) -3(p-4)=0 = (p -4) (p -3) =0 = p=3 or 4 (II) q2 -3q + 2=0 = q2 -2q –q +2=0 = q(q-2) -1(q -2) =0 = (q-2) (q-1)=0 = q=1 or 2

The information provided here is only for refrence. It may vary from the original.

www.recruitment.guru

Obviously p >q 92. (I) 12p2 - 7p = -1 = 12p2 - 7p + 1 = 0 = 12p2 – 4p - 3p + 1 = 0 = 4p(3p – 1) – 1 (3p -1) =0 = (3p -1) (4p -1) =0 = p = 1 or 1 4 3 (II) 6q2 -7q + 2 = 0 = 6q2 - 4q – 3q + 2 =0 = 2q(3q -2) – 1 (3q -2)=0 = (3q -2) (2q -1) =0 = q = 2 or 1 3 2 Obviously p < q 93. (I) p2 + 12p + 35 = 0 = p2 + 7p + 5p + 35 =0 = p (p +7) + 5(p + 7) =0 = p(p+7) +5 (p + 7)=0 = p(p +7) (p + 5) =0 = p = -5 or -7 (II) 2q2 + 22q + 56 = 0 = 2q2 + 14q + 8q + 56 = 0 = 2q(q + 7) +8(q +7) =0 = (q + 7) (2q +8) =0 = q = - 7 or - 4 Obviously p < q 94. (I) p2 + 8p + 15 = 0 = p2 – 3p - 5p + 15 =0 = p(p -3) -5 (p-3) =0 = (p -3) (p -5) =0 = p = 3 or 5 (II) q2 – 5q + 6q +6=0 = q2 - 3q – 5q +6 =0 = q(q – 3) -2(q -3) =0 = q(q-3) (q – 2) =0 = q = 3 or 2 Obviously p < q 95. (I) 2p2 + 20p + 50 = 0 = p2 + 10p + 25 =0 = (p +5)2 =0 = p +5 =0 = p = -5 (II) q2 = 25 = q = + 5 Obviously p < q 96. Production of Company A In 2000 =4 lakh units In 2001 = 3 lakh units Decrease = 4 -3 = 1 lakh units :. % decrease = 1 x 100 = 25%

The information provided here is only for refrence. It may vary from the original.

www.recruitment.guru

4 97. Production of Company B In 2006 = 7 lakh units In 2007 = 11 lakh units :. Required percentage = 11 x 100 7 =157.14 =157% 98. It is obvious from the graph. 99. Total production of Company A =(4 + 3 + 8 + 8 + 8 + 7 + 8 + 12) Lakh units =5800000 Total production of Company B = (1 + 1 + 1 + 2 + 3 + 5 + 7 +11) =31 lakh units = 3100000 :. Required difference = (58-31) x100000 =2700000 100. Required average production of Company B = 31 lakh units 8 =3.875= 3.9llakh units 101. For Course D Number of girls = 30% of 800 = 30 x 800 = 240 100 Number of students = 35 x 1 x 1200 100 = 420 :. Number of boy = 420 – 240 = 180 :. Required ratio = 180 : 240 = 3 : 4 102. Number of boys In Couse E = 12% of 1200 – 14% of 800 = 144 – 112 = 32 In course F = 13% of 1200 – 14% of 800 156 – 112 = 44 In course A = 20% of 1200 – 30% of 800 = 240 – 240 =0 In course D = 35% of 1200 – 30% of 800 = 420 – 240 =180 In course C = 5% of 1200 -2% of 800 = 60 – 16 = 44 Obviously pair C and F is our answer. 103. For Course E, Number of girls = 14% of 800 = 112 Number of boys = 32 :. Required percentage

The information provided here is only for refrence. It may vary from the original.

www.recruitment.guru

= 80 x 100 = 250 32 105. Number of girls in course C = 2% of 800 = 16 106. The given series is based on following pattern 15 – 10 = 5 24 – 15 = 9 37 – 24 = 13 54 – 37 = 17 75 – 57 = 21 100- 75 = 25 Obviously 35 is wrong number 107. Here the middle number = difference of succeeding number and preceding number i.e. 4 – 1 = 3 7 – 3 = 4 11- 4 = 7 18 – 7 =11 27– 11 = 16 Here the sequence gets disturbed 29 - 11 = 18 47- 18 =29 Hence 27 is the wrong number 108. The sequence is based on following pattern 3 x 0.5 + 0.5 =2 2 x 1 +1 = 3 3 x 1.5 + 1.5 = 6 6 x 2 +2 =14 14 x 2.5 + 2.5= 37.5 37.5 x 3 + 3 =37.5 109. 32431 = 7 x 4626 + 72

4626 = 6 x 765 + 62

765 = 5 x 148 + 52

= 148141 x 32 + 42

But 148 = 4 x 33 + 42

33 = 3 x + 32

8 = 2 x 2 +22

Obviously 32 is the wrong number. 110. The sequence is based on following pattern: 3 – 2 = 13

11 – 3 = 8 = 23

38 – 11 = 27 = 33

102 – 38 = 64 = 43

But 229 – 102 = 127 = 53

227 – 102 = 125 = 53

442 – 227 = 125 = 63

obviously 229 is the wrong number. 111. S.P. of the product = Rs. 100 From statement I C. P. = 100 x 90= Rs. 75 120 :. Actual profit = 100 -75 = Rs. 25

The information provided here is only for refrence. It may vary from the original.

www.recruitment.guru

From statement II Let the C.P. be Rs.x Then profit = x 3 :. x + x = 100 3 = 4x = 300 = x =Rs. 75 :. Profit = Rs. 25 112. Time taken in seconds to cross = Sum of length of both trains in metres Relative speed in m/sec. x + 0. 0y + z 24 Where z m is the length of other train To know u,z is required. 113. Let the digits at unit’s and ten’s places be x and y respectively. When x > y then x – y = 6 Then x –y = 6 And x + y = 12 Gives x = 9 And y = 3 Again, where y > x y- x = 6 Hence y – x = 6 And x + y = 12 Gives y = 9 And x = 3 Hence, the numbers are 93 or 39 Obviously when statement I is not taken into consideration, two numbers are possible. 114. X = odd number. It is possible 5 5 Only when X is odd From statement II X + Y = an odd number But, X = 2 and Y =5 gives X + Y =7, an odd number Hence only statement I is sufficient to answer the question. 115. From statement I Capacity of tank = p x 14 x14 x 28 = 17248 m3 From Statement II. Capacity of tank = 616 x 28 = 17248 m3 Obviously, either statement I or statement II is required. 116. Marks obtained by Meera in: English = 60 History = 40 x 80 = 32 100 Computers = 50 x 130 = 65 100 Maths = 90 x 150 = 135 100 Science = 90 x 120 = 108 100 Economics = 60 x 80 = 48

The information provided here is only for refrence. It may vary from the original.

www.recruitment.guru

100 :. Total marks = 60 +32 +65 + 135 +108 + 48 = 448 117. Marks obtained in History by: Meera = 40 x 80 =32 100 Subodh = 70 x40 =28 100 Kunal = 70 x40 =28 100 Soni = 60 x 40 100 Richu = 90 x 40 = 36 100 Irene = 60 x 40 = 24 100 Vijay = 80 x40 = 32 100 Required average = 32 + 28 + 28 + 24 + 36 + 24 +32 7 = 204 = 29.14 7 118. Total marks obtained by Kunal = 54 + 28 + 78+ 135+ 84 + 56 = 435 Total marks = 60 + 40 +130 +150 +120 +80 = 580 :. Required percentage = 435 x 100 = 75 580 121. 95? = 953.7 ÷ 951.0

= 95? = 953.7 – 1 =952.7

= ? = 2.7

122. ? = √1000 + 3 x 1892

5 = 100 + 1135.2 = 1235.2 = 1230 123. = 0.0004 x 36 0.0001 = 4 x 36 = 144 = 145 124. ? = 140% of 12300 = 140 x 12300 100 = 17220 = 17000 125. ? = 3739 + 160 x 30 = 3739 + 4800 = 8539 = 8200 136. (A) Proximate – nearest or next, without anything between a cause and it effect.

The information provided here is only for refrence. It may vary from the original.

www.recruitment.guru

(B) Elevated –dignified, elated, grand, high lofty, noble, raised, sublime (C) Nimble – active, agile, alert, brisk, deft, light-footed, lively, ready. (D) Agile-active, adroit, brisk, clever, quick, sharp, swift, spry Hence C-D bear same meaning. 137. (A) Notion-apprehension, belief, concept, fancy, image, judgement, knowledge (B) Symbol-badge, emblem, image, logo, representation sign, token, type (C) Concept-abstraction conception, construct, idea, image, type, view (D) Massage-rubbing, kneading, manipulation, rub-down Hence, B_D bear same meaning 138. (A) Limpid-bright, clear, comprehensible, crystal-clear, glassy, intelligible, pure (B) Luscious-appetising, delicious, desirable, juicy, savoury, sweet, tasty (C) Acrid –acid, acrimonious, bitter, sharp, burning, pungent, sarcastic (D) Benign-amiable, benevolent, friendly, genial, gentle, good, gracious Hence, B-C bear opposite meanings 139(A) Asinine-of or like an ass, idiotic (B) Furious + angry, boiling, enraged, fierce, fuming, givid, mad, raling, violent, stormy (C) Fortunate Advantageous, blessed, bright, luck, happy, successful, felicitous, opportunate (D) ridiculous absurd, comical, derisory, farcical, foolish, funny, stupid, ludicrous Hence , A-D bear same meaning 140. (A) Companion – accomplice, aide, ally assistant, associate fellow, mate, partner (B) Amateur-buff, dabbler, dilettante, fanier, ham, layman, non-professional (C) Adept- able, accomplished, adroit versed, skilled, proficient, practised nimble. (D) Adgerent-admirer, advocate, devotee, disciple, fan, follower, supporter Hence, A-D bear same meanings. 141. (A) Squander- blow, consume, dissipate, expend, fritter, waste, scatter, misuse (B) Disunite-to separate from union, detach, divide, to severe or surrender (C) Flicker-flare, flash, flutter, waver, vibrate, twinkle, glimmer, gutter (D) Preserve care for, conserve, continue, defend, guard, keep, uphold, hitter Hence, A-D bear opposite meanings 142. (A) Mitigate-to mollify, oppease, to make more easily born, to temper (B) Acquiesce-accede, accept, agree, allow, approve, assent, comply, concur (C) Relinquish-abandon, cede desert, discard, forgo, resign, vacate (D) Duplicate-corresponding, identical, method, twofold, twin Hence, B-C bear opposite meanings 144. (A) Occurrence-aution, adventure, affair, instance, incident, episode (B) Pretence-acting, affection, aim, allegation, appearance, charade, grab (C) Profusions-abundance, copiousness, glut, multitude, wealth, extravagance (D) Extravagance-abundance, excess, folly, squandering, waste, profusion Hence, C-D bear same meaning 145. (A) Excellent-admirable, commendable, unequalled, superb, wonderful, worthy (B) Passionate-ardent, erotic, stormy, violent, wild, zealous, fiery, inflamed, fervent (C) Apathetic-Cold, Cool, impassive, passive, unemotional, indifferent (D) Discrepant-contrary, disagreeing Hence, B-C bear opposite meanings. (146-150) First Sentence –(F) I will answer all your queries a little elaborately. Second Sentence-(C) Firstly, there was no tradition of theatre in my family

Third Sentence-(G) Let alone theatre, arts in general had none place of respect in my family.

Fourth Sentence-(E) My parents were old fashioned. Fifth Sentence-(B) My father was a government servant.

The information provided here is only for refrence. It may vary from the original.

www.recruitment.guru

Sixth Sentence-(B) He never wanted me to be an actor, as he didn’t took upon theatre or acting as respectable vocation.

Seventh Sentence-(A) But I always felt somewhere in my mind that I loved acting. Eighth Sentence-(D) I am talking specifically of acting not theatre in general. 151. The group of words, ”sea water for instance, contains a lot of salt, fresh water contains very

little” should be replaced by ‘sea water for instance, contains a lot of salt fresh water contains a very little’

Look at the sentences: He has little milk to give you (x) He has a little milk to give you (√)

152. The group of words, a sparrow has made a nest in Kesho’s house’ should be replaced by ‘a sparrow had made nest in Kesho’s house’.

Look at the sentences : Heena said that he is wrong (x) Heena said that he was wrong (√) 153. The group of words, ‘monopoly for himself should be replaced by ‘monopoly of himself. 154.The group of words, ‘we can be certain that man did not began’ should be replaced by ‘man

did not begin’ Because Do/Does/Did takes First Form (V1). Look at the sentences: He do goes to market. (x) He do go to market. (√) Or, he goes to market.( √) He did not went to market. (x) V2 V2

He did not go to market. (√) 161. The group of words ‘was Stranding’ should be replaced by ‘had been stranded’. The word

strand means- to cause something to be along, to make a thing or person to be deserted. Look at the sentence: The dead body of the man who hanged active was found. (x) The dead body of the man who was hanged Passive Was found. (√) 162. The group of words, ‘I receive the letter 1 should be replaced by ‘I received the letter’. The

principal clause of the sentence is in Past Tense, hence the sub ordinate clause should also be used in Past Tense.

Look at the sentence: Yesterday when I go there, he had gone Present tense Past tense Out. (x) Yesterday when I went there, he had gone Past tense Past tense Out.(√) 164. The group of words, ‘to the committee’ should be replaced by of the committee. To express possession between two Nouns we use-Noun + of + Noun structure.

The information provided here is only for refrence. It may vary from the original.

www.recruitment.guru

Look at the sentence: The colour to this pen is green(x) Noun possessive noun The colour this pen is green(x) Noun possessive noun

www.gradeup.co

2

1. Recently, RBI revised PCA framework for

banks. Here PCA stands for-

A. Prompt Clear Action

B. Primary Corrective Action

C. Primary Corporate action

D. Prompt Corrective Action

E. None of the above

2. Which bank has launched ‘Mingle’?

A. SBI B. ICICI

C. PNB D. Axis

E. BOB

3. RBI has said in a notification that all existing

asset reconstructions companies (ARC) must

have minimum net owned corpus of how

much amount by March 2019?

A. Rs. 500 crores B. Rs. 100 crores

C. Rs. 200 crores D. Rs. 400 crores

E. None of these

4. In the year 2017, the Indian government has

revised base year of which of the following

indices?

A. CPI B. WPI

C. IIP D. b and c only

E. All the above

5. How much fiscal deficit target in terms of GDP

percent, fixed by government for financial

year 2018?

A. 4.2 Percent B. 3.7 percent

C. 3.2 Percent D. 2.9 Percent

E. 2.7 percent

6. Insolvency and Bankruptcy Board of India

(IBBI) constituted Technical Committee Under

_______ to give recommendations for laying

down Technical Standards for performance of

core services and other services under IBBI

(Information Utilities) Regulations 2017.

A. Dinesh Sharma B. Asim Dasgupta

C. R B Barman D. Sushil Mitra

E. None of the above

7. Recently IMF revised India’s GDP growth rate

forecast from earlier estimated 7.2 Percent to

______.

A. 7 Percent B. 6.9 Percent

C. 6.8 Percent D. 6.7 Percent

E. 6.5 Percent

8. Which of the following country hosted the

14th Edition of United Nations International

Day of Vesak 2017?

A. Greece B. France

C. Belgium D. Sri Lanka

E. None of these

9. Which of the following country has topped in

the World Economic Freedom (WEF) Index

2016?

A. Belgium B. Algeria

C. Hong Kong D. Switzerland

E. None of these

10. Reserve Bank of India (RBI) tightened the

rules around making the JLF more effective,

directing banks not to break any rules and to

meet all deadlines. Here JLF stands for-

A. Joint Lending Firms

B. Joint Lending Forum

C. Joint Lender’s Forum

D. Joint liability Forum

E. Joint litigation Forum

11. US-based think tank GFI in its report titled

‘Illicit Financial Flows to and from Developing

Countries: 2005-2014’, has estimated that

$770 billion worth of black money entered

India during 2005-2014. Here GFI stands for-

A. Global Financial Institution

B. Global Financial Investigator

C. General Financial Institution

D. Global Financial Integrity

E. Global Fair Institution

12. Working capital means

A. Current assets – current liabilities

B. Fixed assets – current assets

C. Fixed assets – fixed liabilities

D. Fixed liabilities – current liabilities

13. What do we call the risk of collapse of an

entire financial system or entire market?

A. Seismic Risk

B. Systemic Risk

C. Systematic Risk

D. Market Risk

E. Economic Risk

14. The Union Budget 2017-18 has allocated Rs.

__________ crore to the flagship MGNREGA

program?

A. 48,000 B. 44,000

C. 55,000 D. 42,000

E. 58,000

15. Reverse Repo Rate is a tool used by RBI to

A. Inject Liquidity

B. Absorb Liquidity

C. Increase the liquidity with banking system

D. To keep the liquidity at one level

E. None of the above

www.gradeup.co

3

16. What is a ante-dated cheque?

A. A cheque issued without drawer's signature

B. A cheque with only signature of the drawer

C. A cheque which has completed three

months from its date of issue

D. A cheque which has completed six months

from its date of issue

E. Cheque is written by drawer and dated at

some date in past but not expired

17. Which bank has launched 'Aadhaar Payment

Bridge System' (APBS) for small ticket micro-

finance loan disbursements?

A. SBI B. ICICI Bank

C. HDFC D. Federal Bank

E. RBL Bank

18. _______ is the rate at which banks borrow

funds overnight from the Reserve Bank of

India (RBI) against approved government

securities.

A. CRR B. SLR

C. Call Money D. MSF

E. Repo Rate

19. The National Financial Switch (NFS) facilitates

routing of ATM transactions through inter-

connectivity between the Bank’s Switches,

thereby enabling the citizens of the country to

utilize any ATM of a connected bank. The

National Financial Switch (NFS) is run by ___.

A. National Payments Corporation of India

(NPCI)

B. Industrial Finance Corporation of India

(IFCI)

C. Institute for Development and Research in

Banking Technology (IDRBT)

D. Reserve Bank of India (RBI)

E. None of these

20. Name the world’s highest wicket-taker in

women ODI cricket?

A. Jhualan Goswami B. Mithila Raj

C. Poonam Raut D. Anjum Chopra

E. Veda Krishnamoorthy

21. Who among the following has been elected as

the chairman of ‘FIFA governance committee’

at the world football governing body's?

A. Altamas Kabir B. R C Lahoti

C. Mukul Mudgal D. P Sathasivam

E. None of these

22. Non-Banking Financial Company (NBFC) are

allowed to accept/renew public deposits for a

maximum period of ___.

A. 12 months B. 24 months

C. 48 months D. 60 months

E. None of these

23. Consider the following statements regarding

Payment Banks:

A) Payment Banks can't advance loans or

issue credit cards.

B) Payment Banks can accept demand

deposits up to Rs 5 lakh.

C) Payment Banks offer remittance services,

mobile payments/transfers/purchases.

D) Payment Banks offer banking services like

ATM/debit cards, net banking and third party

fund transfers.

Which of the above options are correct:

A. A, C and D only B. B, C and D only

C. A and D only D. All of the above

E. Only A & B

24. Recently NITI Aayog has constituted an

‘Expert Task Force’ to provide a major thrust

to job creation by enhancing India’s exports.

Who will head the task force?

A. Dr. Rajesh Kumar

B. Rahul Ranjan Sinha

C. Dr. Rajiv Kumar

D. R. B. Barman

E. None of these

25. Unified Payments Interface (UPI) launched by

the National Payments Corporation of India

(NPCI) would do the following function-

1) It would eliminate the need to exchange

sensitive information about bank account

numbers during a financial transaction.

2) It allows sharing of a bill among peers.

3) It allows individual to even request the

money from its debtors.

Select the correct answer-

A. 1 and 2 only B. 2 and 3 only

C. 1 only D. 1, 2 and 3

E. 3 only

26. Kavinder Singh is related to which game?

A. Boxing B. Archer

C. Hockey D. Shooting

E. Wrestling

27. The base year for calculating key economic

growth from 2004-05 changed to?

A. 2010-11 B. 2011-12

C. 2008-09 D. 2007-08

E. 2005-06

www.gradeup.co

4

28. After the merger of five associates Banks in

State Bank of India (SBI), now the total

number of ATMs reached across the country is

_____.

A. 60,000 ATMs B. 69,000 ATMs

C. 59,000 ATMs D. 72,000 ATMs

E. 45,000 ATMs

29. The target of digital transactions through UPI,

USSD, Aadhar, IMPS for 2017-18 is ____.

A. 22,00 crore B. 1600 crore

C. 2000 crore D. 2,500 crore

E. 19,00 crore

30. Finland has become the first country in

Europe to pay its unemployed citizens a basic

monthly income, amounting to___.

A. 226 euros B. 324 euros

C. 560 euros D. 421 euros

E. 611 euros

31. Government has approved Rs 681 crore as

seed capital for building a total corpus of ____

under the electronic development fund meant

to support entrepreneurship and innovation in

electronics and IT.

A. Rs 5,125 crores B. Rs 6,355 crores

C. Rs 5,732 crores D. Rs 6,831 crores

E. Rs 7,233 crores

32. Government of India has changed Cost

Inflation Index (CII) base year from 1981 to

___.

A. 2000 B. 2002

C. 2001 D. 2005

E. 2004

33. Institute for Development & Research in

Banking Technology (IDRBT) is in?

A. Hyderabad B. Bengaluru

C. Chennai D. Mumbai

E. New Delhi

34. Amendment to Section _______ of Negotiable

Instruments Acts to include the electronic

image of a truncated cheque and a cheque in

electronic form.

A. 7 B. 8

C. 2 D. 5

E. 6

35. The target for agricultural credit in 2017-18

has been fixed at a record level of _____.

A. Rs. 5 lakh crores B. Rs. 10 lakh crores

C. Rs. 20 lakh crores D. Rs. 25 lakh crores

E. Rs. 15 lakh crores

36. A class of derivative whose value is at least

partly derived from one or more underlying

stock exchange securities? (Trade,

Commodity, Credit, Interest Rate, Equity)

A. Derivative

B. Stock Derivative

C. Future Derivative

D. Equity Derivative

E. Other than the given options

37. The government has decided to make tax

evasion of over ____________ under the

proposed Goods and Services Tax (GST) the

regime as a non-bailable offence.

A. Rs 6 crore B. Rs 3 crore

C. Rs 5 crore D. Rs 8 crore

E. Rs 10 crore

38. As per report of Stockholm International

Peace Research Institute (SIPRI) India’s

military expenditure in 2016 grew by-

A. 7 per cent B. 8.5 per cent

C. 8 per cent D. 7.5 per cent

E. 6.5 per cent

39. India Licensing Expo (ILE), India's first and

most influential brand licensing show will take

place on 20 August 2017 in ___.

A. Hyderabad B. Bengaluru

C. Chennai D. Mumbai

E. New Delhi

40. Foreign direct investment (FDI) in India grew

18 per cent during 2016 to touch __________

data released by the Department of Industrial

Policy and Promotion (DIPP).

A. $42 billion B. $46 billion

C. $50 billion D. $39 billion

E. $49 billion

Direction (41-45): Read the following

passage carefully and answer the questions

that follow.

At first sight, it looks as though Panchayati

raj, the lower layer of federalism in our polity,

is as firmly entrenched in our system as is the

older and higher layer comprising the Union

Government and the States. Like the

democratic institutions at the higher level,

those at the panchayat level, the Panchayati

raj institutions (PRIs), are written into and

protected by the Constitution. All the essential

features, which distinguish a unitary system

from a federal one, are as much enshrined at

the lower as at the upper level of our federal

system. But look closely and you will discover

a fatal flaw. The letter of the Constitution as

www.gradeup.co

5

well as the spirit of the present polity have

exposed the intra-State level of our federal

system to a dilemma of which the inter-State

and Union-State layers are free. The flaw has

many causes. But all of them are rooted in an

historical anomaly, that while the dynamics of

federalism and democracy have given added

strength to the rights given to the States in

the Constitution, they have worked against

the rights of panchayats.

At both levels of our federal system there is

the same tussle between those who have

certain rights and those who try to encroach

upon them if they believe they can. Thus, the

Union Government was able to encroach upon

certain rights given to the States by the

Constitution. It got away with that because

the single dominant party system, which

characterised Centre-State relations for close

upon two decades, gave the party in power at

the Union level many extra-constitutional

political levers. Second, the Supreme Court

had not yet begun to extend the limits of its

power. But all that has changed in recent

times. The spurt given to a multi-party

democracy by the overthrow of the

Emergency in 1977 became a long-term trend

later on because of the ways in which a

vigorously democratic multi-party system

works in a political society which is as

assertively pluralistic as Indian society is. It

gives political clout to all the various

segments which constitute that society.

Secondly, because of the linguistic

reorganisation of States in the 1950s, many

of the most assertive segments have found

their most assertive expression as States.

Thirdly, with single-party dominance

becoming a thing of the past at the Union

level, governments can be formed at that

level only by multi-party coalitions in which

State-level parties are major players. This has

made it impossible for the Union Government

to do much about anything unless it also

carries a sufficient number of State-level

parties with it. Indian federalism is now more

real than it used to be, but an unfortunate

side-effect is that India's Panchayati raj

system, inaugurated with such fanfare in the

early 1980s, has become less real.

By the time the PRIs came on the scene, most

of the political space in our federal system

had been occupied by the Centre in the first

30 years of Independence, and most of what

was still left after that was occupied by the

States in the next 20. PRIs might have hoped

to wrest some space from their immediate

neighbour, the States, just as the States had

wrested some from the Centre. But having at

last managed to checkmate the Centre's

encroachments on their rights, the States

were not about to allow the PRIs to do some

encroaching of their own.

By the 1980s and early 1990s, the only

national party left, the Congress, had gone

deeper into a siege mentality. Finding itself

surrounded by State-level parties, it had built

walls against them instead of winning them

over. Next, the States retaliated by blocking

Congress proposals for Panchayati raj in

Parliament, suspecting that the Centre would

try to use panchayats to bypass State

Governments. The suspicion fed on the fact

that the powers proposed by the Congress for

panchayats were very similar to many of the

more lucrative powers of State Governments.

State-level leaders also feared, perhaps, that

if panchayat-level leaders captured some of

the larger PRIs, such as district-level

panchayats, they would exert pressure on

State-level leaders through intra-State multi-

party federalism.

It soon became obvious to Congress leaders

that there was no way the Panchayati raj

amendments they wanted to write into the

Constitution would pass muster unless State-

level parties were given their pound of flesh.

The amendments were allowed only after it

was agreed that the powers of panchayats

could be listed in the Constitution.

Illustratively, they would be defined and

endowed on PRIs by the State Legislature

acting at its discretion.

This left the door wide open for the States to

exert the power of the new political fact that

while the Union and State Governments could

afford to ignore panchayats as long as the

MLAs were happy, the Union Government had

to be sensitive to the demands of State-level

parties. This has given State-level actors

strong beachheads on the shores of both

inter-State and intra-State federalism. By

using various administrative devices and non-

www.gradeup.co

6

elected parallel structures, State

Governments have subordinated their PRIs to

the State administration and given the upper

hand to State Government officials against

the elected heads of PRIs. Panchayats have

become local agencies for implementing

schemes drawn up in distant State capitals.

And their own volition has been further

circumscribed by a plethora of “Centrally-

sponsored schemes”. These are drawn up by

even more distant Central authorities but at

the same time tie up local staff and resources

on pain of the schemes being switched off in

the absence of matching local contribution.

The "foreign aid" syndrome can be clearly

seen at work behind this kind of "grass roots

development".

41. The central theme of the passage can be best

summarized as

A. Our grassroots development at the

panchayat level is now driven by the "foreign

aid" syndrome.

B. Panchayati raj is firmly entrenched at the

lower level of our federal system of

governance.

C. A truly federal polity has not developed

since PRIs have not been allowed the

necessary political space.

D. The Union government and State-level

parties are engaged in a struggle for the

protection of their respective rights.

E. None of these

42. The sentence in the last paragraph, “And their

own volition has been further

circumscribed...”, refers to:

A. The weakening of the local institutions'

ability to plan according to their needs.

B. The increasing demands made on elected

local leaders to match central grants with

local contributions.

C. The empowering of the panchayat system

as implementers of schemes from State

capitals.

D. The process by which the prescribed

Central schemes are reformulated by local

elected leaders.

E. None of these

43. What is the "dilemma" at the intra-State level

mentioned in the first paragraph of the

passage?

A. Should the state governments wrest more

space from the Union, before considering the

Panchayati system?

B. Should rights similar to those that the

States managed to get be extended to

panchayats as well?

C. Should the single party system which has

withered away be brought back at the level of

the States?

D. Should the States get "their pound of

flesh" before allowing the Union government

to pass any more laws?

E. None of these

44. Which of the following most closely describes

the 'fatal flaw' that the passage refers to?

A. The ways in which the democratic multi-

party system works in an assertively

pluralistic society like India's are flawed.

B. The mechanisms that our federal system

uses at the Union government level to deal

with States are imperfect.

C. The instruments that have ensured

federalism at one level, have been used to

achieve the opposite at another.

D. The Indian Constitution and the spirit of

the Indian polity are fatally flawed.

E. None of these

45. Which of the following best captures the

current state of Indian federalism as

described in the passage?

A. The Supreme Court has not begun to

extend the limits of its power.

B. The multi-party system has replaced the

single party system.

C. The Union, state and Panchayati raj levels

have become real.

D. There is real distribution of power between

the Union and State level parties.

E. None of these

Direction (46-50): Read the given passage

carefully and answer the questions that

follow.

The world is leaning on its biggest economy to

sustain the global recovery, according to the

International Monetary Fund. The fund left its

forecast for global growth unchanged in the

latest quarterly update to its World Economic

Outlook, released Monday in Kuala Lumpur.

The world economy will expand 3.4 percent

www.gradeup.co

7

this year, up from 3.2 percent in 2016, and by

3.6 percent next year, the IMF said. The

forecasts for this year and next are

unchanged from the fund’s projections in

April. Beneath the headline figures, though,

the drivers of the recovery are shifting, with

the world relying less than expected on the

US and the UK and more on China, Japan, the

euro zone and Canada, according to the

Washington-based IMF.

The dollar fell to its lowest in 14 months last

week as investors discounted the ability of

President Donald Trump’s administration to

deliver on its economic agenda after efforts by

the Republican Senate to overhaul healthcare

collapsed. The IMF estimated US growth at

2.1 percent this year and again in 2018,

consistent with what the fund said June 27 in

its annual assessment of the US 2.3 percent

and 2.5 percent, respectively, in 2017 and

2018. The economy expanded by 1.6 percent

in 2016. “US growth projections are lower

than in April, primarily reflecting the

assumption that fiscal policy will be less

expansionary going forward than previously

anticipated,” the IMF said in the latest report.

In June, the IMF said it had dropped

assumptions of a boost to growth from

Trump’s plans to cut taxes and increase

infrastructure spending. Trump’s budget

director, Mick Mulvaney, wrote in July that the

administration’s goal is “sustained 3 percent

economic growth,” and he named the

program “MAGAnomies” after Trump’s

campaign slogan, “Make America Great

Again”.

Meanwhile, as the UK works through its Brexit

negotiations, the IMF also chopped its

forecast for UK growth this year by 0.3

percentage point to 1.7 percent on weaker-

than-expected activity in the first quarter.

“This forecast underscores exactly why our

plans to increase productivity and ensure we

get the very best deal with the EU, are vitally

important,” the UK Treasury said in an

emailed statement. “The fundamentals of our

economy are strong.”

Other countries are picking up the slack. The

IMF’s projection for growth in China is 6.7

percent for 2017 – the same as its estimate

made June 14 in an annual staff report, and

up 0.1 point from April’s world economic

outlook. For 2018 the fund sees Chinese

growth at 6.4 percent, an increase of 0.2

points from three months ago. In the report,

the IMF looked for average annual growth of

6.4 percent in China during 2018 through

2020. “Rich market valuations and very low

volatility in an environment of high policy

uncertainty raise the likelihood of a market

correction, which could dampen growth and

confidence, said the fund, which also cited

China’s credit growth and protectionist

policies as threats.

While risks to the global outlook are “broadly

balanced” in the near term, medium term

risks are titled to the downside, the IMF said.

IMF urged advanced countries with weak

demand and low inflation to continue

supporting growth through monetary and

fiscal policy while cautioning central banks

against raising borrowing costs too quickly.

The fund said widespread protectionism or a

“race to the bottom” on financial and

regulatory oversight would leave all countries

worse off.

46. Which of the following statements can

definitely be concluded from the given

passage?

(i) IMF does not perceive any considerable

long term risk to global market in the short

term

(ii) IMF views China’s credit and growth policy

as a risk to global outlook

(iii) The US can perform well in global outlook

by abandoning its protectionist policies

A. Both (i) and (ii)

B. Both (i) and (iii)

C. Only (i)

D. Only (ii)

E. Only (iii)

47. Which of the following is TRUE in the context

of IMF’s growth forecast of China?

(i) An annual average growth of 6.4 percent

during 2018 through 2020

(ii) No change in its forecast for 2017 from its

Annual Staff report of June 14th

(iii) The forecast for 2018 has grown a

meager 0.2 percent from previous quarter

A. Only (ii) and (iii)

B. Only (i) and (ii)

C. Only (i) and (iii)

D. None of these

E. All of the above

www.gradeup.co

8

48. IMF has cut growth forecast of UK by 0.3

percent. What does it imply for the UK?

A. It should increase infrastructure spending

and cut taxes

B. It made a mistake to withdraw itself from

Europe

C. It should work on its fundamental for faster

economic recovery

D. It should ensure best deal out of Brexit

negotiations

E. None of these

49. According to the IMF, the advanced countries

with weak demand and low inflation should-

(i) discard their protectionist policies

(ii) support growth through monetary and

fiscal policy

(iii) keep a check on rapid increase in rate of

interest by central banks

A. Both (i) and (ii) B. Both (ii) and (iii)

C. Both (i) and (iii) D. Only (i)

E. Only (ii)

50. Which of the following factors can be

attributed to the fall of dollar to its lowest in

the 14 weeks period?

A. The US’ diminishing role in global recovery

B. The emergence of Japan, China and Euro

Zone as the driver of global recovery

C. The protectionist policies of Trump

D. The inability of Trump to keep his

economic promises

E. Less expansionary fiscal policy of the US

than expected

Direction (51-54): In the given question,

two sentences with two blanks in each are

followed by five options with two words in

each. Select that option as your answer which

can fill both the blanks of both the sentences I

and II.

51. I. When deciding the term of the loan, base

your _____ on the loan’s total cost and not

_____ payments.

II. The ______ to pay the workers on a

monthly basis is very beneficial for small firms

to keep a check on their ______ profit.

A. Monthly, advance

B. Decision, monthly

C. Determine, yearly

D. Mention, pay

E. None of these

52. I. Cyber-crime is at ______ proportions and

the chance that your website, your name and

your business can be _______ affected are

more of a reality than ever before.

II. An/a _______ of smallpox resulted in the

death of a substantial portion of the native

population of this country and has _______

affected the population of the nation.

A. Epidemic, severely

B. Severely, destroyed

C. Splutter, severely

D. Sagacious, Juxtaposed

E. None of these

53. I. Wood windows will require maintenance

and re-painting to ______ appearance and

______ of the wood against rot and insects.

II. Even though they were tired mentally and

physically, the soldiers continued to ______

their duty to serve their country with _______

and honour.

A. Lurk, germane B. Maintain, Integrity

C. Garnish, Integrity D. Profess, Heft

E. None of these

54. I. The Black Student _______ is holding a

cultural _____ workshop in the media centre

on Saturday.

II. We realized that it was a _____ of people

who were against religious ____ in the

society.

A. Jocular, Adept B. Convulsion, Niggle

C. Huff, Chasm D. Caucus, diversity

E. None of these

Direction (55-58): A sentence is divided into

three parts I, II and III. For each part a

correction statement is given. Determine the

part which requires correction and mark it as

your answer.

55. Rapid economic ascendance has brought

many challenges, / including high inequality;

rapid urbanization; challenges of, /

environmental sustainability; and external

imbalances.

I. Rapid economic ascendance has brought on

many challenges

II. high inequality; rapid urbanization;

challenges to

III. environmental sustainability; and external

imbalance

A. Only III B. Both I and II

C. Both I and III D. Only II

E. Only I

www.gradeup.co

9

56. The academic literature provides mixed

findings, / for the relationship among

immigration and crime worldwide, / but finds

for the United States that immigration has no

impact on the crime rate.

I. The academic literature provided mixed

findings

II. for the relationship between immigration

and crime worldwide,

III. but finds for the United States that

immigration is in no impact on the crime rate.

A. Only II B. Only III

C. Both I and II D. Both II and III

E. Both I and III

57. Fascism has many faces, but the most

constant is that the supremacist delusion that

the West is the carrier of “universal

values”/and that, as exclusive interpreter and

custodian of these values; /the West is

obligated to act upon watchdog of democracy

and human rights throughout the globe.

I. Fascism has many faces, but the most

constant is that of the supremacist delusion

that the West is the carrier of “universal

values”

II. and that, as an exclusive interpreter and

curator of these values;

III. the West is obligated to act as a watchdog

of democracy and human rights throughout

the globe

A. Both I and II B. Both II and III

C. All I, II and III D. Only I

E. Both I and III

58. Several times we sat with the new people and

tried to put together agreements / that would

accommodate the needs of both of our

peoples, / each time it was us who gave

privileges to the new people.

I. Several times we sat with the new people

and tried to put forward agreements

II. That would accommodate the needs for

both of our peoples,

III. each time it was we who gave privileges

to the new people.

A. Only II

B. Only III

C. Both II and III

D. Both I and II

E. None of these.

Direction (59-61): In the given

question, a statement with one blank,

followed by three alternatives, is given. Out of

the given three alternatives, more than one

can fill the given blank. Choose the set of

words from the given options which can be

used to fill the blank.

59. Despite having a border dispute India and

China have ______ thereby creating a

‘managed relationship’.

i. Never exchanged a bullet on the border in

the last 40 years

ii. Preserved accord

iii. Not been belligerent towards each other

A. Only i B. Only ii

C. Both i and iii D. Both ii and iii

E. All the above

60. The need for rural communities to approach

development from a wider perspective has

______ rather than merely creating incentive

for agricultural or resource-based businesses.

i. Made things difficult and more complex

ii. Been a burdensome and strenuous task

iii. created more focus on a broad range of

development goals

A. Only i B. Only ii

C. Only iii D. Both i and ii

E. All the above

61. The best way to end violence against women

and girls is to prevent it from happening in

the first place by ______.

i. addressing its root and structural causes.

ii. redressing the wounds caused by it.

iii. Curbing it wherever visible.

A. Only i B. Only ii

C. Only iii D. Both i and ii

E. Both ii and iii

Direction (62-64): In the given question,

four statements are given which are

connected in different ways in the options

given below. Determine the option in which

the four statements have been the most

appropriately expressed.

62. The weather was stormy. The fishermen had

been warned the previous night. The

fishermen were in danger. Anyone could have

got drowned in such weather.

A. The weather was so stormy that anyone

could have got drowned and the fishermen

were in danger, so they had been warned the

previous night.

www.gradeup.co

10

B. The weather was stormy enough to have

the fishermen in danger as they had been

warned the previous night and anyone could

have got drowned in such weather.

C. The fishermen had been warned the

previous night as they were in danger in the

weather was stormy and anyone could have

got drowned in such weather.

D. The fishermen were in danger as the

weather was stormy as they had been waned

the previous night and anyone could have got

drowned in such weather.

E. None of these

63. Timothy was with Philip since he was a child.

Timothy was a playful dog. Philip considered

Timothy to be a special dog. Philip thought

Timothy could always protect him.

A. Timothy was with Philip since he was a

child and was considered to be special by him

who was a playful dog and could always

protect him.

B. Timothy was with Philip who considered

him to be a special dog and could always

protect him as he was with him since he was

a child and was a playful dog.

C. Timothy was a playful dog who was with

Philip since he was a child and was considered

a special dog who could always protect him.

D. Timothy was with Philip who was a playful

child and considered him to be a special dog

and could always protect him.

E. None of these

64. The situation was difficult. The girl was only

seven years old. The girl had no relative to

look after her. The financial condition too was

grave.

A. The situation was difficult and grave as the

financial condition was bad and the girl was

only seven years old had no relative to look

after her.

B. The situation was difficult as the girl who

was only seven years old had no relative to

look after her and the financial condition was

grave as well.

C. The girl was only seven years old and the

financial condition was too grave as she had

no relative to look after because the situation

was difficult.

D. The situation was grave and difficult

however the girl was only seven years old and

had no relative to look after her and the

financial condition was grave.

E. None of these

65. Direction: In the given question, there are

four statements A, B, C and D that have to be

arranged in a logical order to make a

paragraph between 1 and 6.

1) In a move that will encourage banks to

lend more for housing and possibly make

high-value home loans cheaper, the Reserve

Bank of India (RBI) on Wednesday reduced

the capital requirement for home loans.

A) The repo rate is the rate at which it lends

to banks, while the reverse repo is the facility

through which it borrows from banks.

B) It also retained the cash reserve

requirement (CRR) at 4%.

C) The RBI has also improved the lendable

resources of banks by Rs 50,000 crore by

reducing the proportion of deposits that banks

have to invest in government bonds.

D) In its monetary policy review, the RBI

retained the repo rate at 6.25% and the

reverse repo rate at 6%.

6) Significantly, however, the RBI cut its

projection for consumer inflation to 2-3.5% in

the April to September period, down from

4.5%, and to 3.5-4.5% in October to March,

down from 5%.

A. CDAB B. DABC

C. DBAC D. CBDA

E. BACD

66. Direction: In the given question, there are

four statements A, B, C and D that have to be

arranged in a logical order to make a

paragraph between 1 and 6.

1) Quaint is not an obvious word to use

about America: a country built on revolution,

restless expansion and unabashed pursuit of

profits.

A) Often physical settings added to this sense

of quaintness.

B) From Boston to Philadelphia, or to the

lovingly-restored Georgian streets of

colonial Williamsburg in Virginia many New

World candles of liberty looked strikingly like

the Old all red-brick mansions cobbled lanes

and candle-lit inns, haunted by the host in

tricorne hats.

C) Museums and historic sites depicted the

birth of the United States as a morality tale

and an Anglo-Saxon family dispute, pitting

tyrannical King George and his redcoats

against freedom-loving colonial subjects.

D) Yet for years a cloud of quaintness hung

about many of the country's founding places.

www.gradeup.co

11

6) This can also be found in several other

regions of the great America.

A. DACB B. DCAB

C. BACD D. ABCD

E. DBAC

Direction (67-70): In the following question,

a part of the sentence is underlined. Below

the sentence alternatives to the underlined

part are given at (A), (B), (C), (D) and (E)

which may help improve the sentence. Choose

the correct alternative.

67. The Sanskrit language is of a wonderful

structure; more perfect then the Greek, more

copious than the Latin, and more exquisitely

refined than either, yet bearing to both of

them a stronger affinity.

A. is of a wonderful structure; more perfect

than the Greek, most copious than the

B. is of a wonderful structure; more perfect

then the Greek, more copious then the

C. is of a wonderful structure; more perfect

than the Greek, more copious than the

D. is of a wonderful structure; more perfect

than the Greek, more copious than a

E. are of a wonderful structure; more perfect

than the Greek, more copious than the

68. Field hockey was considered to be the

national game of India, but this had been

recently denied by the Government of India,

clarifying on a Right to Information Act (RTI)

filed that India has not declared any sport as

the national game.

A. is considered to be the national game of

India, but this has been recently denied

B. was considered to be the national game of

India, but this has been recently deny

C. was considerable to be the national game

of India, but this has been recently denied

D. was considered to be a national game of

India, but this has been recently denied

E. was considered to be the national game of

India, but this has been recently denied

69. Indian small screen programming started off

in the mid-1970s where at that time there

was only one national channel Doordarshan,

which was government owned but 1982 saw

a revolution in TV programming in India, with

the New Delhi Asian games, India saw the

colored version of TV, that year.

A. and at that time their was only one

national channel

B. and at that time there was only one

national channel

C. and at that time there was alone one

national channel

D. and of that time there was only one

national channel

E. and at there time there was only one

national channel

70. India has some of the most biodiverse regions

in the world and hosts four of the world’s 35

biodiversity hotspots – or treasure-houses –

that is the Western Ghats, the Eastern

Himalayas, Indo-Burma and Nicobar Islands.

A. India had some of the most biodiverse

regions of the world and

B. India has some of a most biodiverse

regions of the world and

C. India has some of the most biodiverse

regions of the world and

D. India has some of the most biodiverse

region of the world and

E. India has some of the most biodiverse

regions of the world or

71. Direction: In the following question, a part of

the sentence is printed in bold. Below the

sentence alternatives to the bold part are

given at (A), (B), (C) and (D) which may help

improve the sentence. Choose the correct

alternative. In case the given sentence is

correct, your answer is (E) i.e. No correction

required.

The need for conservation of wildlife in India

is often questioned because of the

apparently incorrect priority on the face of

direct poverty of the people.

A. incorrect priority of the fact of direct

poverty of the people

B. incorrect priority in the face in direct

poverty of the people

C. incorrect priority in the mouth of direct

poverty of the people

D. incorrect priority in the face of direct

poverty of the people

E. No correction required

Direction (72-75): In the given question, a

theme followed by three passages is given.

Determine which passage is based on the

given theme and mark it as your option. More

than one passage can be based on the given

theme.

www.gradeup.co

12

72. Women empowerment

A) With the slogan of women empowerment,

the question arises that “are women become

really strong” and “is long-term struggle has

ended”. Many programmes have been

implemented and run by the government such

as International Women’s day, mother’s day,

etc in order to bring awareness in the society

about the true rights and value of the women

in the development of the nation. Women

need to be progressed in the number of

spheres.

B) There is a high level of gender inequality in

India where women are ill-treated by their

family members and outsiders. Women in

India are always subjected to the honour

killings and they never given their basic rights

for the proper education and freedom. They

are the victims who have faced violence and

abuse in the male-dominated country.

C) Giving priority to the gender equality

facilitates women empowerment all over the

country. To get the high-level goal of women

empowerment, it should be promoted from

the childhood in each and every family. It

needs women to be strong physically,

mentally and socially.

A. Only C B. Both A and B

C. Both A and C D. All A, B, C

E. None of the above

73. Competition in the e-commerce sector

A) All of us want to come out on top. We want

to see others subordinated to our superiority.

Our goal is to dominate. If you do not

secretly, or overtly, harbor this desire, then

you should not be in e-commerce. If you see

some good ideas, note them, and implement

them on your own site.

B) If you want to beat your competition, you

are going to have to establish the best supply

chain and maintain the best wholesaler

relationships. This does not always mean the

cheapest source. If you choose the cheapest

wholesaler or distributor, you may end up

compromising on quality or reliability.

C) The market for e-commerce solutions is

extremely competitive and we may find

ourselves unable to compete effectively.

Because there are relatively low barriers to

entry in the e-commerce market, we expect

continued intense competition as current

competitors expand their product offerings

and new competitors enter the market.

A. Only B B. Only C

C. Both A and B D. All A, B, C

E. None of the above

74. Money versus peace of mind

A) Peace of mind is the most important thing

in life, nothing can replace it not even heaps

of money. Money is just a commodity, it can

be earned as well as spent in exchange of

worldly pleasures, but peace of mind is a

state.

B) Since we have done away with the barter

system, we’ve developed various forms of

currency and coins. Money is one of the most

important factors determining one’s success

and life in today’s world. It is true that 40% of

our happiness depends on our actions. Out of

the remaining 60%, 50% is via the genes and

10% via uncontrollable factors. You can’t lose

the important 40% determiner of your

happiness for something called money, can

you?

C) Today, everyone is running: some are

running after money, some are running after

the people they love, some are running after

the situations they desire. No one is at a

stable position i.e. no one is at peace. Inner

peace reflects the peace of mind. In today’s

era, everyone is extremely busy in making

their lives perfect.

A. Only B B. Only C

C. Only A D. Both B and C

E. All A, B, C

75. Impact of Artificial Intelligence

A) Artificial intelligence today is properly

known as narrow AI (or weak AI), in that it is

designed to perform a narrow task (e.g. only

facial recognition or only internet searches or

only driving a car). However, the long-term

goal of many researchers is to create general

AI (AGI or strong AI). While narrow AI may

outperform humans at whatever its specific

task is, like playing chess or solving

equations, AGI would outperform humans at

nearly every cognitive task.

B) Autonomous weapons are artificial

intelligence systems that are programmed to

kill. In the hands of the wrong person, these

weapons could easily cause mass casualties.

Moreover, an AI arms race could inadvertently

lead to an AI war that also results in mass

casualties.

www.gradeup.co

13

C) One of AI’s greatest impacts could be in

food production — an industry challenged by a

rapidly growing world population, competition

for natural resources and plateauing

agricultural productivity. The Food and

Agriculture Organization of the United Nations

(FAO) estimates that the earth’s population

will balloon to 9.7 billion people on by 2050

A. Only A B. Only B

C. Both A and B D. Both A, B, C

E. All A, B, C

Direction (76-80): Study the data given

below and answer the following question. The

pie charts shown below shows the distance

covered by a boat moving upstream and

downstream in different days of a week. And

the table shows the speed of stream in km/hr.

in different days of a week.

76. If the time taken by boat to travel upstream

on Sunday is equal to the time taken by it to

travel downstream on Thursday and the speed

of boat in still water on Thursday is 13 kmph

then find the speed of boat in still water on

Thursday?

A. 27 kmph B. 25kmph

C. 26 kmph D. 28 kmph

E. Can’t determine

77. If the time taken by boat to travel upstream

on Monday is hrs more than the time

taken by it to travel downstream on the same

day, then find the speed of boat in still stream

on Monday if speed of boat in still water is 10

kmph

A. 2.2 kmph B. 1.8 kmph

C. 2.0 kmph D. 2.5 kmph

E. Can’t be determined

78. If the speed of boat in still water on Friday

was 22 km/hr and the speed of boat in still

water on Tuesday was 10/11 times of Friday

and time taken to travel downstream on

Friday is 20/11 hrs more than the time taken

by it to travel downstream on Tuesday, then

find the speed of stream (in kmph) on

Wednesday?

A. 1.5 B. 2

C. 4 D. None of the above

E. Can’t be determined

79. The speed of boat in still water on Saturday

was 20 km/hr. if the time taken by boat to

travel downstream on Saturday is 25/32

times taken to travel upstream on Sunday,

then find the time taken by the boat to cover

a distance of 21.25 km upstream when the

speed of stream is same as that of Sunday.

A. 3 hrs. B. 2.5 hrs.

C. 4 hrs. D. None of these

E. Can’t be determined

80. If the time taken by boat to travel upstream

on Sunday is 6 hours more than the time

taken by it to travel downstream on Thursday

and the speed of boat in still water on

Thursday is 13 kmph, then find the upstream

speed of boat on Sunday?

A. 17 kmph B. 20 kmph

C. 12 kmph D. 15 kmph

E. Can’t be determined

Direction (81-84): There are three bags A,

B and C. In each bag there are three types of

coloured balls Yellow, Green and Black.

In bag A, no. of yellow coloured balls are y

and no. of green coloured balls are g. Number

of green coloured balls are 4 more than the

number of yellow coloured balls. When one

ball is picked at random then the probability

www.gradeup.co

14

of getting black colour ball is , The value of

y is less then g. In bag B, number of

yellow coloured balls is more than

that of bag A. If two balls are picked at

random from bag B then the probability of

getting both green colour ball is . Total

number of balls in bag B is 75.

In bag C, the ratio of number of green

coloured balls and number of black coloured

balls is 7 : 5. Total number of green and black

coloured balls is 36. If one ball is picked at

random then the probability of getting one

yellow ball is

81. If x number of yellow balls from bag B are

taken and placed into bag A and 20% of black

balls from bag A are taken and placed into in

bag B. If we pick one ball from bag B then the

probability that the ball is of black colour is

. Then find the value of x?

A. 2 B. 4

C. 1 D. 1/2

E. 3/2

82. If one ball picked at random from each of the

bag A and bag B then find the probability that

both of the balls are of the same colour?

A. B.

C. D. None of the above

E. Can’t be determined

83. Difference between the number of green balls

in bag A and bag C is how much percent

more/less than the sum of the number of

black balls in bag A and bag C together?

A. 97.5% B. 91%

C. 75% D. 79.5%

E. None of the above

84. Ram counted balls in Bag C and Bag A but

mistakenly he counted green balls in bag B

instead of C. What is error percentage

(approx.)?

A. 3% B. 2%

C. 5% D. 4%

E. 1%

Directions (85-86): Each of A, B, C and D

need a unique time to do a certain work. A

can do the work in x days and B can do the

work in 2x days. A started the work and do it

for days then he is replaced by B and B

completed remaining work in same time as C

and D together can complete the whole work.

The ratio of the efficiency of C and D is 4 : 5.

85. If C and D work for alternative days starting

from C then they can do the total work

in days.

Then find x value?

A. 33.33 B. 66.66

C. 30 D. 70

E. 80

86. If E and F together work for 24 days then

they are replaced by A and B respectively

then they can do the remaining work in 20

days. If the efficiency of E and F is 5 : 4, If E

and F together complete the whole work then

find the difference between the work done by

E alone and the total work done by F alone ?

A. 1/7 B. 1/9

C. 1/6 D. 3/7

E. 1/3

87. ABCD is a trapezoid. PQRS and MLKJ are two

rhombus. Diagonal of PQRS are 6 cm and 8

cm. One of the angle of MLKJ is 120 degree

and the diagonal bisecting that angle

measures 15 cm. Side of PQRS = AB, side of

MLKJ = CD. Find MN (median of trapezoid)

A. 15 B. 20

C. 10 D. 12

E. Can’t determine

88. A vessel contains 2.5 litres of water and 10

litres of milk. 20% of the contents of the

vessel are removed. To the remaining

contents, x litres of water is added to reverse

the ratio of water and milk. Then y litre of

milk is added again to reverse the ratio of

water and milk. Find y.

A. 128 B. 120

C. 60 D. 30

E. 130

www.gradeup.co

15

Directions (89-92): A,B,C,D and E are five

persons to complete jobs X and Y. Line graph

shows the data regarding the number of days

taken by these persons to complete the job X

and Y.

89. A and B working on job Y alternatively for 10

days. C and D worked alternatively for n days.

If 1/15 of the job is remained then how many

days c alone can finish the job?

A. 14

B. 12

C. 13

D. None of the above

E. Can’t be determined

90. If A worked on job X for 5 days each and C

worked for 3 days. the remaining job was

done by E and D. If the ratio of the number of

days for which D and E worked is in ratio 1:1,

then find the number of days for which C,D

and E worked?

A. 6 B. 3

C. 9 D. 5

E. Can’t be determined

91. If the ratio of the number of days for which D

and C worked on job Y is 12:5 and completed

the work then find the difference between

number of days for which C and D worked.

A. 7 B. 14

C. 21 D. Can’t be determined

E. All of above

92. If B worked on job Y with 4/9 times his work

efficiency and assisted by D every 3rd day

then find the number of days for which B and

D worked to complete the job.

A. 18 B. 13

C. 16 D. 12

E. None of the above

Directions (93-96): Find the correct

relationship between the given quantities:

93. Quantity 1: Find A value where S is an acute

angle and PQ||RS

Quantity 2: A=25

A. Quantity 1 < Quantity 2

B. Quantity 2 < Quantity 1

C. Quantity 1 = Quantity 2

D. Can’t establish a relation

E. None of the above

94. There are 63 cards in a box numbered from

01 to 63. Every card is numbered with only 1

number.

Quantity 1: Probability of picking up a card

whose digits, if interchanged, result in a

number which is 36 more than the number

picked up.

Quantity 2: Probability of picking up a card,

the number printed on which is a multiple of 8

but not that of 16.

A. Quantity 1 < Quantity 2

B. Quantity 2 < Quantity 1

C. Quantity 1 = Quantity 2

D. Can’t establish a relation

E. None of the above

95. Quantity 1: Number of hours in which A, B

and C can complete the work if A, B and C

alone can finish it in 25, 20 and 15 hours

respectively.

Quantity 2: Number of hours in which B can

finish the work if A working alone finishes it in

20 hours while working together they finish it

in 5 hours.

A. quantity 1 > quantity 2

B. quantity 1 ≥ quantity 2

C. quantity 1 ≤ quantity 2

D. quantity 1 < quantity 2

E. quantity 1 = quantity 2

www.gradeup.co

16

96. Quantity 1: In an examination, Ankita scored

35 marks less than Puneeta. Puneeta scored

65 more marks than Meenakshi. Rakhi scored

115 marks, which is 20 marks more than

Meenakshi's. Simpy scored 108 marks less

than the maximum marks of the test. What

approximate percentage of marks did Simpy

score in the examination, if she got 67 marks

more than Ankita?

Quantity 2: The length of a rectangle is

increased by 60%. By what percent would the

width have to be decreased to maintain the

same area?

A. Quantity1 < Quantity2

B. Quantity1 ≤ Quantity2

C. Quantity1 ≥ Quantity2

D. Quantity1 > Quantity2

E. None of these

97. Directions: The question given below is

followed by two statements. You have to

determine whether the data given in the

statements are sufficient for answering the

question. You should use the data and your

knowledge of Mathematics to choose between

the possible answers.

How many students from institute 'A' got the

placement?

I. Number of students studying in institutes A

and B are in the ratio of 3:4 respectively.

II. Number of students who got placement

from institute B is 120% of the number of

students who got placement from institute A

III. 80% of the students studying in institute

B got placement.

A. I and II both B. Only I

C. Only II D. Any one of the three

E. Question cannot be answered even with the

information in all three statements

98. Direction: The question given below is

followed by some statements. Read the

question carefully and determine which of the

given statements is/are necessary/required to

answer the question.

What is the cost of flooring the rectangular

hall?

I. Length and the breadth of the hall are in

the ratio of 3:2

II. Length of the hall is 48 m and cost of

flooring is ₹ 850 Per sq.m.

III. Perimeter of the hall is 160 m and cost of

flooring is ₹ 850 per sq m

A. I and II B. I and III

C. Only III D. I and either II or III

E. Any two of the three

99. Directions: The question given below is

followed by two statements. You have to

determine whether the data given in the

statements are sufficient for answering the

question. You should use the data and your

knowledge of Mathematics to choose between

the possible answers.

What is the rate of interest Percent per

annum?

I. An amount doubles itself in 5 yr on simple

interest.

II. Difference between the compound interest

and the simple interest earned on a certain

amount in two years is ₹ 400.

III. Simple interest earned per annum is ₹

2000

A. Only I

B. II and III

C. Any two of three

D. Either I alone or II and III together

E. Either I and II or I and III

100. Direction: The question given below is

followed by some statements. Read the

question carefully and determine which of the

given statements is/are necessary/required to

answer the question.

Find the two-digit number?

I. The difference between the number and the

number formed by interchanging the digit is

27

II. The difference between two digits is 3.

III. The digit at unit's place is less than that at

ten's place by 3

A. I and III B. I and either II or III

C. II and III D. All of these

E. None of these

101. Directions: The following question is followed

by few statements. You have to study the

question and all the statements given and

decide whether the information provided in

the statement(s) is sufficient/required to

answer the question or not:

If k is an integer and x(x – k) = k + 1, what is

the value of x?

I. x < k

II. x = k + 1

III. x = 3 – k

www.gradeup.co

17

A. Only statement I is sufficient.

B. Only statement II is sufficient

C. Any of the two statements out of I, II and

III are sufficient

D. All three are together sufficient

E. None of these

Direction (102-106): Study the given table

carefully and answer the questions that

follow.

Given below is a table which tells us about the

profit or loss% made on 4 items sold by 6

companies, A, B, C, D, E, and F.

Some of the data is missing in the table which

one has to calculate using the given data.

102. The ratio of the selling price of item I and II

for company A is 4 : 3. Find the profit or

loss% made on item II if the ratio of their cost

price was 8 : 9. It is known that company A

made profit on item I.

A. 12.25% profit B. 12.25% loss

C. D.

E. None of these

103. The marked price of items are same for each

company. The marked price of items are:

Item I – Rs. 2500, item II – Rs. 1750, item III

– Rs. 2400, item IV – Rs. 3200. For company

C, a discount% of 10% is given on item III

while two successive discounts of the equal %

was given on item I by company D. Find the

successive discount% given by company D if

the cost price paid by company C for item III

was equal to the cost price paid by company

D for item I. Company C made profit on item

III while company D suffered loss on item I.

A. 15% B. 12%

C. 20% D. 16%

E. 25%

104. Company F marked item IV at 25% higher

than its cost price. Selling price of this item is

Rs. 4140. Find the marked price of item IV, if

‘d’ is discount% and profit% made by selling

the item is (3d – 9)%.

A. Rs. 4400

B. Rs. 4500

C. Rs. 4680

D. Rs. 5140

E. None of these

105. The selling price of the items sold by

Company E was Rs. 2400, Rs. 2100, Rs. 1870

and Rs. 2030 in the order from I to IV. They

made a profit on items I and IV while they

lost money on item II. Find the profit or loss

made on item III if the company made an

overall profit of 5% by selling all the items.

A. Rs. 200 B. Rs. 400

C. Rs. 500 D. Rs. 420

E. Rs. 560

106. The average selling price of item I sold by

company C and company A is Rs. 3493. The

cost price of the item I for company A and

company C is in the ratio of 5 : 4. Find the %

by which the selling price of this item sold by

Company C is greater or less than the selling

price of this item sold by company A. Both

companies made profit on the item.

A. 17.2% B. 18.54%

C. 19.25% D. 20.6%

E. 21.36%

Directions (107-108): The following

information should be used to solve the

following 2 questions. The information given

in a question or the answer may be used in

the other question as the 2 questions are

linked. Please solve carefully before

proceeding:

3 partners – A, B, and C – invested in a

business through different means – cash

capital, land assets and equity certificates.

Cash invested by the 3 were in the proportion

2:3:7 respectively; land assets were invested

in the ratio 4:3:5 respectively; and equity

certificates were invested in proportion 1:5:4

respectively.

www.gradeup.co

18

107. If total cash invested was Rs. 42 lakhs, total

land worth invested was Rs. 93 lakhs, total

value of equity certificates was Rs. 65 lakhs,

and all invested for the same time, then what

will be profit share of ‘A’ if the total profit at

the end of 1st cycle is Rs. 1.5 crores?

A. Rs. 45.65 lakhs B. Rs. 37.755 lakhs

C. Rs. 33.375 lakhs D. Rs. 29.895 lakhs

E. None of the above

108. In the next cycle the investments made were

doubled but in the same proportion as in the

previous cycle. However, the period they were

invested changed for this cycle. A invested for

10 months, B invested for 2.5 years and C

invested for 20 months. What is C’s

approximate profit share if the total profit this

time was Rs.3 crores?

A. Rs. 2.25 crores B. Rs. 2.50 crores

C. Rs. 1.63 crores D. Rs. 1.27 crores

E. Rs. 2.22 crores

Direction (109-110): Study the following

information carefully and answer the question

given billow:

Two trains A & B are travelling in the same

direction at a certain speed. Length of the

train A is 300 metres and the speed of the

train B is 30km/hr. The train A passes a man

who is walking at a speed of 4km/hr along the

line in the same direction as ‘A’ in 30

seconds.

109. If the train A reached the station in 15

minutes after it had passed the man. In what

time the man reached the station?

A. 1 hour B. 1.5 hours

C. 2 hours D. 2.5 hours

E. Data insufficient

110. If the train A passes the train B completely in

5.4 minutes find the length of the train B in

metres?

A. 500m B. 750m

C. 822m D. 850m

E. 600m

Directions (11-112): Study the following

diagram and convert it into other diagrams by

implementing the instructions which is given

in each step to get next step.

Step 1- Diagonally interchange second letter,

if box contain only one letter then consider it

as a first letter and no interchange process

takes place.

Step 2-

I) if box contain even number

a) If one letter is consonant and other one is

vowel then vowel increase by 4 values and

opposite it and consonant letter first opposite

then increases by 5 values and interchange

both positions.

b) If both letters are vowel then increases

both letters position by 5 values and

interchange their positions and opposite both

letters.

c) If both letters are consonant then

decreases both letters positions by 6 values

and interchange their positions and opposite

both letters.

d) If there is only one letter either vowel or

consonant, if letter is vowel then vowel letter

decreases by 7 values and opposite this vowel

letter and then increases by 6 values. If this

letter is consonant then increase letter value

by 7 and opposite this consonant letter and

then decrease by 6 values.

II) If box contain odd number

a) If one letter is consonant and other one is

vowel then vowel decrease by 4 values and

opposite it and consonant letter first opposite

then decreases by 5 values and interchange

both positions.

b) If both letters are vowel then decreases

both letters position by 5 values and

interchange their positions and opposite both

letters.

c) If both letters are consonant then increases

both letters positions by 6 values and

interchange their positions and opposite both

letters.

d) If there is only one letter either vowel or

consonant, if letter is vowel then vowel letter

increases by 7 values and opposite this vowel

letter and then decreases by 6 values. If this

letter is consonant then decrease letter value

by 7 and opposite this consonant letter and

then increase by 6 values.

www.gradeup.co

19

Step 3-is coded in some special pattern

As per the rules followed in the above step,

find out the appropriate steps for the given

input and answer the following questions.

111. Which element comes in step-3 in the second

column of third row?

A. SS8 B. TE6

C. NR2 D. H7

E. None of these

112. Which element replaces IE6 in step-3?

A. NR2 B. H7

C. TE6 D. SS8

E. None of these

Direction (113-114): In the form of

inference/conclusions are based on the

passages given below. Each passage is

followed by five inferences. You are required

to examine each inference separately in the

context of the passage and decide upon its

degree of truth or falsity. Give answer

More than a decade of abrasion in budgetary

assistance and support from the Union

government, has seriously affected Indian

Railways’ capacity to finance its plan

expenditures. The situation has come to a

pass where the railways must now think of

novel and radical ways to get longer mileage

from its investments. Significantly the

resource crunch has had grievous impact on

the railways. As a result, it will not be in a

position to acquire necessary equipment and

this will seriously affect the railways’ capacity

to serve the needs of the economy in future.

113. Railways had so far believed in conventional

ways in inducing income.

A. If the inference is definitely true, i.e., it

properly follows from the statement of facts

given.

B. In the inference is probably true though

not definitely true in the light of the facts

given.

C. If the data are inadequate i.e., from the

facts given you cannot say whether the

inference is likely to be true or false.

D. If the inference is probably false though

not definitely false in the light of the facts

given.

E. If the inference is definitely false i.e., it

cannot possibly be drawn from the facts given

or it contradicts the given facts.

114. The fiscal position of railways in the earlier

plan period was better than the current plan

period.

A. If the inference is definitely true, i.e., it

properly follows from the statement of facts

given.

B. In the inference is probably true though

not definitely true in the light of the facts

given.

C. If the data are inadequate i.e., from the

facts given you cannot say whether the

inference is likely to be true or false.

D. If the inference is probably false though

not definitely false in the light of the facts

given.

E. If the inference is definitely false i.e., it

cannot possibly be drawn from the facts given

or it contradicts the given facts.

Direction (115-117): Study the information

given below and answer the questions based

on it.

Eight persons A, B, C, D, E, F, G and H are

sitting in a straight line and facing north. No

two successive persons are sitting together

according to the alphabetical order. (I.e. A

cannot sit with B, Similarly B cannot sit with

either A or C). They like different colours i.e.

Red, Blue, Black, Yellow, Brown, Pink, Silver

and White, not necessarily in the same order.

B is 3rd to the left of C who likes Blue. One

person sits between B and the one who likes

Black. E is 2nd to the right of C. The one who

likes Brown is immediate left of E. One person

sits between the one who likes Brown and the

one who likes Pink. D likes Red and sits 2nd to

the right of F. G is not neighbour of C. A

www.gradeup.co

20

doesn’t like Brown. The one who likes Silver is

immediate left of the one who likes White.

115. Who among the following likes Yellow?

A. B B. A

C. E D. H

E. G

116. Who among the following is 3rd to the right of

the one who likes Blue?

A. No one B. G

C. E D. H

E. A

117. How many persons sit between B and the one

who Red?

A. 0 B. 1

C. 2 D. 3

E. 4

Direction (118-120): Study the following

information carefully and answer the

questions given below:

‘#’ – Either the hour or minute hand of clock

on 3

‘$’ – Either the hour or minute hand of clock

on 12

‘%’ – Either the hour or minute hand of clock

on 4

'@' – Either the hour or minute hand of clock

on 8

‘+’ – Either the hour or minute hand of clock

on 5

Example: Time ‘#%’ represents 3:20PM All

the times are in PM.

The first symbol represents hours and second

symbol represents minutes

118. A trains reaches station at time '% @' If it

gets late by 3 hours 35 minutes, then at what

time it reaches the station?

A. @+ B. @@

C. @# D. #+

E. %#

119. A person has to catch a train that is scheduled

at ‘@+’. It takes the person 5 hours to reach

the railway station from his home. At what

time should he leave from his home for the

railway station to arrive at the station to

reach exactly 5 minutes before the departure

of the train?

A. #@ B. @#

C. ## D. #%

E. %%

120. A train is scheduled to leave the station at

‘@@’. A person has reached the station 20

minutes before the train’s scheduled time. At

what time the person has reached the station?

A. @# B. @%

C. %@ D. %#

E. $+

Direction (121-125): Study the information

given below and answer the questions based

on it.

6 cars- M, N, O, P, Q and R are parked in a

straight line not necessarily in the same

order. Distance between each car is

successive multiple of 3 but not necessarily in

the order. The distance between car N and car

O is 36m and no car is parked between them.

The distance between car M and car O is

102m. Car R is 99m to the right of car N. Only

one car is parked between car O and car R.

Car M is parked to the immediate left to car P.

The distance between car M and car P is 12m

more than the distance between car P and car

N. The distance between R and M is more

than 60m. The distance between car P and car

Q is 93m. If car Q moves 20m to the north

then takes a left turn and moves 50m then

again takes a left turn and moves for 10m

and stops at point Z. Car A is 16m to the west

of point Z. Car A moves 66m towards west

and stops at point Y.

121. In which of the following direction is point Z

with respect to car N?

A. South-east B. North-east

C. North-west D. South-west

E. North

122. What is the distance between car P and car R?

A. 132m B. 112m

C. 126m D. 144m

E. 99m

123. Point Y is in which of the following direction

and distance with respect to car M?

A. 10m, South B. 10m, North

C. 20m, South D. 20m, North

E. 10m, North west

124. What is the distance between point Z and

Point Y?

A. 82m B. 88m

C. 84m D. 76m

E. 66m

www.gradeup.co

21

125. What is the maximum distance between two

cars?

A. 171m B. 175n

C. 165m D. 163m

E. 172m

Direction (126-127): Each passage is

followed by a set of five statements. Answer

according to the directions given for each

question.

Whenever ISIS carries out a new atrocity,

whether it's beheading a group of Egyptian

Christians or enslaving Yazidi women in Iraq

or burning its victims alive, the big question

most people have is: Why on Earth is ISIS

doing this? What could possibly be the point?

Adding to your list of enemies is never a

sound strategy, yet ISIS' ferocious campaign

against the Shia, Kurds, Yazidis, Christians,

and Muslims who don't precisely share its

views has united every ethnic and religious

group in Syria and Iraq against them. ISIS is

even at war with its most natural ally, al

Qaeda in Syria. The Nazis and the Khmer

Rouge went to great lengths to hide their

crimes against humanity. Instead, ISIS posts

its many crimes on social media for global

distribution with seemingly no thoughts for

the consequences.

126. Which if the above can be safely inferred from

the above passage?

A. The ISIS is not motivated as much by

political gains and perceptions as by the

coverage its activities attract.

B. The Nazis and Khymer Rouge were large

scale terrorist organisations like the ISIS.

C. The Nazis and Khymer Rouge had very

little access to media that pervaded

boundaries since they lived in a world that

was much less connected as it is now.

D. The ISIS is at loggerheads with its natural

ally, the Al Qaeda so that it may establish

natural superiority in the region of the Levant.

E. The ISIS is a fundamentalist group that

believes that we all live in a post apocalyptic

world where concepts of right and wrong do

not matter.

127. Before the arrival of new trainer, the sales

output in AMS learning system Ltd. had been

rising by 20% per year on the average over

the past ten years. However, after new

training innovations by the trainer (which

included computerization of training processes

and reduction in the need for additional work

force) annual sales output has only risen by

10% this year. It appears that Joe’s

innovation has caused the reduction in the

annual growth rate.

Which of the following, if true, would most

seriously weaken the conclusion above?

A. The investment in computerization has a

provision for depreciation of the cost of the

computers

B. Increases in selling price did not follow

increases in the cost of the inputs.

C. The innovation brought in by the new

trainer were intended as long term

investments and not made for short term

profit growth.

D. General demand for the training provided

by the company has declined.

E. None of these

Direction (128-130): Study the given

information carefully and answer the given

questions.

An input-output is given in different steps.

Some mathematical operations are done in

each step. No mathematical operation is

repeated in next step but it can be repeated

with some other mathematical operation (as

multiplication can be used with subtraction in

step 1 and same can be used with addition in

step 2)

As per the rules followed in the steps given

above, find out in each of the following

questions the appropriate step for the given

input

www.gradeup.co

22

128. What is the result of addition of numbers

obtained in step 3?

A. 96 B. 97

C. 98 D. 108

E. None of these

129. What is the result of multiplication of

individual numbers in block obtained in last

step?

A. 14 B. 16

C. 18 D. 20

E. None of these

130. What is the result of division of 2nd block and

1st block values in step 2?

A. 03 B. 04

C. 02 D. 06

E. None of these

131. Direction: In the given question, a statement

is given with some assumptions as options.

An assumption is something supposed or

taken for granted. You have to consider the

statement and the following assumptions and

decide which of the assumptions is implicit in

the statement.

Statement:

The school, which used to provide free

breakfast to ensure that the students receive

at least one proper meal a day, stopped the

facility from January as the Charity Society

which used to fund the breakfast withdrew the

support due to drop in donations.

Which of the following is an assumption

implicit in the above statement?

A. The students may not receive any proper

meal in a day from January

B. In future the donation to the Charity

Society may further decrease.

C. The donations received by the Charity

Society are used for social welfare.

D. The families either do not have the ability

to provide proper meal to their children or

ignorant of what a proper meal is.

E. None of these.

132. Direction: Each of the following question

given below consists of a question and three

statements. You have to decide whether the

data provided in the statements are sufficient

to answer the question.

There are eight persons sitting in the two

rows facing each other. In one row, person I,

J, K, M are sitting and in other row, persons

Q, S, T, U are sitting. Who faces the person

U?

Statement 1: Q is sitting second to the left of

the person U. K and J are immediate

neighbours of each other. M is at one of the

extreme ends.

Statement 2: U is sitting opposite to the

person who is second to the left of the person

K. I is sitting second to the right of the person

J. Q and T are immediate neighbours.

Statement 3: K faces Q. T is second to the left

of the person S. M is second to the left of

person T. S is sitting immediate right of Q

A. Statement I alone is sufficient to answer

the question.

B. Statement II alone is sufficient to answer

the question.

C. All the statements I, II and III taken

together are sufficient to answer the

question.

D. Neither of the I, II, III statements are

sufficient to answer the question.

E. Any of the two statements taken together

are sufficient to answer the question.

133. Directions: In each of the following

questions, a question is followed by

information given in three statements. You

have to study the question alone with the

statements and decide the information given

in which statement is/are sufficient to answer

the question.

Eight boxes A, B, C, D, E, F, G and H are kept

one above another. Which of the following box

is at the bottom position?

I. Two boxes are between D and E. D is above

E. Three boxes are between E and B.

II. Box B is just above D. Two boxes are

between D and E. Box F is just below box E.

III. Two boxes are between E and G. Box C is

above box F.

A. I and II

B. II and III

C. Either I and III or II and III

D. Any of two

E. All of them

134. Read the paragraph and answer the questions

given below:

The building owners wants the rent control by

government to be removed. They say that the

rent will increase but it will be for a short

time. The increased rent will lead to more

investment in real estate and lead to more

number of residential buildings. Thus, the

increased supply will lower the rent rapidly.

www.gradeup.co

23

So, removing the rent control will lower the

rent.

Which of the assumption if true will seriously

damage the argument?

A. The increase in rent is necessary due to

inflation and economic crisis.

B. The probability that income from rent will

be fully invested in more real estate and

constructions.

C. Government introduced rent control to

prevent exploitation due to lack of proper

housing especially to people migrating to

cities.

D. The demand will remain the same but the

supply will increase.

E. None of the above

Direction (135-136): Study the following

information carefully and answer the

questions given below:

Twelve persons A, C, F, H, I, J, K, L, Q, S, T

and W are sitting in two squares, such that

the larger square have middle and corners

seats. The smaller square has only middle

seats. The persons sitting on larger square

are facing towards the centre and person

sitting on smaller square are facing outside

the centre. The smaller square is embedded in

the larger one. A, who is sitting at one of

corners, is sitting second to the right of F. L

and S have only one member between them.

The one, who is sitting opposite to T, is

second to right of J. C and K are immediate

neighbors, such that one of them is sitting at

one of the middle seat. I is sitting third to the

left of Q, who is sitting at one of the middle

seat. J is not immediate member of either of

K or A. W faces K. H has same members

between him and J as between person A and

I. W is sitting second to the right of T, who is

immediate left of S, who is sitting in smaller

square. (Note: The person sitting on Middle

seat means that the person is in between two

corner seats).

135. Four of the following five are alike in a certain

way based on the given seating arrangement

and thus form a group. Which one of the

following does not belong to group?

A. L B. S

C. I D. J

E. None of them

136. Who faces opposite to the person who is

sitting second to the left of Q in smaller

square?

A. T B. L

C. S D. H

E. J

Direction (137-138): Study the following

information carefully and answer the given

questions.

There are equal number of males and females

in a family. Family members are A, B, C, D, E,

F, G & H. There are also three married

couples in the family and two of them are

children. F is the sister-in-law of G. G is the

daughter of B. A is the son of H and brother of

G. F is the mother of C. C is the niece of G. D

is the brother-in-law of A. A is the father of E.

H is the grandfather of E.

137. What is the relation of D with respect to B?

A. Son of B B. Grandson of B

C. Son-in-Law of B D. Brother-in-law of B

E. None of these

138. Who is granddaughter of H?

A. G B. C

C. E D. F

E. None of these

139. Direction: Each passage is followed by a set

of five statements. Answer according to the

directions given for each question.

The average American voter does not care

about Libya. That may sound harsh, but it’s

generally true. The average American does

however care a great deal about gas prices.

So perhaps it is no surprise that as the regime

of Moammar Gadhafi falls, American news

outlets are trying to explain the news out of

Libya within the frame of prices at the pump.

Heck, it was the first idea that came to my

mind as I thumbed through the morning

news.

Adding insult to injury is the amount of fuel

being used to conduct these fuel-based

military operations in the Middle East. A

CNN.com article pointed out recently that,

“One out of eight U.S. Army casualties in Iraq

was the result of protecting fuel convoys. A

post on Scaling Green contained a video of

FTI Consulting’s Adam Siegel recalling a chat

with Gen. Richard Zilmer. In that

www.gradeup.co

24

conversation, the former commander of

troops in the Anbar province of Iraq told

Seigel, “I need renewable energy because

getting fuel to my base is putting people's

lives at risk.”

Which of the above statements can be a just

explanation for the way American news

agencies are portraying the Libyan news?

A. The general American public has several

issues closer home related to everyday

politics to worry about and does not want to

spend precious time worrying about the fall of

an African country far away.

B. American news media, as is the tendency

of the rest of the Western media indulges in a

misguided portrayal of Middle Eastern

monarchies and dictatorships.

C. Renewable energy will not require convoys

to transport it and hence save American lives.

D. With the dearth of oil in the United States,

it is but natural that the USA should look out

for its interests and use the prospect of felling

a dictatorial regime for gains in oil.

E. American news outlets are pandering to the

general public and putting the Libyan crisis in

a context that the average American can

understand.

140. Study the following paragraph and answer the

given question.

BSNL has launched its Customer Acquisition

and Customer Communication Management

(CACMS) project that digitalizes mission

critical, customer-centric business process to

provide customer experience across all

channels throughout the customer lifecycle.

Which of the following will be an impact of the

CACMS launched by BSNL?

A. To centralize the business processes to

ensure faster service, accurate customer

responses and customer delight.

B. Customer onboarding is now being

increased from hours to days

C. Customer onboarding is now being

decreased from days to hours

D. Both 1 and 2

E. Both 1 and 3

Direction (141-145): Study the following

information carefully and answer the following

questions:

There are ten exams to be conducted from 9

to 13 of a month. There are two time slots for

every day, morning slot and evening slot. The

maximum marks of these exams are from 70

to 160. The maximum marks of every subject

are unique. The time durations of the exams

is from the range of 1hr to 5hr. The time

duration is also unique for each subject. The

subjects are Math, English, Physics, Bio,

Chemistry, Economics, History, Home

Science, Geography and Accounts.

The first exam is of math and of 2hrs. The

time duration of last exam is 15 minutes more

than first exam. The exam of English is of 80

marks and on the same day of the exam of

math. Total marks of the exams of first day

are 180 marks. There are two exams between

history and accounts. The last exam is of

accounts and of 110 marks. There are three

exams between English and the exam which is

of 90 marks. The exam of physics is in

morning and it is on an even date. The exam

of 3hr and 5hr are on the same day. The sum

of timings of math and physics is 5hr. The

exam of 130 marks and 90 marks are on the

same day. The exam of chemistry is not of 90

marks. The exam of geography is of 41/2 hrs.

and on the last day. The exam of home

science is on an even date and of least marks.

The exam of chemistry is before exam of

economics. The exam of 150 marks is of

maximum time duration. The exam of

economics is of economics is of least time

duration. The sum of time duration of

economics and history is of 5hrs.The exam of

Chemistry is of 31/2 hrs. duration. The

maximum marks of physics is 30 marks less

than that of biology. The maximum marks of

41/2 hrs. exam is 30 marks more than marks

of accounts. History is of maximum marks.

The maximum marks of home-science are

least of all.

141. Which two exams are scheduled for 10th of the

month?

A. Physics and Biology

B. Chemistry and History

C. Geography and Accounts

D. History and Home Science

E. Math and English

www.gradeup.co

25

142. Find the odd one out?

A. Math B. Physics

C. Chemistry D. English

E. History

143. Which of the following is an incorrect

statement?

A. Math is of 100 marks exam

B. Chemistry is scheduled on 11th of the

month

C. History is of 4 hrs

D. Accounts is of 140 marks

E. Biology is of 5 hrs

144. Which statement is correct for Economics

exam?

A. Economics is of 90 marks

B. Economics exam is on the same day on the

exam of chemistry

C. Economics exam is of 1 hr

D. The exam of economics is on 11th of the

month

E. All are correct

145. Which is the correct combination of the Exam,

time, day and Maximum marks?

A. Math – 3hr – 9th – 130 marks

B. Math – 2hr – 9th – 100 marks

C. Physics – 3hr – 11th – 120 marks

D. Geography – 4.5 hr – 12th – 140 marks

E. All are correct

Direction (146-147): Study the information

given below and answer the questions based

on it.

First 10 numbers starting from 1 is written

from top to bottom. The word “COVER” is

written in alphabetical order against each

even number from top to bottom. Two letters

are between C and B. The number of letter is

written between M and E is same as O and G.

M is written above E. Two letters are written

between V and S. (No letter is repeated

against any number)

146. If there are 2 alphabets in English

alphabetical series between alphabets written

against numbers 2 and 4. Then how many

alphabets in English alphabetical series are

there between the alphabets written against

number 2 and 1?

A. Four B. Five

C. Six D. Seven

E. Eight

147. “G” is written against which of the following

number?

A. Number 7 B. Number 9

C. Number 10 D. Number 5

E. Number 1

148. Direction: Read the short passage given below

and answer the question that follows it.

Cash is the mother's milk of crime. Its appeal

to criminals is clear. Unlike cars or paintings,

it can be concealed immediately after being

pinched. It has no security features to prevent

its being easily and anonymously spent on

legal or illegal goods. Unlike nearly any other

object that can be stolen, it needs no fence.

But a new paper from the National Bureau of

Economic Research found that electronic

payments led to a drop of 9.8% in the overall

crime rate and caused the rates of burglary,

assault, and larceny to fall by 7.9%, 12.5%,

and 9.6% respectively. The introduction of

electronic payments was also associated with

a lower number of arrests, an indication that

the crime rates decline did not stem from

more aggressive policing. The paper said that

the shift from cash to cards since 1990 debit-

card transactions have risen 27-fold, whereas

cash volume has grown by just 4% a year

may have also contributed to the decline in

crime.

148. Which of the following conclusions can

most properly be drawn from the

information above?

A. Criminals' need for cash motivates much

predatory street crime.

B. The more payment transactions are

digitized, the less opportunity for crime

C. If cash motivates crime, an absence of

cash can reduce crime

D. A cashless economy leads to a safer

society

E. Instead of cash barter system should be

reintroduced.

Direction (149-153): Study the information

given below and answer the questions based

on it.

There are 10 shelves numbered 1, 2… 10.

They are arranged in two rows one above the

other. The shelves 1, 2, ….5 are in row 1 and

rest in row 2, which is above row 1. The

shelves are arranged in increasing order of

www.gradeup.co

26

number given to them. Like the shelf number

1 is placed on extreme left of row 1, then

shelf number 2 and so on. Similarly, the shelf

number 6 is placed on extreme left of row 2,

and so on. Each shelf contains a certain

number of glass slabs and photo frames.

There is at least one glass slab in each shelf.

The length of each glass slab is 16 cm and

that of each photo frame is 8 cm. The shelf 2

has length 24cm. There is one shelf between

shelf 2 and Green shelf. The Green shelf

contains 1 glass slab and 4 photo frames

more than shelf 2. The Black shelf is just

above the Green shelf. The Black shelf

contains same glass slab as Green Shelf and 2

photo frames less than Green shelf. No shelf

is between the Blue shelf and the Black shelf.

There is one shelf between Blue shelf and Pink

shelf. The Pink shelf is not at the extreme

right end. The length of Blue shelf and Black

shelf is same but Black shelf has 1 glass slab

less than Blue shelf. The length of Pink shelf is

16cm less than the Black shelf. The Pink shelf

has same photo frames as Yellow shelf. The

Yellow shelf is even number shelf in row 1.

Total length of shelf 7 and shelf 8 is 88cm.

Shelf 7 is white colour which has length more

than Pink shelf. The white shelf has same

glass slab as Blue shelf. The Brown shelf is

just below the Orange shelf.

There is one shelf between Brown shelf and

Red shelf. The Orange shelf has same number

of glass slabs and photo frames as Pink shelf.

The Brown shelf has same glass slab as Yellow

shelf and same photo frames as Green shelf.

The Violet shelf is immediate next in number

to Green shelf. The Violet and Red shelf have

same length, and both have 2 glass slabs

each. The total length of row 1 is 248cm.

149. Which of the following is the colour of shelf 6?

A. Green B. Blue

C. Pink D. Orange

E. White

150. Which of the following is the length of the

Green shelf?

A. 48cm B. 72cm

C. 40cm D. 36cm

E. 56cm

151. What is the total length of shelf 1 and shelf 2?

A. 70 cm B. 90 cm

C. 80 cm D. 100 cm

E. 88 cm

152. What is the colour of the shelf which is just

below the Blue shelf?

A. Violet B. Green

C. Red D. Yellow

E. Brown

153. What is the total length of row 2?

A. 258cm B. 242cm

C. 240cm D. 252cm

E. 262cm

154. Direction: Read the short passage given

below and answer the question that follows it.

Grilling meat gives it great flavour. This taste,

though comes at a price, since the process

creates free radicals which damage DNA and

thus increase the eater's chances of

developing colon cancer. But recent research

has suggested that to find a way around this

problem, one should add beer when

barbecuing meat. Free radicals which form

from fat and protein in the intense heat of this

type of cooking can be stopped by applying

chemicals called antioxidants which mop them

up.

154. The conclusion above would be more

reasonably drawn if which of the following

were inserted into the argument as an

additional premise?

A. Beer reduces the health risk of barbecuing

meat

B. Steaks should be left unmarinated

C. Beer is rich in antioxidants

D. Unmarinated steaks have more free

radicals than those steeped in beer

E. Steaks can be marinated under proper

climatic conditions.

155. Prabir Sengupta was born on 8th May 1963.

He has seemed 65 per cent marks in the

selection process. He has been working for

the past fifteen years in an organization out of

which twelve years as Senior Manager-HR

after completing his post-graduation in HR

with 55 per cent marks. He has successfully

implemented HR system in his organization

during the last two years.

A. The candidate is to be selected.

B. The candidate is Not to be selected

C. The data provided are Not adequate to

take a decision.

D. The case is to be referred to Executive

Director

E. The case is to be referred to Chairman.

The information provided here is only for refrence. It may vary from the original.

www.recruitment.guru

State Bank of India

Probationary Officer’s Examination

Held on 26.11.2006

REASONING

Directions (1-5): Study the following information to answer these question.

I . Seven persons P, Q, R, S, T, V, X belonging to different cities viz. Delhi, Mumbai, Chennai,

Kolkata, Bangalore, Hyderabad and Trivandrum, not necessarily in the same order, went to USA

for attending a Conference. Each one had a different specialisation viz. Literature, Physics,

Economics, Marketing, Computers, Textile Engineering and Information Technology.

II. ‘S’ is from Chennai and ‘Q’ does not have specialisation in Textile or Economics. R is a man of

Marketing and comes from Mumbai. The person from Kolkata has specialisation in Computers.

‘P’ who is specialised in Literature does not belong to Delhi. V having specialisation in Physics is

from Trivandrum. Information Technology is the specialisation of X who comes from Bangalore.

1. Which of the following persons s from Delhi?

(a) P

(b) S

(c) T

(d) S or T

(e) None of these

2. Who is a Textile Engineer?

(a) S

(b) T

(c) V

(d) S or T

(e) None of these

3. Person with specialisation in Literature comes from which of the following cities?

(a) Bangalore

(b) Kolkata

(c) Delhi

(d) Hyderabad

(e) None of these

4. Which of the following combinations of person, city and specialisation is correct?

(a) S-Chennai-Economics

(b) S-Chennai-Textile

(c) Q-Kolkata-Computers

(d) T-Delhi-Economics

(e) None of these

5. Who is specialised in Computers ?

(a) Q

(b) S

(c) T

(d) Data inadequate

(e) None of these

The information provided here is only for refrence. It may vary from the original.

www.recruitment.guru

Directions (6-10): Study the following information carefully and answer the questions given

below:

XYZ Limited Company organised an exhibition of machine tools. The exhibition was opened on

all the weekdays for public. Certain passcode were issued to the visitors as entry card. The

passcode of entry card was changed every hour according to a certain rule as shown below. The

entry time of the first batch of the visitors was 9 AM and that for the last batch was 7 PM. Each

batch was allowed only one hour. The lunch time was from 1 PM to 2 PM.

Batch I (9 AM to 10AM)

Passcode: course easy set for each year was

Batch III (10 AM to 11 AM)

Passcode: easy each course for was set year

Batch III (11 AM to 12 Noon)

Passcode: each was easy for year course set and so on.

6. If the passcode for the batch entering at 12 Noon is “she the girl is clever very good”,

then what will be the passcode for the batch entering at 3 PM?

(a) Clever good is the very she girl

(b) Clever good the is she very girl

(c) Clever good the very is she girl

(d) Clever good very is the she girl

(e) None of these

7. The passcode of which of the following batches will be similar to the passcode for the

batch III?

(a) VI

(b) VII

(c) VIII

(d) IX

(e) None of these

8. If the passcode for the batch III is “pin to the point is sharp not”, then what will be the

passcode for the batch V ?

(a) Is not to sharp point pin the

(b) Is not to point sharp pin the

(c) Not is to sharp point pin the

(d) Not is to point sharp pin the

(e) None of these.

9. If the passcode for the second batch is “for the life is good change got” then the passcode

for which of the following batches is “got change good is life the for” ?

(a) IV

(b) III

(c) V

(d) VI

(e) None of these

The information provided here is only for refrence. It may vary from the original.

www.recruitment.guru

10. If the passcode for batch IV is “do how will the you job now”, then what will be the

passcode for batch II ?

(a) Job will now the do you how

(b) Job now will the do you how

(c) Job will how the do you now

(d) Job will the now do you how

(e) None of these

Directions (11-15): in each question below is given a statements followed by two assumptions

numbered I and II. An assumption is something supposed or taken for granted. You have to

consider the statement and the following assumptions and decide which of the assumptions is

implicit in the statement.

(a) If only assumption I is implicit

(b) If only assumption II is implicit

(c) If either I or II is implicit

(d) If neither I nor II is implicit and

(e) If both I and II are implicit.

11. Statement: ‘Country ‘A’ would explore all channels to diffuse current tensions with

country ‘B’ and bring peace on its borders’ – Statement of spokesperson of country ‘A’.

Assumptions:

I. Country ‘A’ is desirous to diffuse current tension and restore peace with country ‘B’.

II. It is desirable to use more than one channel when complex issues are to be settled

amicably.

12. Statement: Two months ago, it was announced that central government pensioners

would get dearness relief with immediate effect but till date, bank have not credited the

arrears’-A statement from a Pensioner’s Forum.

Assumptions:

I . Most of the banks normally take care of the pensioners.

II . Two months’ time is sufficient for the government machinery to move and give effect

to pensioners

13. Statements: The bridge was built at the cost of Rs. 128 crores and even civil bus service

is not utilising it, what a pity to see it grocely underutilised’ A citizen’s view on a new

flyover linking east and west sides of a suburb.

Assumptions:

I. The building of such bridges does not serve any public objective.

II. There has to be some accountability and utility of money spent on public projects.

14. Statement: Use our product to improve memory of your child, it is based on natural

herbs and has no harmful side effects’-Advertisement of Pharmaceutical Company.

Assumptions:

I. People generally opt for a medical product which is useful and has no harmful side

effects.

II. Improving memory of child is considered as important by many parents.

15. Statements: The traders of State ‘K’ would observe a state-wide bandh as the state has

failed to meet their demand to resolve sales tax and other issues.

Assumptions:

I. The traders of State ‘K’ have earlier tried other usual procedures to get their problem

solved.

The information provided here is only for refrence. It may vary from the original.

www.recruitment.guru

II. The State ‘K’ is not keen to solve the problem of traders.

Directions (16-20) : In the following questions the symbols +, x, ?, @ and $ are used with the

following meaning”

A + B means A is neither equal to nor smaller than B.

A x B means A is neither equal to nor smaller than B.

A ? B means A is neither greater nor equal than B.

A @ B means A is neither greater nor equal to B.

A $ B means A is equal to B.

Now in each of the following questions assuming the given statements to be true find out which

of the conclusions I and II given below them is/ are definitely True?

(a) If only conclusion I is true

(b) If only conclusion II is true.

(c) If either I or II is true

(d) If neither I nor II is true

(e) If both I and II are true.

16. Statements :

C + D, D $ M, M ? C

Conclusion:

I. M $ C

II. D + M

17. Statements :

K $ M, M x B, K + B

Conclusions:

I. M x K

II. K ? M

18. Statements :

F @ G, G ? P, P x H

Conclusions:

I. F @ H

II. H x G

19. Statements :

M @ T, T x R, R $ Q

Conclusions :

I. M ? R

II. M ? Q

20. Statements :

D x G, G @ K , K + M

Conclusions:

I. M + G

II. D x K

The information provided here is only for refrence. It may vary from the original.

www.recruitment.guru

Directions (21-25) : Each of the followings below consists of a questions and two statements

numbered I and II given below it. You have to decide whether the data provided in the

statements are sufficient to answer the questions. Read both the statements and

(a) If the data in statement I alone are sufficient to answer the question, while the data in

statements II alone are not sufficient to answer the question.

(b) If the data in statements II alone are sufficient to answer the question while the data in

statement I alone are not sufficient to answer the question.

(c) If the data either in statement I alone or in statement II alone are sufficient to answer

the question.

(d) If the data even in both statements I and II together are not sufficient to answer the

question.

(e) If the data in both statements I and II together are necessary to answer the question.

21. Which village is to the North-East of village ‘A’?

I. Village ‘B’ is to the North of village A’ villages C and D are to the East and West of

village B, respectively.

II. Village ‘P’ is to the South of village ‘A’ and village ‘E’ is to the East of village ‘P’ village

‘K’ is to the North of village ‘P’.

22. Cab Rohan retire from office ‘X’ in January 2006, with full pension benefits?

I. Rohan will complete 30 year of service in office ‘X’ in April 2006 and desires to retire.

II. As per office ‘X’ rules an employee has to complete minimum 30 years of service and

attain age of 60. Rohan has 3 years to complete age of 60.

23. Among five friends P, Q, R, S and T, who ranks third in terms of salary obtained by them?

I . T’s salary is more than P and Q but not more than S.

II. R’ salary is lowest among them.

24. How is P related to Q?

I . J has two daughters, one of them ‘R’ is marred to ‘P’.

II. Q is the mother of ‘S’, the younger sister of ‘R’.

25. Which word in the code language means ‘flower’?

I . ‘de fu ia pane’ means ‘rose flower is beautiful’ and ‘la quiz’ means ‘beautiful tree’.

II. ‘de la chin’ means ‘red rose flower’ and ‘pa chin’ means ‘red tea’.

Directions (26-30) : Below is given a passage followed by several passible inferences which

can be drawn from the facts stated in the passage. You have to examine each inference

separately in the context of the passage and decide upon its degree of truth or falsity.

(a) If the inference is “definitely true”, i.e. it properly follows from the statement of facts

given.

(b) If the inference is “probably true” though not “definitely true” in the light of the facts

given.

(c) If the “data are inadequate” i.e., from the facts given you cannot say whether the

inference is likely to be true of false.

(d) If the inference is “probably false” though not “ definitely false” in the light of the facts

given”.

(e) If the inference is “definitely false”, i.e., it cannot possibly be drawn from the facts given

or it contradicts the given facts.

In India the asbestos industry is growing and employs more than 15,000 people in 75 units

which are spread over several states like Gujarat. Madhya Pradesh, Maharashtra, Andhra

The information provided here is only for refrence. It may vary from the original.

www.recruitment.guru

Pradesh etc. Surprisingly advanced countries are banning cancer causing asbestos products,

multinational companies are from those countries which are setting up units in developing

countries like India. One reason being lack of awareness in the society and indifference of the

government machinery of these countries. Prolonged exposure to asbestos dust and fibres can

cause lung cancer but most workers in India are too afraid to protect for fear of losing jobs.

Some of these factories are operating in Mumbai. Quite a few of the factories in India are not

known to take adequate precautions to protect workers from asbestos dust.

The Government is taking several steps to provide medical inspection of workers. In fact it has

amended factories act to extend the provision to even those factories employing less than 10

workers.

26. The asbestos industry is one of the largest industries in India.

27. The asbestos industries in India are located in few metropolitan pockets only.

28. The advanced countries are concerned and careful to protect health hazards of its

people.

29. The demand for asbestos products appears to be growing in India

30. The Governments of developing countries appear to be not taking appropriate

measures while granting permission to set up production units of multinational

companies in their countries.

Directions (31-35) : Given below are pairs of events A and B. You have to read both the

events A and B and decide their nature of relationship. You have to assume that the

information given in A and B are true and you will not assume anything beyond the given

information in deciding the answer.

(a) If ‘A’ is the effect and ‘B’ is its immediate and principal cause

(b) If ‘A’ I the immediate and principal and ‘B’ is its effect.

(c) If ‘A’ is an effect but ‘B’ is not its immediate and principal cause.

(d) If ‘B’ is an effect but ‘A’ is not its immediate and principal cause.

(e) None of these

31. Event (A) : We can get anything with money.

Event (B) : Today money is the most important.

32. Event (A) : Due to mechanisation the life of human beings is becoming more

comfortable in unban areas.

Event (B): Life is becoming insecure in urban areas.

33. Event (A) : The Government has decided recently to provide additional dearness

allowance to its employees.

Event (B) : Consumer Price Index is increasing for the last five months.

34. Event (A) : The children of younger generation do better in their study.

Event (B) : The parents of children now realise the importance of education very well.

35. Event (A) : There is considerable increase in the number of people having computers.

Event (B) : Computer education is being made compulsory in schools.

Directions (36-40): Read the following in formation and answer the questions based on it.

(a) The length breadth and height of a rectangular piece of wood are 4 cm, 3 cm, and 5 cm.

respectively.

The information provided here is only for refrence. It may vary from the original.

www.recruitment.guru

(b) Opposite sides of 5 cm, x 4 cm. piece are coloured in red colour.

(c) Opposite sides of 4 cm x 3 cm are coloured in blue

(d) Rest 5 cm x 3 cm. are coloured in green in both sides.

(e) Now the piece is cut in such way that a cuboid of 1 cm. x 1 cm. will be made.

36. How many cuboids shall have all the three colours ?

(a) 8

(b) 10

(c) 12

(d) 14

(e) None of these

37. How many cuboids shall have not any colour?

(a) No any

(b) 2

(c) 4

(d) 6

(e) None of these

38. How many cuboids shall have only two colours red and green in their two sides?

(a) 8

(b) 12

(c) 16

(d) 20

(e) None of these

39. Anita was doing a survey in different companies by working on all days of the week. She

started with company ‘A’ and on completing her work she went to company ‘B’ on the

third day. She was required to work there for three days and then went to company ‘C’

on the fourth day. She worked in company ‘C’ on the fourth day. Thereafter she worked

in company ‘D’ and completed her work on fourth day after working for three days

which was a Saturday. On which day did she start her work in company A?

(a) Thursday

(b) Saturday

(c) Sunday

(d) Monday

(e) None of these

40. How many pairs of letters are there in the word ATMOSPHERE which have as many

letters between them as in the English alphabet ?

(a) Two

(b) Three

(c) Four

(d) Five

(e) None of these

Directions (41-45) : In each question below are given some statements followed by four

conclusions numbered I, II, III and IV. You have to take the given statements to be true even if

they seem at variance from commonly know facts. Read all the conclusions and then decide

which of the given conclusions logically follows from the two given statements disregarding

commonly known facts.

41. Statements :

The information provided here is only for refrence. It may vary from the original.

www.recruitment.guru

Some ice are rings.

No ring is paint.

Some rings are gold.

Conclusions :

(I) No any gold is paint

(II) No any ice is gold

(III) Some rings are paints

(a) Only I and III follows

(b) Only I and II follow

(c) Only III and IV follow

(d) Only II and III follow

(e) None of these

42. Statements :

All gates are flowers.

Some gates are fruits.

Some flowers are clips.

Conclusions :

(I) Some flowers are fruits.

(II) Some clips are fruits.

(III) Some clips are gates.

(IV) No any flower is fruit.

(a) Only I follows

(b) Only I and IV follows

(c) Only II and IV follows

(d) Only I and III follows

(e) None of these

43. Statements :

No candle is bell.

Some shoes are bells.

All tables are shoes.

Conclusions:

(I) Some tables are bells

(II) No table is bell.

(III) Some shoe are candles

(IV) No shoes are candle.

(a) Only I and IV follow

(b) Only I and II follow

(c) Only III and IV follow

(d) Only I and III follow

(e) None of these

44. Statements :

Some films are clouds.

All rate are clods.

Some clouds are chairs.

Conclusions :

(I) No film is chair.

(II) Some rats are films

(III) Some clouds are rats.

The information provided here is only for refrence. It may vary from the original.

www.recruitment.guru

(IV) Some chairs are rats.

(a) Only I and III follow

(b) Either II or IV follow

(c) No any conclusion follows

(d) Only IV follows

(e) None of these

45. Statements :

Some lice are slates

All slates are apples.

No apple is car

Conclusions :

(I) Some cars are slates.

(II) Some lice are cars.

(III) Some apples are lice

(IV) No car is lice.

(a) No any conclusion

(b) Only II follows

(c) Only III follows

(d) Either II or IV and III follow

(e) None of these

Directions (46-50): Read the following information to answer the questions Symbolise the

given number and symbol.

Number : 1 2 3 4 5 6 7 8 9

Symbol : x * ? $ + ! D

(i) If any number by odd number then odd number then the odd number symbolised by @.

(ii) If any number ends by even number then the even number symbolised as ©.

46. Which of the following will be the symbol of 673258?

(a) + ? * $ !

(b) @ + ? * $!

(c) © ? + $ * !

(d) + ? * $ ©

(e) None of these

47. Which of the following will be the symbol of 236475 ?

(a) ? * x @

(b) © ? +@

(c) x ? +@

(d) © + $

(e) None of these

48. Which of the following will be the symbol of 846721

(a) © + * x

(b) ! + * @

(c) ! + * x

(d) © + * x

(e) None of these

49. Which of the following will be the symbol of 178524 ?

The information provided here is only for refrence. It may vary from the original.

www.recruitment.guru

(a) © + ! $ * ©

(b) @ x + ! $ ©

(c) © + ! $ *

(d) * + ! $ ©

(e) None of these

50. Which of the following will be the symbol of 25486 ?

(a) * $ ! D

(b) * $ ! ©

(c) © $ !

(d) * $ !

(e) None of these

51. If 3 is subtracted from the middle digit of the following numbers and then the position of

the digits are reversed, which of the following will be the last digit of the middle

number after they are arranged descending order ?

589 362 554 371 442

(a) 5

(b) 4

(c) 2

(d) 1

(e) 3

52. In a certain code language GERMINATION is written as IMGRENNOAIT. How is

ESTABLISHED written in that code ?

(a) BATESLDEIHS

(b) BAETSLDEIHS

(c) BAETSLEDIHS

(d) BEATSLDEIHS

(e) None of these

53. In a certain code language POULTRY is written as PRQXNVTY. How is TREASON written

I that code ?

(a) TVSGCUQN

(b) TVTGCUQN

(c) TTVGCUQN

(d) TVTHCUQN

(e) None of these

54. If in the English alphabet, all the letters at odd numbered positions are written in serial

order from left to right followed by the letters at even numbered positions written in

reverse order, which letter will be sixth to the left of seventeenth letter from left ?

(a) D

(b) B

(c) V

(d) U

(e) None of these

55. If it is possible to make a meaningful word from the second, fourth, fifth, eleventh and

thirteenth letters of the word ESTABLISHMENT using each letter only once, write

second letter of that word as your answer, if more than one such word can be formed,

write M as your answer and if no such word can be formed, write ‘X’ as your answer.

(a) B

(b) A

The information provided here is only for refrence. It may vary from the original.

www.recruitment.guru

(c) E

(d) M

(e) X

Directions (56-60) : Study the following information carefully and answer the questions given

below:

An institute XYZ provides scholarship to its employees for higher studies in the United States of

America. Following are conditions for awarding scholarship to the employees .

The candidate must:

(i) Not be more than 30 year of age as on April 1, 2006

(ii) Have secured more than 70 per cent marks in Post-graduation and 75 per cent marks in

Graduation

(iii) Have at least two years work experience in the XYZ institute.

(iv) Be ready to sign a bond of two years with the company,

(v) Have got A or A + rating for his/her works in the last two years.

However, in case of a candidate who fulfils all other criteria EXCEPT

(a) (ii) above but has secured minimum 60 per cent marks in Graduation and post

Graduation and he/she has got Ph.D. may be referred to the Director.

(b) (v) above but has work experience of three years with ratings B + A or A + may be

referred to the Chairman

(c) (iv) above but he/she has to leave the institute and he is ready to pay Rs. 50,000, may

be referred to the President.

Based on these criteria and information provided below, decide the course of action in each

case. You are not to assume anything. The cases are given to you as on April 1, 2006. If the data

provided are not adequate to decide the given course of action your answer will be “data

inadequate”.

56. Deepali Mirza working in XYZ institute for the last two and half years. Her performance

is good and has secured ratings A +. She has secured 70 per cent and 75 per cent marks

in Graduation and Post-Graduation respectively. She is Post Graduate from

Mathematics. She is ready to sign the bond of two years. Her date of birth is 31 .03 .1977.

she has also done Ph.D.

(a) Data inadequate

(b) Refer to the Director

(c) Refer to the President

(d) Refer to the Chairman

(e) Not to be selected

57. Ajit D’ souza is a Post-Graduate in Physics with 75 per cent marks and passed

Graduation with 77 per cent marks. He is working in the institute since 2002 and has

always got A ratings on performance. He was 25 years old at the time of joining the

institute. He is keen to seek job in the United States of America.

(a) Refer to the Chairman

(b) Refer to the President

(c) Data inadequate

(d) Not to be selected

(e) To be selected.

58. Rajendra Bhave is a post-graduate in Psychology. He had secured 72% marks at post

graduation and 78 % marks at graduation. He is working in XYZ institute for more than

two years and his performance ratings are A + and A respectively in the last two years.

The information provided here is only for refrence. It may vary from the original.

www.recruitment.guru

He is not keen to sing the bond but does not mind paying Rs. 50,000/- if required to

leave . his date of birth is 14.8.1976.

(a) To be selected

(b) Refer to the Chairman

(c) Refer to the Director

(d) Refer to the President

(e) Not to be selected

59. Harish Chheda ha a bright academic career throughout. He has done post-graduation

with 65% marks and has submitted Ph. D. thesis. His date of birth is 27.9.1979. He is

working for last three years in the XYZ institute and has always got A ratings on

performance. He is ready to pay Rs. 50,000/- in case if required to leave.

(a) To be selected

(b) Refer to the President

(c) Refer to the Director

(d) Data inadequate

(e) Not to be selected

60. 28 years old Tarique Anwar is a post-graduate in Chemistry and has registered for Ph.

D. He scored 80% and 78 % marks at graduation and post-graduation respectively. He is

working in the XYZ institute since 2001. His performance ratings in the last three years

are B + , A+ and A respectively. He is ready to sing two years’ bond with the institute

(a) To be selected

(b) Refer to the President

(c) Refer to the Chairman

(d) Refer to the Director

(e) No to be selected

Direction (61-65) : In each of the questions given below which of the five answer figures

should come after the problem figures if the sequence were continued ?

61.

62.

The information provided here is only for refrence. It may vary from the original.

www.recruitment.guru

63.

64.

65.

The information provided here is only for refrence. It may vary from the original.

www.recruitment.guru

Directions (66-70) : In each of the following questions a related of figures is followed by five

numbered pars of figures. Select the pair of figure which shows similar relationship as that

given in the original pair. You are required to select the best answer from among fairly close

alternatives.

66.

67.

68.

s

69.

70.

The information provided here is only for refrence. It may vary from the original.

www.recruitment.guru

Directions (71-75) : In each of the following questions is four out of the five figures, element I

is related to element II in the same particular way. Find out the figure in which the element I is

so related to element II.

71.

72.

73.

74.

75.

ENGLISH LANGUAGE

Directions (76-90) : Read the following passage and answer the questions given below it .

Certain words/phrases are given in bold lo help you locate them while answering some of the

questions.

The information provided here is only for refrence. It may vary from the original.

www.recruitment.guru

In a disarmingly frank talk at the Indian Merchants Chamber in Mumbai the Japanese

Ambassador in India dwelt at length with issues that exercise the mind of Japanese investors

when they consider investment proposals in India.

Raising the question “ What comparative advantages does India offer as an investment

market?”, he said though labour in India is inexpensive, wage levels are offset by productivity

level to a large extent.

Acknowledging that vastness of the Indian market is great inducement for investment in

manufacturing industry he wondered if it was justifiable to provide that overseas remittance of

profit in foreign exchange to be fully covered by exchange earnings as had been done.

Significantly, on the eve of the Prime Minister’s visit to Japan, the Government delinked profits

repatriation from exports, meeting this demand.

The Ambassador said foreign investors needed to be assured of the continuity and consistency

of the liberalization policy and the fact that new measures had been put into force by means of

administrative notifications without amending Government laws acted as a damper.

The Ambassador pleaded for speedy formulation of the exit policy and pointed to the highly

restrictive control by the Government on disinvestment by foreign partners in joint ventures in

India.

While it is all too easy to dismiss critical comment on conditions in India Contemptuously,

there can be little doubt that if foreign investment is to be wooed assiduously, we will have to

meet exacting international standard and cater at least partially to what we may consider the

idiosyncrasies of our foreign collaborators. The Japanese too have passed through a divided as

substandard and shoddy. That they have come out of the ordeal of fire to emerge as an

economic super power speaks as much of their doggedness to pursue goals against all odds as

of their ability to improvise and adapt to internationally acceptable standards.

There is no gain-saying that the past record of Japanese investment is a poor benchmark for

future expectations.

76. The author has appreciated the Japanese for their

(a) Quality of products manufactured in the fifties

(b) Passing through an ordeal

(c) Perseverance for raising quality of products

(d) Future expectations

(e) None of these

77. According to the Japanese Ambassador, which of the following motivates the foreign

investors to invest in India manufacturing industry?

(a) Very large scope of Indian market

(b) Overseas remittance of profit in foreign exchange

(c) Assurance of continuity of the liberalisation policy

(d) High productivity levels

(e) None of these

78. The purpose of the author in writing this passage seems to be to

(a) Discourage foreign investment in India

(b) Critically examine Indian investment environment

(c) Paint a rosy picture of India’s trade and commerce

(d) Criticise Government’s liberalisation policy

(e) Raise the expectations of foreign investors

79. Which of the following suggestions were expected by the Japanese Ambassador?

(A) Speedy formulation of the exist policy.

The information provided here is only for refrence. It may vary from the original.

www.recruitment.guru

(B) Imposing restrictions of disinvestment by foreign partners in joint ventures in India

(C) Continuity and consistency of the liberalisation policy

(a) All the three

(b) (A) & (B) only

(c) (B) & (C) only

(d) (A) & (C) only

(e) None of these

80. According to the Japanese Ambassador, India offers a comparative advantage of foreign

investors in terms of –

(a) Inexpensive labour

(b) Abysmally low wage levels

(c) Higher productivity

(d) Skilled workforce

(e) None of these

81. For seeking more and more foreign investment the author suggest that we should –

(a) Satisfy fully the whims of our foreign collaborators

(b) Dismiss all critical comments of Indian conditions

(c) Link profit repatriations to exports

(d) Raise the quality of product of match international standards

(e) None of these

82. From the passage it can be inferred that the author is-

(a) A political commentator

(b) A secretary of the Japanese Ambassador

(c) A Japanese investor

(d) An Indian investor

(e) None of these

83. The author attributes Japan’s emergence as an economic super power to

(A) Their ability to overcome any ordeal.

(B) Their tenacity and perseverance despite unfavourable circumstances.

(C) Their ability to improvise and adapt to globally acceptable quality levels

(a) (A) & (B) only

(b) (B) & (C) only

(c) (A) & (C) only

(d) ALL of three

(e) None of these

84. Which of the following statements (s) is/are true about the critical comments on

investment conditions in India.

(A) These comments are difficult to be countered.

(B) These comments are received from various international quarters.

(C) These comments are based more on based than on facts.

(a) Only (C)

(b) Only (B)

(c) Only (A)

(d) (A) & (B) only

(e) (A) & (C) only

Directions (82-86) : Choose the word which is most nearly the SAME in meaning as the word

printed in bold as used in the passage.

85. Assiduously

The information provided here is only for refrence. It may vary from the original.

www.recruitment.guru

(a) Persistently

(b) Hastify

(c) Feebly

(d) Deliberately

(e) Innocently

86. Idiosyncrasies

(a) Demands

(b) Needs

(c) Deviations

(d) Ideologies

(e) Identify

87. Shoddy

(a) Extraordinary

(b) Shabby

(c) Cheap

(d) Disadvantageous

(e) Unprofitable

Directions (88-90) : Choose the word which is most OPPOSITE in meaning of the word printed

in bold as used in the passage.

88. Inducement

(a) Incentive

(b) Motive

(c) Breach

(d) Temptation

(e) Impediment

89. Justifiable

(a) Unreasonable

(b) Formidable

(c) Irrevocable

(d) Unscrupulous

(e) Inevitable

90. Contemptuously

(a) Amicably

(b) Reasonably

(c) Respectfully

(d) Methodically

(e) Indecisively

Directions (91-95) : Rearrange the following six sentences (A), (B), (C), (D), (E), and (F) in the

proper sequence to form a meaningful paragraph; than answer the question given below them.

(A) The application of economic. Environmental and consumer pressures have been on an

increase in recent years.

(B) As a result, our agro-food production and technology are amongst the most advanced in

the world.

(C) They are thus able to provide expertise and technology to satisfy the needs of agro-food

production.

(D) In turn, the support industries have developed to an equally advanced state.

(E) They have also equipped themselves with the necessary expertise to satisfy the most

exacting requirements of the overseas markets.

The information provided here is only for refrence. It may vary from the original.

www.recruitment.guru

(F) These have greatly influenced the development of the agriculture and food industries in

our country.

91. Which of the following should be the SECOND sentence after rearrangement ?

(a) F

(b) E

(c) D

(d) C

(e) B

92. Which of the following should be the THIRD sentence after rearrangement ?

(a) F

(b) E

(c) D

(d) C

(e) B

93. Which of the following should be the FOURTH sentence after rearrangement ?

(a) F

(b) E

(c) D

(d) C

(e) B

94. Which of the following should be the FIFTH sentence after rearrangement ?

(a) F

(b) E

(c) D

(d) C

(e) B

95. Which of the following should be the SIXTH (LAST) sentence after rearrangement ?

(a) F

(b) E

(c) D

(d) C

(e) B

Directions (96-100) : Which of the phrases (a), (b), (c) and (d) given below should replace the

phrase given in bold in the following sentence to make the sentence grammatically correct ? if

the sentence is correct as it is and no correction is required, mark (e) as the answer.

96. They didn’t pay any heed to their superior’s instructions. I did neither.

(a) I either did

(b) Either I did not

(c) Neither didn’t I

(d) Nor did I

(e) No correction required

97. Every poet gives vice to his anger and his compassion through his poems.

(a) Voiced to give

(b) Gives voice for

(c) Gave voice against

(d) Voiced at given

(e) No correction required

The information provided here is only for refrence. It may vary from the original.

www.recruitment.guru

98. The police nabbed a notorious criminal who had been terrorising builders and

extorted money from them for the past two years.

(a) Was terrorising builders and extorted

(b) Had not been terrorised builder and extorted

(c) Had been terrorising builders and extorting

(d) Had terrorised builders and extorted

(e) No correction required

99. Managers frequently encounter situations where they need to help other solved

problems.

(a) To helping others solved

(b) Help others solved

(c) To help others solve

(d) Help others solving

(e) No correction required

100. From among various alternatives we should choose the one which is viable

and consumes less time and energy.

(a) Is viable and consuming lesser

(b) Is viability and consumes less

(c) Being viable and consumed less

(d) Has viable and consuming less

(e) No correction required

101. We are looking forward for good rains this year.

(a) Are looking forward to

(b) Have looked forward for

(c) Have been looking forward at

(d) Should look forward with

(e) No correction required

102. There is full justification to close down the units which are neither profitable

nor serving any social cause.

(a) Which are not profitable or serve

(b) Which have neither profitable nor serving

(c) Which have neither profitable nor serve

(d) Which are neither profitable nor serve

(e) No correction required

103. Please take delivery of the material and acknowledgement at the earliest.

(a) Send acknowledging at the earliest.

(b) Acknowledge at the earliest

(c) Acknowledge the earliest

(d) Early acknowledgement

(e) No correction required

104. The Officer had triggered a controversy by charging his superiors by ignoring

his warning.

(a) By charging his superiors with ignoring

(b) With charging his superiors by ignoring

(c) By charging his superiors after ignoring

(d) For charging his superiors on ignoring

(e) No correction required.

The information provided here is only for refrence. It may vary from the original.

www.recruitment.guru

105. Had we been alone we would have contended ourselves with any plain food that

give us strength

(a) That gives us strength

(b) That gave us strength

(c) Which give us strength

(d) Which give strength to us

(e) No correction required

Directions (106-110) :In each of the following question an idiomatic expression and its four

possible meanings are given. Find out the correct meaning of the idiomatic expression and mark

the number of that meaning as you answer

106. To eat humble pie

(a) To go to ruins

(b) To be earnest

(c) To spread rapidly

(d) To refuse after consenting

(e) To apologize

107. A wild goose chase

(a) An absurdly hopeless enterprise

(b) A fuss over a trifling matter

(c) To be insensitive to criticism

(d) To speak boastfully of oneself

(e) To protect oneself from wild animals

108. To be hard up

(a) To look depressed

(b) To keep starvation away

(c) To act excitedly

(d) To be short of money

(e) To behave like a fool

109. To cry wolf

(a) To come to what is most important

(b) To give false alarm

(c) To turn pale

(d) To be astonished

(e) To run away

110. To take to one’s heels

(a) To act against one’s own interest

(b) To assault

(c) To run away

(d) To have concern

(e) To run oneself

Directions (111-115) : In each of the following sentences there are two blank spaces. Below

each sentence there are five pair of words denoted by numbers a, b, c, d and e, . Find out which

pair of words can be filled up in the blanks in the sentence in the same sequence to make the

sentence meaningfully complete.

111. He objected to the proposal because it was founded on a ____ principle and also

was ___ at time.

(a) Faulty ___ desirable

(b) Imperative ___ reasonable

The information provided here is only for refrence. It may vary from the original.

www.recruitment.guru

(c) Wrong ____ inconvenient

(d) Sound ___ acceptable

(e) Uncomforting ___ deplorable

112. The criterion for ___ a player should be his recent performance; but

unfortunately, the journalists are ___ to be carried away by earlier successes.

(a) Condemning ___ satisfying

(b) Revealing ___ reluctant

(c) Judging ___ prone

(d) Eager ____ acclaiming

(e) Criticising ____clean

113. For the last half century he ____ himself to public affairs ___ taking a holiday.

(a) By____ committed

(b) After ____ offered

(c) Devoted ____ without

(d) Sacrificed____after

(e) Prepared ____ before

114. You will see signs of everywhere, which speak well for the ___ of these people.

(a) Decoration-senses

(b) Clear-debris

(c) Beauty-careful

(d) Industry -prosperity

(e) Repairs-extravaganza

115. The police arrested Ramesh on a ___ of theft but for lack of evidence___ him.

(a) Crime __ imprisoned

(b) Punished __ complaint

(c) Left __ condition

(d) Tip __ absconding

(e) Charge ___ released

Directions (116-125) : In the following passage, there are blanks, each of which has been

numbered. These numbers are printed below the passage and against each, five words are

suggested, one of which fits the blank appropriately. Find out the appropriate word in each case.

(116) stringent antipollution laws, mass awareness levels in India about the need to (117) the

environment are low. Which is (118)many people insist that mere laws won’t do; what we

(119) need are “environment conscious” citizens. It is in this context that the University’s

(120) to introduce environment studies as a compulsory paper at the undergraduate level

(121) significance. There was some (122) initially about who would teach the paper because

financial (123) make it impossible for colleges to (124) approval for new teaching posts. In fact,

in August 1999, the University Grants Commission (UGC) imposed a ban on the creation of new

teaching posts in colleges. (125) with this problem, authorities at the university have decided

that serving teachers belonging to various disciplines will teach the paper.

116. (a). Despite

(b) Having

(c) Enacting

(d) Adopting

(e) Although

117. (a) contaminate

The information provided here is only for refrence. It may vary from the original.

www.recruitment.guru

(b) Clean

(c) Filter

(d) Protect

(e) Pollute

118. (a) resulting

(b) Why

(c) Obvious

(d) As

(e) Because

119. (a) seldom

(b) Don’t

(c) Hardly

(d) Perfectly

(e) Actually

120. (a) inability

(b) Deferral

(c) Decision

(d) Failure

(e) Reluctance

121. (a) extracts

(b) Trivalency

(c) Expects

(d) Loses

(e) Assumes

122. (a) displeasure

(b) Antagonism

(c) Hurdles

(d) Confusion

(e) Priority

123. (a) losses

(b) Constraints

(c) Apathy

(d) Soundness

(e) Independence

124. (a) receive

(b) Establish

(c) Emphasize

(d) Expect

(e) Sanction

125. (a) Down

(b) Familiarity

(c) Faced

(d) Convinced

(e) Solution

QUANTITATIVE APTITUDE

Directions (126-130) : Each of the questions below consists of a question and three

statements denoted A, B and C are given below it. You have to study the question and all the

The information provided here is only for refrence. It may vary from the original.

www.recruitment.guru

three statements and decide whether the question can be answered with anyone or two of the

statements or all the statements are required to answer the question.

126. How much marks did Arun secure in English?

A. The average mark obtained by Arun in four subjects including English is 60.

B. The total mark obtained by him in English and Mathematics together is 170.

C. The total mark obtained by him in Mathematics and Science together is 180.

(a) All three A, B and C together are necessary.

(b) Only A and B together are necessary

(c) Only B and C together are necessary

(d) Only A and C together are necessary

(e) None of these

127. How much profit did Mahesh earn on the cost price of an article by selling it ?

A. He get 15% discount on the marked price at the time of purchase

B. He sold if for Rs. 3060.

C. He earned 2% profit on the marked price.

(a) Only A and B both together are necessary

(b) Only B and C both together are necessary

(c) Only A or C and B together are necessary

(d) Even A, B and C all together are not sufficient to answer the question.

(e) All three A, B and C together are necessary

128. What will be sum of two numbers?

A. Among the two numbers, the bigger number is greater than the smaller number by 6.

B. 40% of the smaller number is equal to 30% of the bigger number.

C. The ratio between half of the bigger number and

rd of the smaller number is 2 : 1.

(a) Only B and C together are necessary

(b) Only A and B together are necessary

(c) Out of A, B and C any two together are necessary

(d) All three A, B and C together are necessary

(e) None of these

129. What is the area of a right angled triangle?

A. The perimeter of the triangle is 30 cm.

B. The ratio between the base and the height of the triangle is 5 : 12

C. The area of the triangle is equal to the area of a rectangle of length 10 cms.

(a) Only B and C together are required

(b) Only A and B together are required

(c) Only either A or B and C together are required

(d) Only A and C together are required

(e) None of these

130. What is R’s share of profit in a joint venture ?

A. Q started business investing Rs. 80,000

B. R joined him after 3 months.

C. P joined after 4 months od Rs. 1,20,000 and got 6,000 as his share of profit.

(a) Only A and C are required

(b) Only B and C are required

(c) All A, B and C together are required

(d) Even with all A, B and C the answer cannot be arrived at

(e) None of these

The information provided here is only for refrence. It may vary from the original.

www.recruitment.guru

Directions (131-135) : Study the following table carefully and answer the questions given

below it.

The number of candidates appeared, passed and selected in a competitive examination from five

states over the years.

State

Years

2000

A B C D E

A P S A P S A P S A P S A P S

850 215 25 1050 245 35 990 195 28 1080 300 36 1120 240 40

2001 880 240 20 980 230 30 650 150 28 1150 320 38 960 180 26

2002 750 180 22 1120 210 28 840 180 25 995 280 42 885 177 32

2003 920 290 36 890 190 32 780 160 32 975 260 39 1040 220 30

2004 960 300 32 950 225 40 1020 220 36 888 240 32 980 280 34

2005 820 250 28 1180 200 38 930 215 35 864 216 30 900 228 24

A = Appeared, P= Passed, S = Selected

131. In the year 2000, which state had the lowest percentage of candidates selected over

the candidates appeared ?

(a) A

(b) B

(c) C

(d) D

(e) E

132. During which of the following years the passing percentage over appeared is the

highest from state ‘D’?

(a) 2005

(b) 2004

(c) 2003

(d) 2002

(e) None of these

133. Total number of candidates selected from state ‘A; is approximately what percentage of

the total number of candidates selected from state ‘B’ ?

(a) 72

(b) 88

(c) 85

(d) 75

(e) 80

134. During which of the following years the percentage of candidates selected over passed

is the lowest for state ‘B’ ?

(a) 2000

The information provided here is only for refrence. It may vary from the original.

www.recruitment.guru

(b) 2001

(c) 2003

(d) 2004

(e) None of these

135. What is the ratio between number of candidates passed from state A in 2001 to that

from state E in 2004 ?

(a) 6 : 7

(b) 14 : 15

(c) 13 : 16

(d) 12 : 16

(e) None of these

Directions (136-140) : Study the following graph carefully to answer the question given below it. Production of paper by 3 different companies A, B and C over the years.

136. What is the difference between the production of company C in 1991 and the production of company A in 2004 ? (a) 50,000 tonnes (b) 5,00,00000 tonnes (c) 50,00,000 tonnes (d) 5,00,000 tonnes (e) None of these

137. What is the percentage increase in production of company ‘A’ from 2000 to 2001 ? (a) 37.5 (b) 38.25 (c) 35 (d) 36 (e) None of these

138. For which of the following years the percentage of rise/fall in production from the previous year is the maximum for company B (a) 2000 (b) 2001 (c) 2002 (d) 2003 (e) 2004

0

10

20

30

40

50

60

70

1999 2000 2001 2002 2003 2004

A

B

C

Quan

tity

of

pape

r in

lakh

tonn

es

The information provided here is only for refrence. It may vary from the original.

www.recruitment.guru

139. The total production of company C in 2001 and 2004 is what percentage of the total production of company A in 1999 and 2000? (a) 95 (b) 90 (c) 110 (d) 115 (e) 133.33

140. What is the difference between the average production per year of the company with highest average production and the company with lowest average production in lakh tonnes? (a) 3.17 (b) 4.33 (c) 4.17 (d) 3.33 (e) None of these

Direction (141-145) : Study the following graph carefully and answer the question given below it. The number of students who joined and left the school in the beginning of years for six years, from 2000 to 2005. Initial strength of the school in 1999 = 1500

141. What was the increase/decrease in strength of the school from 2001 to 2002? (a) Increase by 100 (b) Decrease by 100 (c) Increase by 200 (d) Decrease by 200 (e) None of these

142. For which of the following years, the percentage rise/fall in number of students left from the previous years is the highest ? (a) 2001 (b) 2002 (c) 2003 (d) 2004 (e) 2005

143. How many students were there in the school during the year 2003?

0

0.5

1

1.5

2

2.5

3

3.5

4

2000 2001 2002 2003 2004 2005

Number of students left

Number of students joined

The information provided here is only for refrence. It may vary from the original.

www.recruitment.guru

(a) 1495 (b) 1600 (c) 1550 (d) 1700 (e) None of these

144. During which of the following pairs of years, the strength of school is equal ? (a) 2001 and 2002 (b) 2002 and 2004 (c) 2003 and 2005 (d) 2002 and 2005 (e) 2000 and 2002

145. The number of students in 2003 is approximately what percent of the number of students in 2001 ? (a) 85 (b) 117 (c) 95 (d) 103 (e) 108

Directions (146-150) : In each of the following questions a number series is given. After the series, a number is given followed by (a), (b), (c), (d) and (e). You have complete the series starting with the number given following the sequence of the given series. Then answer the question given below it.

146. 9 19.5 41 84.5 12 (a) (b) (c) (d) (e) Which of the following numbers will come in place of (c) ? (a) 111.5 (b) 118.5 (c) 108.25 (d) 106.75 (e) None of these

147. 4 5 22 201 7 (a) (b) (c) (d) (e) Which of the following number will come in place of (d) ? (a) 4948 (b) 4840 (c) 4048 (d) 4984 (e) None of these

148. 5 5.25 11.5 36.75 7 (a) (b) (c) (d) (e) Which of the following number will come in place of (c) ? (a) 34.75 (b) 24.75 (c) 24.5 (d) 34.5 (e) None of these

149. 38 19 28.5 71.25 18 (a) (b) (c) (d) (e) Which of the following number will come in place of (d) ? (a) 118.75 (b) 118.25 (c) 108.25 (d) 118.125 (e) None of these

The information provided here is only for refrence. It may vary from the original.

www.recruitment.guru

150. 25 146 65 114 39 (a) (b) (c) (d) (e) Which of the following number will come in place of (e) ? (a) 122 (b) 119 (c) 112 (d) 94 (e) None of these

Directions (151-155) : Read the following statement carefully to answer the given question. A committee of 12 persons is to be formed from 9 women and 8 men.

151. In how many ways this can be done if atleast 5 women have to be included in a committee? (a) 6000 (b) 6010 (c) 6062 (d) 6005 (e) None of these

152. In how many of these committees, the women are in majority? (a) 2000 (b) 2700 (c) 2705 (d) 2702 (e) None of these

153. In how many of these committees, the men are in majority? (a) 1008 (b) 1100 (c) 1200 (d) 1225 (e) None of these

154. An urn contains 9 red, 7 white and 4 black balls, If two balls are drawn at random, find the probability that both the balls are red

(a)

(b)

(c)

(d)

(e) None of these

155. How many different words can be formed with the letters of the word ‘ALLAHABAD’? (a) 7500 (b) 7560 (c) 7510 (d) 7580 (e) None of these

Direction (156-160) : Study the following graph carefully and answer the question given below. Percentage of different types of employees in a company in two consecutive years

The information provided here is only for refrence. It may vary from the original.

www.recruitment.guru

A 22%

B 10%

C 11%

D 9%

E 27%

F 21%

2006 Total no. of employees

48,640

156. In 2005 the total no. of which the following types of pairs of employees was approximately equal to A type of employees in 2006? (a) B and C (b) A and C (c) D and E (d) C and D (e) C and F

157. From 2005 to 2006 in the case of which of the following types of employees the change was maximum? (a) B (b) D (c) C (d) A (e) None of these

158. What was the approximate different in the number of B type of employees during 2005 and 2006?

A 20%

B 6%

C 10%

D 15%

E 27%

F 22%

2005 Total no. employees 42,980

The information provided here is only for refrence. It may vary from the original.

www.recruitment.guru

(a) 2285 (b) 2325 (c) 2085 (d) 2620 (e) 1825

159. If the no. D type of employees in 2006 was 5000, what would have been its approximate percentage in the company? (a) 8 (b) 12 (c) 14 (d) 16 (e) 10

160. The no. of A type of employees was approximately what per cent of the no. of A type of employees in 2005 ? (a) 115 (b) 140 (c) 125 (d) 130 (e) 95

161. 12 men take 36 day to do a work while 12 women complete

th of the same work in 36

days. In how many days 10 men and 8 women together will complete the same work? (a) 6 (b) 27 (c) 12 (d) Date inadequate (e) None of these

162. Rs. 800 becomes Rs. 956 in 3 years at certain simple rate of interest. If the rate of interest is increased by 4%, what amount will Rs. 800 become in 3 years? (a) Rs. 1020.8 (b) Rs. 1025 (c) Rs. 1052 (d) Data inadequate (e) None of these

163. What least number would be subtracted from 427398 so that remaining number is

divisible by 15? (a) 6 (b) 3 (c) 16 (d) 11 (e) None of these

164. A car covers its journey at the speed of 80 km/hour in 10 hours. If the same distance is to be covered in 4 hours, by how much the speed of car will have to increase? (a) 8 km/hr (b) 10 km/hr (c) 12 km/hr (d) 16 km/hr (e) None of these

165. What approximate value will come in place of the question mark (?) in the following equation ?

33

% of 768.9 + 25% of 161.2 – 58.12 =?

(a) 230 (b) 225 (c) 235

The information provided here is only for refrence. It may vary from the original.

www.recruitment.guru

(d) 220 (e) 240

166. If on selling 12 notebooks any seller makes a profit equal to the selling price of 4 notebooks, what is his per cent profit? (a) 50 (b) 25

(c)

(d) Data inadequate (e) None of these

167. Present age of Rahul is 5 years less than Ritu’s present age. If 3 years ago Ritu’s age was x, which of the following represents Rahul’s present age? (a) (b) (c) (d) (e) None of these

168. A grocer purchased 2 kg . of rice at the rate of Rs. 15 per kg. And 3 kg. of rice at the rate

of Rs. 13 per kg. At what price per kg. should he sell the mixture to earn

% profit on

the cost price. (a) Rs. 28.00 (b) Rs. 20.00 (c) Rs. 18.40 (d) Rs. 17.40 (e) None of these

169. A boat takes 6 hours to travel from place M to N downstream and back fro N to M upstream. If the speed of the boat in still water is 4 km/hr,. what is the distance between the two places? (a) 8 kms. (b) 12 kms. (c) 6 kms. (d) Data inadequate (e) None of these

170. Mr. Yadav spends 80% of his monthly salary on consumable items and 50% of the remaining on clothes and transport. He saves the remaining amount. If his Rs. 5370, how much amount per month he would have spent on clothes and transport? (a) Rs. 4,037 (b) Rs. 8,0763 (c) Rs. 9,691.20 (d) Rs. 4,845.60 (e) None of these

Directions (176-180) : In each question below one or two equation (s) is/are provided. On the basis of these you have to find out relation between p and q.

(a) If p =q (b) If p > q (c) If p < q (d) If p q and (e) If q p 171. I.

II. 172. 173. 174. 175.

The information provided here is only for refrence. It may vary from the original.

www.recruitment.guru

GENERAL AWARENESS 176. Why was Fillip Calderon is the new ?

(a) He has been appointed the secretary General of European Union (b) He has been relected the President of Mexico (c) He has been awarded the international Astronaut of the Year Award by NASA. (d) He has been relected the Prime Minister of Guyana (e) None of these

177. Where was ninth China-European Union summit held ? (a) Olso (b) Sanya (c) Helsinki (d) Shanghai (e) None of these

178. ‘Crown Prince Cup Basketball’ in which India won a Bronze Medal was organised recently in- (a) India (b) Thailand (c) China (d) Japan (e) Singapore

179. ‘Jnanpith Award’ is given for the excellence in the field of (a) Music (b) Politics (c) Literature (d) Sports (e) None of these

180. Ozone gets depleted in the Stratosphere due to the presence of : (a) Active NO3 (b) Active NO2 (c) Active CI (d) Active SO2 (e) None of these

181. This of the following countries has awarded Congressional Gold Medal to Dalai Lama, Tenzing Gyatso? (a) Britain (b) France (c) Italy (d) USA (e) None of these

182. Who amongst the following is the author of the book ‘Bearders-My Life in Cricket’? (a) Shane Warne (b) Bill Frindall (c) Brian Lara (d) Imran Khan (e) None of these

183. Which step is taken first in designing a programme ? (a) Data validation (b) Task analysis (c) Input design (d) Problem identification (e) None of these

184. The government offices of which country are now become paperless? (a) Holland (b) Sweden (c) New Zealand

The information provided here is only for refrence. It may vary from the original.

www.recruitment.guru

(d) Estonia (e) None of these

185. Who amongst the following is the author of the book ‘The city of Joy’? (a) Dominique Lapierre (b) Guentar Grass (c) Daniel Steele (d) Graham Green (e) None of these

186. Who amongst the following inaugurated the Hanover Technology Trade- 2006? (a) Dr. Manmohan Singh (b) Dr. P. Chidambaram (c) Mr. Kamal Nath (d) Mr. Pranb Mukherjee (e) None of these

187. How many items are deserved recently from the reserved list of small scale Industries ? (a) 60 (b) 68 (c) 100 (d) 180 (e) None of these

188. The Government of India decides the minimum support price of which of the following commodities? (a) Sugarcane (b) Green vegetables (c) Medicinal plants (d) Milk (e) None of these

189. The Mid day Meal Scheme is launched by the Union Ministry of- (a) Home Affairs (b) Social Welfare (c) Human Resource Development (d) Rural Development (e) None of these

190. The United Western Bank which was recently in news is a . (a) Nationalized Bank (b) Co-operative Bank (c) Private Bank (d) Foreign Bank (e) None of these

191. Dr. Manmohan Sing was on a visit to Uzbekistan a few months back where he signed seven MoUs/ Pacts. Who amongst the following signed these MoUs/pacts on behalf Uzbekistan? (a) Mr. Islam Karimov (b) Mr. Hamid Wazir (c) Mr. Hassan Wirajuda (d) Sultan Bin Kasim (e) None of these

192. Who amongst the following is the chairperson of the ‘National Knowledge Commission? (a) Mr. Sam Pitroda (b) Dr. Ashok Ganguly (c) Dr. Jayanti Ghosh (d) Dr. Pratap Bhanu Mehta (e) None of these

The information provided here is only for refrence. It may vary from the original.

www.recruitment.guru

193. The final of the Sultan Azlan Shah Hockey Tournament 2006 was organised recently (18 June, 2006) in . (a) Kuala Lumpur (b) Jakarta (c) Manila (d) New Delhi (e) None of these

194. Ustad Bismillah Khan who died recently (21 August, 2006) was famous. (a) Classical Dancer (b) Classical Singer (c) Shehnai Player (d) Tabla Player (e) Santoor Player

195. The two day meeting of the Finance Ministers of G-8 nations was organised recently in (10-11 June 2006) (a) St Petersburg (b) Berlin (c) Milan (d) Moscow (e) Paris

196. Who amongst the following is the recipient of the ‘Rajiv Gandhi Khel Ratna Award’ given away recently (29 August, 2006)? (a) Mahesh Bhupathi (b) Sania Mirza (c) Rahul Dravid (d) Anju Bobby george (e) Pankaj Advani

197. Who amongst the following won the French Open 2006 Tennis cup held recently (May-June 2006) ? (a) Roger Federer (b) Rafael Nadal (c) Bob Bryan (d) Vera Zvonareva (e) None of these

198. Who amongst the following is the recipient of the ‘Sahitya Shiromani Samman’ given away recently ? (a) Javed Akhtar (b) Gulzar (c) Kamleshwer (d) Malti Joshi (e) None of these

199. Which of the following is a graphic package ? (a) Adobe Page Maker (b) Acrobat Reader (c) Microsoft Word (d) Corel Draw (e) None of these

200. Hardware that adds two numbers is . (a) The control unit (b) An internal CPU register (c) Arithmetic logical unit (d) Browser (e) None of these

201. Which of the following statements is/are True about the integrated Anti Submarine Warfare (ASW) equipment ?

The information provided here is only for refrence. It may vary from the original.

www.recruitment.guru

(A) The Defence Research and Development Organisation (DEDO) handed over the integrated Anti Submarine Warfare equipment to Anti navy.

(B) It has the ability to detect enemy usages while being underwater and also fire torpedoes or missiles.

(C) The cost of this equipment is about Rs. 4 crores. (a) A and B (b) A and C (c) B and C (d) All are correct (e) None of these

202. In which plane is the Brahmos missile fitted ? (a) Sukhoi-30MKI (b) Jaguar (c) Mirage (d) Mig-29 (e) None of these

203. From which date the wildlife (Protection) Amendment Act 2006 has come into effect after the president’s approval? (a) 1st September, 2006 (b) 4th September, 2006 (c) 15th August, 2006 (d) 15th July, 2006 (e) 4th June, 2006

204. At which place would the Indian navy set up the second naval base? (a) Vasakhapatnam (b) Mangalore (c) Tuticorin (d) Nellore (e) None of these

205. How many agreements were signed between India and Brazil during the Prime Minister Manmohan Singh Brazil visit ? (a) 5 (b) 7 (c) 9 (d) 11 (e) None of these

206. Who amongst the following was the head of the Investment commission which submitted its report to the Government of India recently? (a) Mr. Ratan Tata (b) Dr. Rakesh Mohan (c) Mr. Kurnar Mangalam Birla (d) Mr. Rahul Bajaj (e) None of these

207. As per the report in the newspaper the old age pension is now raised to which of the following amounts from the present Rs. 75? (a) Rs. 100 (b) Rs. 150 (c) Rs. 175 (d) Rs. 200 (e) Rs. 250

208. The interest rate at which the RBI lends against the Government Securities is known as (a) CRR (b) SLR (c) Bank Rate

The information provided here is only for refrence. It may vary from the original.

www.recruitment.guru

(d) Repo Rate (e) None of these

209. Which of the following is the abbreviated name of an Indian Organisation/agency associated with export of processed food products? (a) APEDA (b) AAPSO (c) AINEF (d) AITUC (e) None of these

210. Which of the following Industries suffered a huge loss of almost Rs. 20 crore pre day due to floods in Gujarat ? (a) Oil and Natural gas (b) Information Technology (c) Sugar mills (d) None of these

211. Which of the following is NOT one of the core and identified under the Bhart Nirman Programme? (a) Irrigation (b) Rural electrification (c) Rural housing (d) Computer education in schools (e) Drinking water supply

212. The Base II Accord for Banking Industry is base on three pillars. Which of the following is/are NOT included in the same ? (a) Minimum Capita Requirement (b) Supervisory Review (c) Market Discipline (d) Credit Risk

Codes : (a) Both (A) and (C) (b) Only (D) (c) (A), (B) and (C) (d) Only (A) (e) None of these

213. ‘Shequel’ is the currency of- (a) Israel (b) Kenya (c) Iraq (d) Iran (e) None of these

214. Indian Exports recorded an increase in May 2006. What was the increase in terms of percentage ? (a) 15 per cent (b) 20 per cent (c) 25 per cent (d) 30 per cent (e) None of these

215. Who amongst the following is the first chairman of the newly Constituted National Statistical Commission ? (a) Prof. Suresh D. Tendulkar (b) Dr. Ashok Lahiri (c) Dr. Rakesh Mohan (d) Prof. S.K. Ghosh (e) None of these

The information provided here is only for refrence. It may vary from the original.

www.recruitment.guru

216. Government of India recently was decided to import wheat at a lower rate of custom duty. The effective rate of the duty was. (a) 15 per cent (b) 12 per cent (c) 10 per cent (d) 5 per cent (e) None of these

217. As per the reports published recently in the newspapers banks surpassed the target set for farm credit was given in this sector in terms of percentage? (a) 15 (b) 20 (c) 26 (d) 33 (e) 43

218. As per the reports published in the newspapers the National Housing Bank (NHB) is planning to launch a Reserve Mortgage scheme specially to held which of the following sections of society ? (a) People in rural India (b) Senior Citizens (c) People living in government accommodations (d) Women who are sole bread winners (e) None of these

219. Which of the following companies has got two contracts to establish two power projects in Afghanistan ? (a) Bharat Heavy Electricals (BHEL) (b) National Hydro Electric Power Corporation (NHPC) (c) National Thermal Power Corporation (NTPC) (d) Reliance Energy Ltd. (e) None of these

220. Government of India recently introduced some measures/incentives to improve tourism in India. Which of the following is one of these measures ? (a) Issuance of electronic visa (b) Free stay for the three days in Government hotels in four metros for business

travellers and frequent players (c) Highly subsidised air travel on metro routes for frequent flyers (d) Free trip to Agra, Delhi and Jaipur to those coming on educational/study tours (e) None of these

221. How much amount can the Postal Department invest in revenue generating instruments and/or stock market to reduce its budgetary deficit? (amount is from its insurance schemes). (a) Rs. 1,400 crore (b) Rs. 2,500 crore (c) Rs. 4,500 crore (d) Rs. 7,400 crore (e) Rs. 10,000 crore

222. Plotter are very useful in applications such as. (a) Computer aided design (b) Word Processing (c) Financial Accounting (d) Spell checking (e) None of these

223. Employees details is. (a) A master file for the payroll system (b) A replica of the monthly pay slip (c) Not related to the payroll system

The information provided here is only for refrence. It may vary from the original.

www.recruitment.guru

(d) A transaction file for monthly pay slip (e) None of the

224. A computerized business information system includes (a) Hardware (b) Software (c) Data facts (d) All (e) None of these

225. Data processing cycle consists of – (a) Input cycle and output cycle (b) Output cycle and processing cycle (c) Input cycle, output cycle and processing cycle (d) Only input cycle (e) None of these

The information provided here is only for refrence. It may vary from the original.

www.recruitment.guru

Answers

The information provided here is only for refrence. It may vary from the original.

www.recruitment.guru

Explanations

(1-5)

Person City Specialisation P. Hyderabad Literature Q. Calcutta Computers R. Mumbai Marketing S. Chennai Textile/ Economics T. Delhi Textile / Economics V Trivandrum Physics X. Bangalore Information Technology

(6-10). After careful analysis of the given passcodes for successive batches we can conclude

that the passcode for the next batch is generated by rearranging the words of the passcode of

previous batch in the following manner:

BATCH I ( 9 AM TO 10 AM)

Passcode:

1 2 3 4 5 6 7

Course easy set for each year was

BATCH II ( 10 AM to 11 AM)

Passcode:

2 5 1 4 7 3 6

Easy each course for was set year

1 2 3 4 5 6 7

BATCH III (11 AM to 12 Noon)

Passcode:

2 5 1 4 7 3 6

Each was easy for year course set

1 2 3 4 5 6 7

BATCH IV (12 Noon to 1 PM)

Passcode:

2 5 1 4 7 3 6

Was year each for set easy course

1 2 3 4 5 6 7

The information provided here is only for refrence. It may vary from the original.

www.recruitment.guru

BATCH V (2 PM to 3 PM )

Passcode:

2 5 1 4 7 3 6

Year set was for course each easy

1 2 3 4 5 6 7

BATCH VI (3 PM to 4 PM )

Passcode:

2 5 1 4 7 3 6

Set course year for easy was each

1 2 3 4 5 6 7

BATCH VII (4 PM to 5 PM)

Passcode :

2 5 1 4 7 3 6

Course easy set for each year was

1 2 3 4 5 6 7

BATCH VIII (5 PM to 6 PM )

Passcode :

2 5 1 4 7 3 6

Easy each course for was set year

1 2 3 4 5 6 7

BATCH IX (6 PM to 7 PM

Passcode :

2 5 1 4 7 3 6

Each was easy for year course set

1 2 3 4 5 6 7

BATCH X (7 PM to 8 PM)

Passcode :

2 5 1 4 7 3 6

The information provided here is only for refrence. It may vary from the original.

www.recruitment.guru

Was year each for set easy course

Thus, we see that after Batch VI, the passcode for each subsequent batch is repeated starting

from the Batch I. In other words, the following pairs of Batches have the same passcode.

(a) Batch I and Batch VII

(a) Batch II and Batch VIII

(a) Batch III and Batch IX

(a) Batch IV and Batch X

On the basis of above analysis we can write any required passcode directly from the given

passcode in the following manner.

BATCH PASSCODE

I 1

Course

2

Easy

3

set

4

for

5

each

6

year

7

was

II

III

IV

V

VI

VII

VIII

IX

X

2

5

7

6

3

1

2

5

7

5

7

6

3

1

2

5

7

6

1

2

5

7

6

3

1

2

5

4

4

4

4

4

4

4

4

4

7

6

3

1

2

5

7

6

3

3

1

2

5

7

6

3

1

2

6

3

1

2

5

7

6

3

1

6. BATCH IV (12 Noon to 1 PM)

7 6 5 4 3 2 1

She the girl is clever very good

BATCH VI (3 PM to PM)

3 1 6 4 2 7 5

Clever good the is very she girl

7. The passcode for the batch IX will be similar to the passcode for the batch III.

The information provided here is only for refrence. It may vary from the original.

www.recruitment.guru

8. BATCH (11 AM to 12 Noon )

5 7 2 4 6 1 3

Pin to the point is sharp not

BATCH V ( 2 PM to 3 PM)

6 3 7 4 1 5 2

Is not to point sharp pin the

9. BATCH II (10 AM to 11 AM )

2 5 1 4 7 3 6

For the life is good change got

Passcode :

6 3 7 4 1 5 2

got change good is life he for

Cleary, this is the passcode for Batch V.

10. BATCH IV (12 Noon to 1 PM)

7 6 5 4 3 2 1

Do how will the you job now

BATCH II (10 AM to 11 AM)

2 5 1 4 7 3 6

Job will now the do you how

11. It is clear from the statement that both the assumptions are implicit. The statement clearly

implies the first assumption. Again, if it is not desirable complete issue why such statement

would have been made.

12. Only assumption II is implicit in the statement. If two months period is not sufficient to

implement such policy decision, the Pensioner’s Forum would not have made such a statement.

13. Neither of the assumption is implicit in the statement. On the basis of the precedent it is

erroneous to hypothesise that the flyovers do not serve any public objective.

14. Both the assumptions are implicit in the statement. The point which is highlighted in an

advertisement is liked by people and is also desirable.

15. Only assumption I is implicit in the statement. The traders of state ‘K’ resorted to observe

bandh when other measures or efforts failed to resolve the problem. Note down the use of

The information provided here is only for refrence. It may vary from the original.

www.recruitment.guru

phrase “the state has failed “ . It implies that the State ‘K’ made efforts to resolve the issue but

failed. It does not imply that it is not interested in resolving the problem.

(16-20). First of all define each symbol clearly:

I. A + B means A < B or A > B

It implies that A = B

II. A x B means A = B or A < B

It implies that A > B

III. A? B means A > B or A = B

It implies that A < B

IV. A @ B means A > B

V. A $ B means A = B

It implies that A > B or A < B

16. Statements

C + D, D $ M, M? C

After conversion

C = D, D > M or D < M, M < C

Or (i) C= D > M < C

Or

(ii) C = D < M < C

Conclusions

I . M $ C = M < C or M < C

It is clear that M is smaller than C.

II. D + M = D = M

It is clear that D is either greater or smaller than M.

17. Statements

K $ M, M x B, K + B

After conversion

K > M or K < M, M > B, K = B or, M > B = K

Conclusions

I . M x K = M > K

K is equal to B and M is greater than B. Therefore, M is definitely greater than K.

II. K? M = K < M

The information provided here is only for refrence. It may vary from the original.

www.recruitment.guru

It is clear that K is

18. Statements

F @ G, G? P, P x H

After conversion

F > G, G < P, P > H

Or (i) F > G < P > H

Or

(ii) F = G < P > H

Conclusions

I. F @ H = F > H

II. H x G = H > G

Neither conclusion I nor II is definitely true.

19. Statements

M @ T, T x R, R $ Q

After conversion

M > T, T > R, R > Q or R < Q

Or (i) M = T > R > Q

(ii) M > T > R > Q

(iii) M > T > R < Q

(iv) M = T > R < Q

Conclusions

I . M ? R = M < R

It is clear that M is greater than R.

Therefore, conclusion I is not true.

II. M ? Q = M < Q

From statements (i) and (ii) M is greater than Q builts is not possible to determine any definite

relation between M and Q from statements (iii) and (iv).

20. Statements

D x G, G @ K, K + M

The information provided here is only for refrence. It may vary from the original.

www.recruitment.guru

After conversion

D > G, G > K, K = M

Or (i) D > G > K = M

Or

(ii) D > G = K = M

Conclusions

I . M + G = M = G

M is either equal to or smaller than G.

II. D x K = D > K

It is clear that D is greater than K.

Therefore, only conclusion II is definitely true.

21. From statement I

D B C N

NW NE

A W E

SW SE

S

It is clear that village C is to the North- East of village A

From statement

A

K

P E

We cannot solve the question with the help of this diagram.

22. From both the statements it is clear that if Rohan retires from office ‘X’ in January 2006 he

could not get full pension benefits.

23. From statements I

The information provided here is only for refrence. It may vary from the original.

www.recruitment.guru

S > T > P and Q

From statements II

> > > > R

From both the statements

S > T > P and Q > R

Either P or Q will come at the third place.

24. From Statements I.

P is the husband of R, who is the daughter of J.

There is no information about the other daughter of J.

From statements II

Q is the mother of S and R.

S is daughter of Q and sister of R.

From both the statements

J is the husband of Q

Q is the wife of J.

J is father of R and S

R and S are daughter of J and Q.

Q is the mother of R and S

P is the husband of R and hence P is the son-law of Q.

25. From statement I.

De fu pane = rose flower is

Quiz = beautiful tree

No answer.

From statement II

De la = rose flower

Pa = tea

Both the statements provide different coding pattern.

la beautiful

la

Chin red

Chin red

The information provided here is only for refrence. It may vary from the original.

www.recruitment.guru

26. It is not judicious to regard asbestos industry as one of the largest industries in India

considering the number of employees engaged in this industry and also the number of units of

this industry in India.

27. It is said that asbestos industries in India are located in several states.

28. Advanced countries are banning cancer causing asbestos products: it imples that advanced

countries are concerned and careful to product health hazards o their people.

29. It can be safely assumed that if the industry is growing in India there certainly exists

demand for its products.

30. Consider the statement “… indifference of government machinery….”

36- 38 4 cm BLUE

3 cm

GREEN 5 cm

5 cm

3 cm

4 cm

36. The cubes located at the eight corners would have all the three colours.

37. The six inside cubes will have no face coloured.

38. Twelve cubes will have two colours red and green in their two sides.

39.

10th Day Saturday

3rd Day Saturday

Now, 2nd Day Friday

1st Day Thursday

40. A T M O S P H E R E

1, 2 (3) 4,5 6 7,8,9 10

A

Saturday

A B C D

2 Days 3 Days 1 Day 3 Days

RED

The information provided here is only for refrence. It may vary from the original.

www.recruitment.guru

Thus, there are two such pairs of letters.

41.

Gold

The shaded portion represents doubtful proposition.

Since two of the premises are particular, the Universal conclusions can be discarded.

Therefore, conclusions I, II and IV are invalid.

For conclusion III, the second and the third premises are relevant. We can align the premises

by converting the third premise and changing the order of the premises. Thus,

Some gold are rings

No ring is paint

We know that

I + E = O type conclusion

Therefore, our derived conclusion would be :

“Some gold are paints”.

Similarly, we can take the first and the second premises. Thus,

Some ice are rings

No ring is paint

We know that,

I + E = O type conclusion

Thus, our derived conclusion would be:

“Some ice are not paints”.

42.

lce Ring

Paint

Clip

Fruits Flowers

Gates

The information provided here is only for refrence. It may vary from the original.

www.recruitment.guru

For conclusion I the first and the second premises are relevant. We can align these premises

by converting the second premise and changing the order of the premise.

Some fruits are gates

All gates are flowers

We know that,

I + A = I type conclusion.

Thus, our derived conclusion would be:

“Some fruits are flowers”

Conclusion I is the conversion of our derived conclusion.

Since all the premises are Affirmative, no Negative conclusion can be derived from them.

Therefore, conclusion IV is invalid.

43. From the first to premises we can derive the conclusion:

“Some shoes are not candles”

No candle is bell

Some bells are shoes

We know that,

E + I = o, type conclusion

The shaded portion implies, doubtful proposition.

Conclusion I and II from complementary pair. Therefore, either conclusion I or II follows.

Shoes

Tables Bell

Candle

The information provided here is only for refrence. It may vary from the original.

www.recruitment.guru

Similarly, conclusions III and IV form complementary pair.

44. All the three premises are Affirmative and hence the Negative conclusion is invalid.

Therefore, conclusions I dose not follow.

Conclusion III is the conversion of the second premise.

45.

For conclusion III the first and the second premises are relevant Both the premises are already

aligned

Some lice are slates

All slates are apples

We know that

I + A = I type conclusion would be

“Some lice are apples”.

Conclusion III is the conversion of the derived conclusion.

Conclusion II and IV form complementary pair. Hence conclusion II or IV follows.

46. The given number ends with an even number and hence the last even number will be symbolised as © 6 7 3 2 5 8

• + ? * $ © . 47. 2 3 6 4 7 5

Clouds Rats

Chairs

Films

Lice Slates

Apples Car

The information provided here is only for refrence. It may vary from the original.

www.recruitment.guru

* ? • ÷ + $ 48. 8 4 6 7 2 1 ! + • + * x 49. The number begins with odd number and hence 1 will be written as @ and the number ends with even number and hence 4 will be written as © 1 7 8 5 2 4 @ + ! $ * © 50. The given number ends with even number and hence 6 will be written as ©. 2 5 4 8 6 * $ ÷ ! © 51. According to question. 589 = 559 = 955 362 = 332 = 233 554 = 524 = 425 371 = 341 = 143 442 = 412 = 214 Descending order 955 > 425 > > 214 > 143 Middle Number 23 Last digit 52. There are 11 letters each in the given word and its code. It means that there is one code letter for each letter of the given word. After careful observation of the given word and its code it is clear that the code has been generated by rearranging the letters of the given word. The middle letter of the given word remains at its position while other letters of both sides of it are rearranged in a certain way. 1 2 3 4 5 1 2 3 4 5 G E R M I A T I O N The letter in each half have been rearranged in the order: 54132 Now, G E R M I A T I O N I M G R E N O A I T Similarly, 1 2 3 4 5 1 2 3 4 5 E S T A B I S H E D B A E T S D E I H S Therefore, our required answer is option (b). 53. There is one letter more in the code as compared to the letters of the given word.

233

3

N

N

N

L

L

The information provided here is only for refrence. It may vary from the original.

www.recruitment.guru

The pattern off coding is as follows: P +2 P R O +2 Q U +3 X L +2 N T + 2 V R +2 T Y Y Similarly, T T +2 V R +2 T E +3 H A +2 C S +2 U O +2 Q N N Therefore, our required answer is option (d). 54. According to question the new sequence would be : ACEGIKOQS WYZXVTRPNLJHFDB 11th from left 6th to the left of 17th left means (17-6) =11th letter left i.e. 21st letter from left in the original sequence. 55. 1 3 6 7 8 9 10 12 T L I S H M N Specified letters : S, A , B, E. T Meaningful words: BEAST, BEAST (56-60) : The given information can be analysed as:

Candidate CRITERIA

U

2

S

4

S

5

B

11

E

13

T

The information provided here is only for refrence. It may vary from the original.

www.recruitment.guru

56. Deepali Mirza satisfies all the criteria except (ii but she does satisfy (a) instead.

Therefore, her case may be referred to the Director.

57. In case of Ajit D’souza no information is given regarding the criterion (iv).

58. Rajendra Bhave satisfy all the criteria except (iv) but he does satisfy (c) instead. Therefore,

his case may be referred to the President.

59. In case of Harish Chheda complete information (Mark in Graduation) is not given regarding

the criterion (ii) or (a).

60. Tarique Answer satisfy all the criteria.

61. The movement and changes of designs in the subsequent figures can be shown as:

From Problem Figure (1) to (2)

New Design

From Problem Figure (2) to (3)

New Design

These two steps are repeated alternately and a new design appears in the centre in each subsequent figure. 62. In the subsequent figures respectively one, one and half and half part (s) of triangle get (s) shaded in a set pattern.

or (iii)

or or

(i) (ii) (a) (iv) (c) (v) (b)

Deepali √

__ √

__ √

__

Ajit √

__ √

NG __ √

__

Rajendra √

__ √

__ √

__

Harish √

N G √

__ √

__

Tarique √

__ √

__ √

__

• •

• •

• •

• •

The information provided here is only for refrence. It may vary from the original.

www.recruitment.guru

63. From Problem Figure (1) to (2) the lower design ascends vertically and it is replaced by a new design, the left design moves anticlockwise and the right design moves to the left. Similar changes would occur from Problem Figure (5) to Answer figure. 64. From Problem Figure (1) to (2) the line segments at the left end rotate through 45˚ anticlockwise while that of upper line segments at the right end rotate through 45˚ anticlockwise. Similar changes occur from Problem Figure (3) to (4) and from Problem Figure (5) to Answer Figure. 65. From Problem Figure (1) to (2) all the designs moves half a step in clockwise direction and the first and the last designs interchange position while a new designs in introduced in front of the per-existing designs. From Problem Figure (2) to (3) all the designs move one step in clockwise direction and again the first and the last designs interchange position so as the two middle designs and a new design is introduced in front of the pre-existing designs. From Problem Figure (3) to (4) all the designs move 1½ step in clockwise direction and the first two designs interchange position with the last two designs and a new design is introduced in front of the pre-existing design. From problem Figure (4) to (5) all the designs move two step in clockwise direction and the first and the last designs have interchanged position so as the second and the two middle design have interchanged position while a new design is introduced in front of the pre-existing designs. Therefore, from Problem Figure (5) to Answer Figure all the designs would move two and one half steps in clockwise direction and also rearrangement of designs would take place and a new design will be introduced in front of the pre-existing designs. 66. Following changes occur from first figure to second figure: New Design 67. Second figure is the mirror image of the first figure. 68. From first figure to second figure black leaflet rotates through 45˚ anticlockwise and the

white leaflet rotates through 90˚ clockwise. 69. From first figure to second figure the line segment with triangle rotates 90˚ clockwise

while the line segment with circle rotates 135° anticlockwise. 70. From first figure to second figure the diagonally opposite designs interchange position. 71. The main design rotates 90° anticlockwise. The inner and outer line segments move two steps anticlockwise. 72. Except in Figure (2) in all other pairs of figures the shaded portion ascends or descends. 73. Except in figure pair (3) in all other pairs of figures from element I to II the number of arrows is decreased by one while the number of equal signs is increased by one. Again the

extreme left arrow rotates 135° clockwise but n figure (3) it has been rotated through 180° clockwise. 74. The smaller designs rotates 90° anticlockwise and the smaller designs become larger while the larger designs become smaller from element I to II Again the right design is inverted. 75.In all other pairs of figures there is one unshaded triangle to the left of shaded triangle.

• •

• •

The information provided here is only for refrence. It may vary from the original.

www.recruitment.guru

85. The word assiduously (Adverb) comes from ‘the words assiduous (Adjective). Assiduous means working hard and showing careful attention to detail. For example They planned their careers and worked Assiduously to see them achieved. On the other hand deliberately means intentionally carefully. So, we can choose option (d) as the answer. 86. Idiosyncrasy means a person’s particular way of thinking, behaving, etc. that is clearly different from that of others. For example One of her little idiosyncrasies is always washing in cold water. As ideologies is closest in meaning to it so the answer is (d) 87. The lexical meaning of shoddy (Adjective) is of poor quality, done or made badly: Shoddy goods/workmanship. In the passage Shoddy means cheap. 88. Inducement (Noue) means a thing that persuades somebody to do something, an incentive. For example, Estate agents are offering inducements to first time buyers. In the passage also inducements means incentive. As impediment means barrier, hurdled, so the answer is (e) 89.Unscrupulous means without moral principles; not honest or fair unscrupulous business methods. 90. Contemptuous (Adjective) means feeling or showing contempt. Contemptuously (Adverb) means with contempt. (91- 95). First sentence (A) The application of economic, environmental and consumer pressures have on an increase in recent years. Second sentence (F) These have greatly influenced the development of the agriculture and food industries in our country. Third sentence (D) In turn, the support industries have developed to an equally advanced state. Fourth sentence (E) They have also equipped themselves with the necessary expertise to satisfy the most exacting requirements of the overseas markets Fifth sentence (C) They are thus able to provide expertise and technology to satisfy the needs of agro food production Sixth sentence (B) As a result, our agro- food production and technology are amongst the most advanced in the world 96. I did neither should be replaced by Nor did I. Structure would be: For example … + neither/nor + auxiliary verb + subject

The information provided here is only for refrence. It may vary from the original.

www.recruitment.guru

Ruth didn’t turn up and Kate did neither,(x) Ruth didn’t turn up and nor did Kate.(√) Auxiliary Subject Verb 98. The sentence is in Past Perfect continuous. Hence extorted should be replaced by extorting. 99. Infinitive without ‘to’ is used after need. For example, He wondered whether they need send a deposit He wondered whether they need to send a deposit (x)

Infinitive with ‘to’ He wondered whether they need send a deposit(√) Infinitive without ‘to’ 111. generally an objective is made when a proposal is founded on a wrong principal and is inconvenient. 112. Journalists don’t select; they judge. So, it is said- “The criterion for judging a player..” “Recent” and “earlier” denote different periods of time. So, option (b) is the correct one. 113. One devotes oneself in public affairs. So, we should use devoted in the first blank. Without taking a holiday denotes continuous services. So, without is the appropriate use in the second blank. 114. Sings of industry is an indication of prosperity. So, we can choose option (d) 115. One can be arrested on a particular charge of theft. But he can be released for the lack of evidence. 126. A. Total marks in 4 subject including English = 4 x 60 = 240 B. Total marks in English and Maths = 170 C. Total marks in Maths and Science =180 The question can’t be answered because nothing has been said about the marks in the fourth subject. Also, there are four unknowns but any three equations can be formed with given data. 127. Let the marked price be Rs. x A . Cost price (1 -0.15)x = Rs. 0.85x B. S.P. = Rs. 3060 C. Profit = 2% of x = 0.02x Profit earned on the cost price

= 0.02x x 100 = 2.35% 0.85x 0.02x = 3060 – 0.85x 0.87x = 3060 x = 3060 0.87 Actual profit = 0.02x

= 0.02 x 3060 = Rs. 70.34

0.87 128. A. x-y = 6

The information provided here is only for refrence. It may vary from the original.

www.recruitment.guru

B. 0.4y =0.3x X = 4 Y 3

C. x : y =2:1

2 3

x x 3 = 2

y 2 1

x = 4

y 3 B and C give the same expression/information and hence are equivalent.

x = 4 y

3 x- y = 6

4 y – y = 6

3 4 =6 3 y = 18 x = 4 x 18 = 24 3 129. Hypotenuse = √52 + 122

=√25 + 144 =√169 = 13 Base : Height : Hypotenuse = 5 : 12 : 13 Base : Height : Hypotenuse = 30 cm :. Base = 5 x 30 = 5 cm 5 + 12 +13 Height = 12 x 30 = 12 cm 5 + 12 + 13 Area = 1 x base x height 2 = 1 x 5 x 12 = 30 cm2

2 130. The question cannot be answered because R’s share in investment is not given. 131.Percentage of candidates selected over appeared in 2000.

25 x 100 = 2.94% A = 850

35 x 100= 3.33% B = 1050

28 x 100 = 2.83%

C = 990

36 x 100 = 3.33%

The information provided here is only for refrence. It may vary from the original.

www.recruitment.guru

D = 1080 40 x 100 = 3.57% E = 1120 The percentage is lowest for state C. 132. Passing percentage over appeared for state D in:

300 x 100 = 27.7%

20001080

320 x 100 = 27.8%

2001--> 1150 280 x 100 = 28.1% 2002--> 995 260 x 100 = 26.67% 2003 --> 976 240 x 100=27% 2004 --> 888 216 x 100 = 25% 2005--> 864 The required percentage is highest in the year 2002. 133. Total no of candidates selected from state ‘A’ = 25 + 20 + 22 + 36 + 32 +28 = 163 Total no. of candidates selected from state ‘B’ = 35 + 30+ 28 + 32 + 40+ 38 =203 :. Required % = 163 x 100 = 80% 203 134. Percentage of candidates selected over passed for state ‘B’ in

35 x 100 = 14.3%

2000 --> 245 30 x 100 = 13 % 2001 --> 230 28 x 100 = 13.3% 2002--> 210 32 x 100= 16.8% 2003--> 190 40 x 100 = 17.8% 2004 --> 225 38 x 100 = 19% 2005--> 200 Required percentage is lowest in 2001 135. No. of candidates passed from state A in 1996 = 240 No. of candidates passed from state E in

The information provided here is only for refrence. It may vary from the original.

www.recruitment.guru

1999= 280 :. Required ratio = 240 : 280 = 6 : 7 136. Production of company C in 1999 = 45 lakh tonnes Production of company A in 2004 = 58lakh tonnes :. Required difference = 50- 45 = 5 lakh tonnes 137. Required percentage 55 -40 x 100 = 75 = 37.5% 40 2 139. Total production of company C in 2001 and 2002 = 120 lakh tonnes Total production of company A in 1999 and 2000 = 90 lakh tonnes :. Required percentage = 120 x100 90 = 133 1% 3 140. Average production of company A = 50 + 40 + 55 + 45 +60 + 50 6 = 300 = 50 lakh tonnes 6 Average production of company B = 55 + 60 + 50 + 55 +50 + 55 6 = 325 = 54.17 lakh tonnes 6 Average production of company C = 45 + 60 + 60 + 45 + 40 6 = 300 = 50 lakh tonnes 6 :. Required difference = 54. 17 – 50 = 4.17 lakh tonnes. 141. No. of students who left in the beginning of 2002 = 400 No of students who joined in the beginning of 2002 = 500 :. There was net increase of 100 (=500 – 400) students in the strength of the school from 2001 to 2002. 142. We can come to this conclusion even without performing complete exact calculation, simply by looking / comparing the terms written within the brackets. 143. Total no. of students who joined till 2003 = 300 + 250 +500 + 450 = 1500 Total no. of student who left till 2003 = 250 + 350 + 400 +300 = 1300 Net increase in the strength of the school = 1500 – 1300 = 200 Initial strength of the school in 1999 = 1500 :. Strength of school during 2003 = 1500 + 200 = 1700

Year Strength

1999 1500 2000 1500 + 300 – 150 = 1550

The information provided here is only for refrence. It may vary from the original.

www.recruitment.guru

144. Strength of the school was equal in 2000 and 2002. 145.From solution to Q. No. 159 we have no. of students in 2003 = 1700 and students in 2001 = 1450 :. Required % = 1700 x 100 1450 = 117% (approx.) 146. The given series is based on the following pattern: 9 x 2 + 1. 5 = 19. 5 19.5 x 2 + 2 = 41 41 x 2 + 2.5 = 84. 5 Therefore, the new series is as follows: 12 x 2 + 1.5 = 25. 5 ... (a) 25.5 x 2 +2 = 53 … (b) 53 x 2 +2.5 = 108.5 …. (c) 108.5 x 2 + 3 = 220 ... (d) 220 x 2 +3.5 = 443.5 .... (e) Therefore, the number 108.5 will come in place of (c) in the new series. 147. The series is based on following pattern 4 x 1 + 1 = 5 + 3 5 x 4 + 2 = 22 + 5 22 x 9 + 3 =201 Similarly the new series is as follows: 7 x 1 + 1 = 8 ..(a) 8 x 4 + 2 = 34 .. (b) 34 x 9 + 3 = 309 .. (c) 309 x 16 + 4 = 4948 ..(d) Therefore, the number 4948 will come in place of (d) in the new series 148. The series is based on following pattern: 5 x 1 + 0.25 x 1= 5.25 + 3 5.25 x 2 + 0.25 x 4 = 11.5 + 5 11.5 x 3 +0.25 x 9 =36.75 Similarly, the new series is as follows: 3 x 1 +0.25 x 1 = 3 .25 ... (a) 3.25 x2 + 0.25 x 4 = 7.5 ...(b) 7.5 x 3+0.25 x 9 = 24.75 ...(c) Therefore, the number 24.75 will come in place of (c) in the new series.

2001 1550 + 250-350 =1450

2002 1450 + 500- 400= 1550

2003 1550 +450 – 300 =1700

2004 1700 +400 -500=1600

2005 1600 +550 – 500= 1650

The information provided here is only for refrence. It may vary from the original.

www.recruitment.guru

149. The series is based on following pattern: 38 x 0.5 = 19 19 x 1.5 = 28.5 28.5 x 2.5 = 71.25 Similarly, the new series is as follows: 18 x0.5 =9 … (a) 9 x 1.5 = 13.5 … (b) 13.5 x2.5 =33.75 ...(c) 33.75 x 3.5 = 118.125.. (d) Therefore, the number 118.125 will come in place of (d) in the new series. 150. The series is based on following pattern: 25 + (11)2 = 25 + 121 = 146 146 – (9)2 = 1446 -81 = 65 65 + (7)2 = 65 + 49 =114 Similarly, the new series is as follows: 39 + (11)2 = 39 + 121 = 160 … (a) 160- (9) 2 = 160 -81 = 79 …(b) 79 +(7)2 = 79 + 49 = 128 …(c) 128 + (5)2 = 128 – 25 = 103 .. (d) 103 +(3)2 = 103 + 9 =112 ..(e) Therefore, the number 112 will come in place of n(e) in new series. 151. There are 9 women and 8 men. A committee of 12, consisting of at least 5 women, can be formed by choosing: (i). 5 women and 7 men (ii) 6 women and 6 men (iii) 7 women and 5 men (iv) 8 women and 4 men (v) 9 women and 3 men :. Total number of ways of forming the committee = 9C5 x 8C 7 + 9 C6 x 8C6 + 9C7 x 8C5 + 9C8 x 8C4 + 9C9 x 8C3 = 126 x 8 +84 x 28 + 36 x 56 + 9 x 70 + 1 x 56 = 6062 152. Women are in majority in (iii), (iv) and (v) cases as discussed in question 151. :. Total number of such committees = 9C5 x 8C5 + 9C8 x 8C4 + 9C9 x 8C3

= 36 x 56 +9 x70+1 x 56= 2702 153. Men are in majority in only (i) case as discussed in question 151. :. Total number of such committees = 9C5 x 8C7 = 126 x 8 = 1008 154. There are 20 balls in the turn out of which 2 balls can be drawn in 20C2 ways :. Total number of elementary events = 20C2 = 190 There are 9red balls out of which 2balls can be drawn in 9C2 ways. :. Favourable number of elementary events = 9C2 = 9 x 8 = 36 1 x 2 :. Required probability 36 = 18 190 95 155. There are 9 letters in the word ALLAHABAD out of which 4 are As, 3 are Ls and the rest are all distinct. So, the requisite number of words

The information provided here is only for refrence. It may vary from the original.

www.recruitment.guru

= 9! = 7560 4! 2! 156. No. of A type of employees in 2006. = 0.22 x 48640 = 10700 In 2005, A = 0.20 x 42980 = 8596 B = 0.06 x 42980 = 2579 C = 0.10 x 42980 = 4298 D = 0.15 x 42980 = 6447 E = 0.27 x 42980 = 11605 F = 0.22 x 42982 = 9455 C + D = 4298 + 6447 = 10745 157. In 2006, A : 10700 B : 0.10 x 48640 = 4864 C : 0.11 x 48640 = 5450 D : 0.09 x 48640 = 4377 E : 0.27 x 48640 = 13133 F : 0.21 x 48640 = 10214 % change during 2005-2006 A : 10700 – 8596 x 100 = 24.5% 8596 B : 4864 -2579 x 100 = 88.6% 2579 C : 5350 – 4298 x 100 = 24.5% 4298 D: 6447 – 4377 x 100 = 32.1% 6447 E : 13133 – 11605 x 100 = 13.2% 11605 F : 10214 – 9455 x 100 = 8% 9455 The % change was maximum for B. 158. 4864 – 2579 = 2285 159. 5000 x 100 = 10.3% =10% 48640 160. 10700 x100 = 125% 8596 161. In 36 days 12 men can do 1 complete work.

In 36 days 12 women can do ¾ th of the work. Since time and the no. of persons is the same in both cases.

1 women’s daily work = ¾th of 1 men’s daily work\ 8 women’s daily work

= ¾ x 8 = 6 men’s daily work

(10) men + 8 women daily work) = (10 men + 6 men) = 16 men’s daily work. 12 men can do the work in 369 days :. 16 men can do the work in 36 x 12 = 27 days 16

The information provided here is only for refrence. It may vary from the original.

www.recruitment.guru

162. Increase is interest in 3 years due to increase in rate by 4% = 800 x 3 x 4 = Rs. 96 100 Total amount at the end of 3 years = Rs . 956 + Rs. 96 = rs. 1052 163. 427398 = 15 x 28493 +3 :. The least number which should be subtracted from 427398 so that it becomes divisible by 15 = 3. 164. Initial speed = 80 km/hr Total distance = 80 x 10 = 800 km. Now speed =800 =200 km/hr 4 Increase in speed = 200 – 80 = 120 km/hr. 165. 33 1% of 768.9 +25% of 161.2 – 58.12 3 = 100 x 788. 9 + 25 x 161.2 -58.12 3x100 100 = 256.3 + 40.3- 58.12 = 238.48 = 240 (approx.) 166. Profit = Selling price of 4 notebooks Cost price = selling price of (12 -8) = 4 notebooks. :. %profit = 4 x 100 =50 8 167. 3 years ago Ritu’s age = x :. Ritu’s present age =x + 3 Rahul’s present age = Ritu’s present age -5 = x + 3 -5 = x – 2 168. Mixture : 2 kg of rice at Rs. 15 kg + 3 kg of rice at Rs. 13kg Total weight = 2 + 3 = 5 kg Total cost price = (2 x 15) + (3 x 13) = 30 + 39 = Rs. 69 Cost price per kg of the mixture = 69 = Rs. 13.80 5 Selling price to get 33 1 % profit 3 100 + 33 1 3

= x Rs. 13.80

= 400 x Rs .13.80 3 x 100 = 4 x Rs. 13.80 3 =Rs. 18.40 169. Total time = 6 hours. Speed of the both in still water = 4 km/hr let the distance between M and N be D and the speed of the stream be x. D 1 + 1 = 6 4+ x 4 - x

The information provided here is only for refrence. It may vary from the original.

www.recruitment.guru

D 4 – x + 4 + x = 6 (x + x) (4- x) D 8 = 6 42 - x2

8D = 6 16 – x2

D = 6 (16 –x2 ) = 3 (16 –x2) 8 4 Since the speed of the stream (x) is not given, the distance D cannot be determined. 170. Let Mr. Yadav’s annual salary be x. Amount spent on consumables = 0.80x Amount spent on clothes and transport = 0.50 (x-0.80x) = 0.50 x 0.20x = 0.10x Savings = x – 0.80x – 0.10x = 0.10x :. 0.10x = Expenditure on clothes and transport = Rs. 5370 :. Monthly expenditure =5370 12 = Rs. 447.50

171. I. p2 + 24 = 10p

p2 - 10p + 24 = 0 p2 – 6p -4p +24 =0 p(p-6) -4 (p – 6) = 0 (p -6) (p – 4) =0 P = 4, 6 II. 2p2 + 18 = 12q q 2 + 9 = 0 q2 – 6q + 9 =0 (q -3)2 =0 q = 3 Thus, p > q 172. I. pq + 30 = 6p+ 5q 6q + 5q – pq = 30 p + q - pq = 1 5 6 20 P + q (p + q) = 1 5 6 5 6 :. P = 5 and q =6 q > p 173. I. q2 + q = 2 q2 + q - 2= 0 q2 - q + 2q – 2 =0 q(q-1) + 2(q-1) =0 (q-1) (q+2) = 0

The information provided here is only for refrence. It may vary from the original.

www.recruitment.guru

q = -2, 1 p2 +7p + 10= 0 p2 + 5p + 10 =0 p2 + 5p + 2p +10 =0 II. p(p+5) +2(p+5) = 0 (p+5) (p+2)=0 P = 5, -2 Thus, q > q 174. I. p2 + 16=8p p2 - 8p + 16 = 0 (p-4)2 =0 P =4 II. 4q2 + 64 = 32q q2 + 16 = 8q q2 -8q + 16 = 0 (q -4)2 =0 q =4 Thus, p=q 175. 2p2 + 12p + 16 =0 p 2 +6p +8 =0 p2 +2p + 4p +8=0 p(p+2) +4(p+2)=0 (p+2)(p+4)=0 P = -2, -4 II. 2q2 + 14q +24 =0 q2 +7q +12=0 q2 +3q +4q +12=0 q(q+3)+4(q+3) =0 (q +3)(q+4) =0 q=-3, -4, Thus,, p> q